تغطية شاملة

كم يزن الكيلو جرام؟

من المقرر أن تنتهي الجهود المبذولة للتخلص من الأسطوانة المعدنية التي تعود إلى القرن التاسع عشر والتي لا تزال تحدد الكيلوغرام في عام 19

نسخة طبق الأصل من الكيلوجرام الدولي القياسي. المصدر: المعهد الوطني للمعايير والتكنولوجيا.
نسخة طبق الأصل من الكيلوجرام الدولي القياسي. مصدر: المعهد الوطني للمعايير والتكنولوجيا.

بقلم تيم فولجر، تم نشر المقال بموافقة مجلة ساينتفيك أمريكان إسرائيل وشبكة أورت إسرائيل 13.04.2017

  • منذ عام 1889، تم تعريف الكيلوجرام على أساس وزن أسطوانة واحدة من سبائك البلاتين والإيريديوم محفوظة في قبو بالقرب من باريس. هذا هو المقياس الوحيد الذي لا يزال مرتبطًا بجسم مادي.
  • لكن الكيلوجرام الأصلي يفقد كتلته. وهذا أحد أسباب القرار الذي اتخذه المؤتمر الدولي للأوزان والمقاييس عام 2011 بإعطاء الكيلوجرام تعريفًا جديدًا سيكون مرتبطًا بالثابت في ميكانيكا الكم.
  • وفي عام 2017، دخلت عملية إعادة التعريف مرحلتها النهائية. تشارك مختبرات القياس الرسمية في خمس دول في هذه العملية وتتضمن بعضًا من أصعب القياسات العلمية التي يجب إجراؤها.

جون برات وشعر بالتوتر يتصاعد داخله لدى اقترابه من نقطة التفتيش الأمنية في مطار دالاس الدولي في واشنطن. وكان في حقيبة الكاميرا التي كان يحملها أربع أسطوانات معدنية. مثل هذه الأشياء لا يمكن أن تفشل في جذب انتباه أفراد الأمن المشبوهين. وكان وزن كل اسطوانة كيلوغراما واحدا بالضبط. وبلغت قيمة إحداها، وهي أسطوانة من سبائك البلاتين والإيريديوم بحجم نصف علبة، 40 ألف دولار على الأقل. (سعر البلاتين يحوم الآن حول 1,000 دولار لكلأونصة، وهو مقياس شائع للمعادن الثمينة.) وكانت الثلاثة الأخرى عبارة عن أسطوانات من الفولاذ المقاوم للصدأ تم إنتاجها بعملية ميكانيكية دقيقة.

وكانت مهمة برات هي إيصالهم إلى زميله في ضواحي باريس بأمان ودون أن يلمسهم أحد في الطريق.

كان لدى برات وثائق منمعهد المعايير في الولايات المتحدة الأمريكية والتي تهدف إلى تسهيل اجتياز الفحوصات الأمنية عليه. وأوضحت الوثائق أنها كانت تحمل أربعة كيلوغرامات أمريكية رسمية: وهي الكتل المستخدمة كأساس لجميع قياسات الوزن الأمريكية. كما أوضحوا أنه يمنع لمس الكيلوجرامات أو إخراجها من العبوات التي تحميها.

برات، وهو موسيقي سابق في موسيقى البانك روك، يدير قسم قياس الكم في مختبرات المعهد في غايثرسبيرغ، ميريلاند. "سبب لي رجل الأمن في المطار بعض المشاكل، ولكن بعد قراءة جميع الوثائق أصبحت القصة هي التي صنعت يومه." وبعد بضع دقائق، سُمح لبرات بمواصلة طريقه واستقل الطائرة التي تستغرق سبع ساعات إلى باريس، حيث كان عليه أن يواجه معضلة أخرى: ماذا سيفعل بأمتعته الثمينة عندما يضطر إلى النهوض من مقعده؟ فهل يحتفظ بالقضية معه دائما كما نصحه بعض زملائه؟ يقول برات: "أعترف أنني تركت الحقيبة تحت المقعد أمامي أثناء ذهابي إلى الحمام". "لقد غاب عن عيني لفترة قصيرة، كان من الممكن خلالها أن يلمس أحدهم الجنيه ويملأه بالتراب".

كان من الممكن أن يؤدي لمس الأسطوانات إلى إفساد عدة أشهر من العمل الدقيق المكرس لقياس هذه الكيلوجرامات بدقة تصل إلى بضعة أجزاء في المليار. كان برات في طريقه مع الأسطوانات إلى المكتب الدولي للأوزان والمقاييس (BIPM) في بلدة صور قرب باريس. وبعد بضعة أشهر قارنوا علماء الأرصاد الجوية وبين هذه الأسطوانات المعدنية وبين أسطوانات معدنية متطابقة من ثلاث دول أخرى، وكذلك بينها وبين كرة سيليكون نقية جداً تم إنتاجها فيالمختبر الوطني للمترولوجيا في ألمانيا. وكانت هذه هي الخطوة الأخيرة التي تم اتخاذها كجزء من عملية تاريخية لتغيير طريقة قياس الجماهير في العالم.

وفي عام 1889، وهو العام الذي تم فيه افتتاح برج إيفل، تم تحديد الكيلوجرام من خلال كتلة أسطوانة مصنوعة من سبيكة من البلاتين والإيريديوم، تم وضعها تحت ثلاثة أجراس زجاجية وحفظها في غرفة آمنة مقفلة في مقر المكتب الدولي. الأوزان والمقاييس. الكيلوجرام القياسي الدولي، أو Le Grand K (الكبير K)، كما يطلق عليه، هو النموذج الأولي للكيلوجرام الذي تعتمد عليه جميع معايير الكتلة الوطنية الأخرى. في نهاية عام 2018، ستفقد Grand-K مكانتها. وبدلاً من ذلك، سيحصل الكيلوجرام على تعريف جديد يعتمد على ذلك ثابت بلانك: عدد ثابت في نظرية الكم يتعلق بكمية الطاقة التي يحملها جسيم ضوئي واحد، الفوتون.

لماذا تقرر تقاعد Grand-K؟ لسنوات عديدة، أراد علماء القياس إنشاء معيار دولي دقيق وموثوق للكتلة يعتمد على ثابت أساسي في الكون بدلاً من كتلة معدنية تم حفظها من أي حارس منذ العصر الفيكتوري. ولكن هناك سببًا أكثر إلحاحًا لهذا التغيير: يبدو أن سيارة Grand-K تفقد كتلتها. مرة واحدة كل 30 عامًا أو نحو ذلك، يتم إخراج Grand-K من القبو لتنظيفه ومقارنته بست نسخ رسمية، ويتم الاحتفاظ بـ temoins ("الشهود") معه. عند مقارنة أول "شاهدين" بـ Grand-K في عام 1889، كان كلاهما مطابقًا للأصل. ولكن في القياسات التي تم إجراؤها بعد فترة وجيزة من الحرب العالمية الثانية، ومرة ​​أخرى في عام 1992، اتضح أن وزن النسخ كان أكبر قليلاً من وزن Grand-K. إن الفرضية القائلة بأن كتلة النسخ زادت للتو بينما ظل Grand-K دون تغيير تبدو غير قابلة للتصديق. هناك بالطبع تفسير أكثر منطقية: يقول: "يمكن للمرء أن يفترض أن الكيلوغرام الدولي القياسي يفقد كتلته". ميشيل ستوك مدير قسم المقاييس الفيزيائية في BIMP. يعد عدم اليقين هذا أحد أسباب القرار الذي تم اتخاذه في عام 2011 بالمؤتمر العام للمكاييل والموازينوهي الهيئة التي تدير المكتب الدولي للأوزان والمقاييس، لوضع معيار كتلة جديد.

يتم الآن معايرة الكيلوغرام الوطني الأمريكي K20 مقابل الكيلوغرام الدولي الأصلي المحفوظ في قبو بالقرب من باريس، وبعد إعادة التعريف، بدلاً من استخدام الكيلوغرام المادي، سيتحول علماء القياس إلى استخدام NIST-4. المصدر: المعهد الوطني للمعايير والتكنولوجيا .
تتم الآن معايرة الكيلوغرام الوطني الأمريكي K20 مقابل الكيلوغرام الدولي الأصلي المحفوظ في قبو بالقرب من باريس. بعد إعادة التعريف، بدلاً من استخدام العظام المادية، سيتحول علماء القياس إلى استخدام NIST-4. مصدر: المعهد الوطني للمعايير والتكنولوجيا.

لا أحد يعرف سبب فقدان Grand-K للوزن. كانت قيمته عالية جدًا بحيث لا تسمح له بالخضوع للاختبارات التي كان من الممكن أن تكتشف ذلك. هذا اللغز يخلق مشكلة حقيقية. ومع تقدم التكنولوجيا في العقود المقبلة، من المتوقع أن تصبح قياسات الكتلة على المستوى الجزيئي، وحتى على نطاق أصغر من ذلك، أمرًا شائعًا في العديد من المجالات. يقول برات: "نرغب في الحصول على طرق لقياس الكتل على نطاق ميكروجرام بدقة لا تقل عن ثلاثة أرقام، ومع وجود كيلوغرام فيزيائي قياسي، هناك الكثير من عدم اليقين على مثل هذا المقياس الصغير".

المشاكل المرتبطة بـ Grand-K لا تقتصر فقط على مجال قياسات الكتلة. وحدات القوة والطاقة مشتقة منها أيضًا. يقول ستوك: "نحن الآن في مرحلة حيث يمكننا أن نرى تغيرات في قيمة الثوابت الأساسية لأن الكيلوغرام القياسي الدولي يتغير، وهذا غير منطقي".

معيار جديد

ومن بين الوحدات الأساسية السبع للنظام المتري، فإن الكيلوجرام هو الوحدة ذات تاريخ التحديث الأقرب، ولكنها لن تكون الوحدة الأخيرة التي تتغير. بالإضافة إلى ذلك يتم تضمين الكيلوغرام فيالنظام الدولي للوحدات (SI) ومن هذه الوحدات أيضًا: المتر، الأمبير (وحدة التيار الكهربائي)، الثانية، الكانديلا (مقياس لشدة الإضاءة لمصدر الضوء)، المول (الذي يربط وزن المادة بعدد الذرات التي تحتوي عليها) والكلفن درجة (درجة الحرارة).

تمت إعادة تعريف اثنتين من وحدات النظام الدولي قبل بضعة عقود. تم إعادة تعريف المقياس في عام 1983. وحتى ذلك الحين، كان يتم تعريفه بالمسافة بين خطين محفورين في سبيكة من البلاتين والإيريديوم المحفوظة في نفس الغرفة المحجوزة والمغلقة حيث يتم الاحتفاظ بـ Grand-K، بينما في تعريفه الجديد، المتر هو المسافة التي يقطعها الضوء الجزء 299,792,458 من الثانية. بعد تحسين دقة الساعات الذرية في الستينيات من القرن العشرين، تم إعادة تعريف الثانية أيضًا. كان تعريفه القديم هو جزء من اليوم، في حين تم تعريفه الجديد من حيث التردد المحدد لإشعاع الميكروويف المنبعث من ذرة السيزيوم. مثل الكيلوجرام، ومن المقرر أيضًا تحديث المول والكلفن والحدب في عام 60.

وضع هامبر الحالي غريب بشكل خاص. يتضمن تعريفها الرسمي سلكين بطول لا نهائي، أحادي البعد وعديم الكتلة. وهذا تعريف مجرد بحيث لا يمكن إنشاؤه بدقة في المختبر. سوف يتغير هذا الوضع في عام 2018، عندما سيتم تعريف الأمبير من حيث شحنة الإلكترون. أصبح هذا التقدم ممكنًا بفضل تطوير أجهزة تكنولوجيا النانو القادرة على حساب الجزيئات المشحونة المفردة التي تمر عبر دائرة كهربائية.

"إذا فكرنا في تعريفات جديدة سيتم وضعها مستقبلا، فقد تشمل وحدة الإضاءة، الكانديلا، والتي سيتم تعريفها بدلالة ميكانيكا الكم، وربما تعريف بصري للأخيرة بدلا من تعريف الموجات الميكروية، " يقول آلان ستيل، كبير علماء القياس في كندا. "لكن هذه التغييرات لن تحدث إلا خلال 15 عامًا على الأقل، وربما أكثر."

إن إعادة تعريف الكيلوجرام هي في قلب الجهود المبذولة لإنشاء نظام عالمي حقيقي للقياسات لن يقتصر على الاتفاقيات المحلية أو تلك التي ستقتصر فقط على عالمنا الخاص. من حيث المبدأ، يمكن للكائنات الذكية أن تفهم الوحدات الجديدة في أي مكان، من هنا إلى مجرة ​​المرأة المسلسلة. هذه فترة مهمة في عالم علماء المقاييس. يقول ستيل: "يحدث هذا النوع من الأشياء مرة واحدة في العمر". "آخر مرة حاولنا فيها القيام بشيء على هذا المستوى الأساسي كانت عندما أعيد تعريف المقياس. ثق بي، في مثل هذه الأوقات يجب أن تكون كبير علماء القياس. إنه ليس مثل تحقيق السلام العالمي، ولكنه أمر رائع".

الامان

Grand-K ليس الكيلوغرام الرسمي الأول. تم إنشاء النظام السابق خلال الثورة الفرنسية، مع ولادة النظام المتري بأكمله. قبل الثورة، كانت جميع مقاييس الوزن والطول تقريبًا المستخدمة في فرنسا تعتمد على العادات المحلية. تم قبول أحجام مختلفة في مدن مختلفة وهذا الإحراج أثقل كاهل الدولة. في المجموع كان هناك أكثر من 700 وحدة قياس مختلفة في فرنسا. التوازعلى سبيل المثال، كان مقياسًا يعادلفهم تم تعريفها باللغة الإنجليزية على أنها المسافة بين أطراف ذراعي الشخص عندما تكون منتشرة على الجانبين. لكن التواز في باريس (الذي كان يساوي 72 بويز أو بوصة) لم يكن بالضرورة هو نفسه المستخدم في مرسيليا. حاول العلماء، كما كان الفرنسيون يطلقون على العلماء بينهم، إعادة النظام إلى الفوضى من خلال إنشاء نظام جديد "لجميع الأوقات، لجميع الشعوب"، كما أعلن الشعار على ميدالية من تلك الفترة.

يقول: "كانت فكرتهم في عام 1791 هي أن المعيار سيعتمد على ظواهر طبيعية وغير متغيرة". ريتشارد ديفيس، المدير السابق لقسم الكتلة في المكتب الدولي للأوزان والمقاييس، القسم المكلف بحفظ جراند-ك. ووفقا له، "ما زلنا نفعل نفس الشيء". الفرق هو أن علماء القياس يريدون الآن أن يبنوا المعايير على ثوابت طبيعية لا تتغير حقًا.

نحن نجلس في مكتب Stock فيبافيلون دي بريتويلوهو مبنى أنيق من القرن السابع عشر يقف على تلة خضراء فيسانت كلاود بارك الناظر على نهر السين. كانت الحديقة بمثابة محمية صيد لملوك فرنسا، ولا تزال حديقة ورود ماري أنطوانيت تُزرع بعناية هنا. لقد أصبح مقرًا للمكتب الدولي منذ اتفاقية العدادات لعام 1875، وهي اتفاقية وقعتها 17 دولة.

"عندما عبرت الجسر باتجاه سفير، هل لاحظت الجزيرة على اليسار؟" يسأل ديفيس. ووفقا له، كان يوجد في الجزيرة مصنع لشركة رينو حيث تم تصنيع الدبابات للجيش الألماني في الحرب العالمية الثانية. وحاولت القاذفات الأمريكية ضربها عدة مرات. بعد أن هز أحد التفجيرات بافيلون دي بريتويل، تم وضع Grand-K في حاوية خاصة مضادة للصدمات. تم بالفعل نقل "الشهود" إلى قبو تحت الأرض في البنك الفرنسي وظلوا هناك معظم سنوات الحرب، ولكن وفقًا لاتفاقية العداد، كان على Grand-K البقاء في المكتب.

بعد الحرب، في عام 1946، عندما تم إخراج Grand-K من الخزنة لغرض التنظيف ومقارنتها بالنسخ الستة، تبين أنها أخف بمقدار 30 ميكروغرام من "الشهود". وعندما جاء موعد التنظيف التالي، بعد 45 عامًا، زاد الفرق إلى 50 ميكروجرامًا، أي وزن جناح الذبابة.

"خمسون ميكروغرامًا في مائة عام"، يقول ستوك وننظر إلى الرسم البياني الذي يوضح التغييرات على شاشة الكمبيوتر في مكتبه. "يمكنك أن ترى أن الفرق صغير." في الوقت الحالي، لا يخلق هذا الاختلاف صعوبات عملية، "لكن إذا واصلنا هذا، فسيؤدي ذلك في يوم من الأيام إلى مشاكل".

وفي عالم تكنولوجيا النانو، يمثل 50 ميكروجرام وزنًا كبيرًا. علاوة على ذلك، فإن عدم اليقين بشأن كتلة الكيلوجرام سوف ينتشر ويؤثر على قائمة طويلة من الوحدات الأساسية الأخرى. يتم تعريف الوحدة المترية للقوة، النيوتن، بالكيلوجرام. ويعرّف نيوتن بدوره الجول، وحدة الطاقة؛ في حين أن الجول يحدد الواط، وهكذا. في النهاية، قد تلتصق علامة استفهام صغيرة بكل قياس يتم إجراؤه في العالم المادي تقريبًا.

إن التنظيف والمقارنة بين Grand-K وكتل الاختبار ليست عمليات روتينية. لقد تم تصنيعها أربع مرات فقط منذ عام 1889. لإزالة أول Grand-K من تجويفه، أي من الغرفة الآمنة حيث يتم الاحتفاظ به، يجب أن يكون هناك ثلاثة أشخاص يجب عليهم فتح ثلاثة أقفال مرتبة أفقيًا. يوجد داخل هذه الغرفة المغلقة خزنة كبيرة أخرى مزودة بقفل مدمج، وداخل الخزنة الداخلية يوجد Grand-K تحت الأجراس الزجاجية الثلاثة الموضوعة واحدة فوق الأخرى. النسخ الست محفوظة أيضًا في هذه الخزنة. مدير المكتب الدولي للأوزان والمقاييس، ومدير الأرشيف الوطني بباريس، ورئيس المؤتمر الدولي للأوزان والمقاييس الذي يشرف على عمل المكتب، هم الأشخاص الثلاثة الوحيدون في العالم الذين لديهم مفاتيح غرفة القبو. وبما أن كل مفتاح مختلف، فيجب أن يكون الثلاثة موجودين لفتح الغرفة المحفوظة.

يقول باري إنجليس، وهو مهندس كهربائي أسترالي: "في تاريخ اتفاقية المقاييس لعام 1875، أنا ثاني شخص من خارج أوروبا يتم انتخابه رئيسًا للمؤتمر الدولي للأوزان والمقاييس". "سألت ماذا سيحدث إذا عدت إلى الوطن وتحطمت الطائرة فوق المحيط الهندي. كيف سيتدبرون أمرهم بدون المفتاح؟ لكنني متأكد من أن هناك صانع أقفال يمكنه فتح هذا القفل القديم دون بذل الكثير من الجهد."

تمكن عدد قليل من موظفي المكتب من رؤية Grand-K بأعينهم، وهناك شائعات بأن صوره الرسمية هي صور مكررة. تقول سوزان بيكارد، التي تعمل في المكتب منذ عام 1987: "لقد رأيته مرة واحدة". يقوم حاملو المفاتيح الثلاثة بفتح الخزنة مرة واحدة سنويًا لرؤية Grand-K، دون لمسها، فقط للتأكد من وجودها هناك.

بعد دخول حرم Grand-K عندما يحين وقت تنظيفها ووزنها، يلتقط أحد الفنيين الأسطوانة اللامعة بملقط مبطن بالجلد الناعم ويأخذها إلى محطة التنظيف حيث يتم صقلها بقطعة قماش جلدية مبللة بالكحول والأثير. ومن ثم شطفه بالماء المقطر المزدوج. يؤدي الانفجار الأخير لغاز النيتروجين إلى إزالة آخر القطرات المتبقية منه. تستغرق العملية برمتها حوالي ساعة. وقد جرب المكتب طرق تنظيف أخرى على كتل تجريبية، والتي تضمنت، من بين أمور أخرى، استخدام الأشعة فوق البنفسجية. ولكن اتضح أن هذه الأساليب تنظف السبائك أكثر من اللازم. يقول ستوك: "يبدو أنهم يزيلون أوساخًا أكثر من طريقتنا". "ولكن بعد هذا التنظيف، تصبح الكتلة غير مستقرة لأنه نتيجة للتنظيف الذي يتم الحصول عليه، فإن ميل السطح للتفاعل مع البيئة يزداد بشكل كبير." مثل هذا الوضع كان من شأنه أن يقلل من مصداقية Grand-K كمعيار، لذلك يستمر المكتب في استخدام طريقته القديمة، التنظيف بقطعة قماش جلدية ناعمة والغسيل بالماء.

بعد الغسيل، يتم نقل Grand-K و"الشهود" إلى غرفة نظيفة ويتم وضعهم في جهاز يسمى "يقارن المقالات". إنه جهاز بقيمة 500,000 دولار يمكنه قياس فروق الكتلة التي لا تزيد عن ميكروجرام واحد. إن مُقارنة الكتلة و"أجسام العمل" العشرة ذات الوزن القياسي بالكيلوجرام هي "قوة العمل" في قسم الكتلة التابع للمكتب الدولي للأوزان والمقاييس. ويتم استخدامها في أعمال المعايرة اليومية، بينما يخرج جهاز Grand-K و"الشهود" إلى العالم مرة واحدة فقط كل بضعة عقود لغرض التحقق من وزن الكيلوغرامات الرسمية لمختلف البلدان.

بعد أن استمرت محادثتي مع ديفيس وستوك لبعض الوقت، سألتهم إذا كان بإمكاني رؤية غرفة القبو من الخارج حيث يقيم Grand-K. أعلم أنه لا توجد فرصة لرؤية الجليل النبيل بحد ذاته. فانفجروا بالضحك وهزوا رؤوسهم: «لا، لا، لا، لا!»

يقول ديفيس: "هذه ليست المرة الأولى التي يُطلب منا فيها رؤية الخزنة".

"إنها هنا، في هذا المجمع، أليس كذلك؟"

يجيب ديفيس: "نعم، هذه التفاصيل ليست سرا".

قياس صعب

تقارن موازين كيبل، مثل جهاز NIST-4 التابع للمعهد الوطني الأمريكي للمعايير والتكنولوجيا، بين القوة الكهربائية والقوة الميكانيكية، وهو جهاز دقيق وصعب الاستخدام، ويلعب دورًا مركزيًا في عملية إعادة تعريف الكيلوجرام. المصدر: كيرت سبلي/NIST.
موازين السعة، مثل جهاز NIST-4 التابع للمعهد الوطني الأمريكي للمعايير والتكنولوجيا، تقارن بين الطاقة الكهربائية والقدرة الميكانيكية. وهو جهاز دقيق وصعب الاستخدام، ويلعب دورًا مركزيًا في عملية إعادة تعريف الكيلوجرام. مصدر: كيرت سوبلي/NIST.

قريبًا، عندما يعتمد التعريف الشامل الدولي الجديد على ثابت بلانك، سيصبح Grand-K عنصرًا ستكون قيمته تاريخية بشكل أساسي. يشمل تعريف ثابت بلانك كلاً من وحدات الطاقة ووحدات الزمن، ويمكن التعبير عنه بدلالة الكتلة بناءً على المعادلة E=mc2. مثل ثابت الجاذبية، G، أيضًا ثابت بلانك، h، هو كيان مشتق من النظرية، لكن قيمته العددية لا يمكن تحديدها إلا تجريبيًا، وبفضل الأدوات الأفضل، أصبحت درجة الدقة التي يمكن بها قياس الثوابت الطبيعية أفضل وأفضل .

للانتقال إلى المعيار الكمي الجديد، قام المكتب الدولي للأوزان والمقاييس بتطوير استراتيجية من جزأين. أولاً، تحدد مختبرات القياس الوطنية في خمس دول مختلفة القيمة العددية لثابت بلانك. بعد ذلك يتم وزن الكيلوجرامات الرسمية الوطنية الخاصة بهم حسب القيمة التي سيتم الحصول عليها وأخيرا مقارنة درجة التوافق بين القياسات المختلفة للكيلوجرامات. وقد أجرى الديوان هذه الاختبارات في صيف عام 2016. ومن المتوقع أن تصل النتائج مطلع عام 2017، وإذا كانت مرضية فإن المختبرات المشاركة في الدراسة ستبدأ عملية عكسية، تستخدم فيها الكيلوغرامات الرسمية التي عقدتها مختلف البلدان لزيادة تحسين دقة قياس ثابت بلانك. وفي نهاية العملية، سيتم استخدام القيمة الجديدة والأكثر دقة لثابت بلانك لصياغة التعريف النهائي للكيلوغرام.

ويتم معظم هذا العمل بمساعدة جهاز معقد للغاية يسمى "موازن كيبال". حتى عام 2016، كان هذا الجهاز يسمى "موازين الوات"، لكن علماء القياس قرروا تغيير اسمه بعد وفاة الفيزيائي البريطاني الذي اخترعه عام 2016. بريان كيبل. من الصعب جدًا إجراء تجارب على موازين كيبل، حتى أن مجلة Nature أدرجتها ضمن هذه الموازين في عام 2012 أصعب خمس مهام في الفيزياءإلى جانب اكتشاف بوزون هيغز أو موجات الجاذبية.

في مايو 2016 قادني ستيفن شلامينجر من المعهد الوطني للمعايير والتكنولوجيا (NIST) إلى مبنى أبيض مكون من طابقين على حافة حرم جامعي مشجر يغطي حوالي 24 دونما. تم تركيب موازين قابلة للطحن في هذا المبنى، وهو أقدم المنشأتين المملوكتين للمعهد. منذ اكتمال النموذج الأحدث في عام 2014، لم يعد المرفق الأقدم قيد الاستخدام. "هذا المكان يشبه منزلًا صغيرًا في المرج"، يقول شلامينغر مازحًا عندما توقفنا أمام المبنى المعزول. هذا هو المكان الذي تم فيه إجراء معظم قياسات المعهد لتحديد قيمة ثابت بلانك، وسيعمل النموذج الجديد بنفس الطريقة تقريبًا.

أي تشابه مع المنزل الصغير في المرج يختفي عندما ندخل المبنى. من الداخل يبدو وكأنه مكان لرواية من عصر الثورة الصناعية مليئة بالمحركات البخارية. الجدران مغطاة بصفائح نحاسية حتى سقف الطابق الثاني. "هل ترى أن جميع الأجهزة مصنوعة من النحاس؟" يسأل شلامينغر. "لا يوجد حديد هنا." يحمي النحاس والنحاس الجهاز من المجالات المغناطيسية الخارجية. لكن المجالات المغناطيسية التي تنشأ داخل المبنى قوية بما يكفي لمسح بطاقات الائتمان. في منتصف غرفة في الطابق الأول يوجد عمود دعم طويل مع مغناطيس فائق التوصيل في قاعدته. عند تشغيله، يتم تبريده بالهيليوم السائل.

تقع منشأة القياس نفسها في الطابق الثاني. وتتكون من عجلة من الألومنيوم تقف بشكل عمودي على الأرض مع موازين معلقة على كلا الجانبين. أثناء القياس، يتم وضع كتلة كيلوغرام على أحد المقاييس. يتم تعليق ملف من الأسلاك مباشرة أسفل هذه الملعقة على عمود يبلغ طوله من ثلاثة إلى أربعة أمتار. يوجد في الملعقة الموجودة على الجانب الآخر من الميزان ثقل موازن ومحرك كهربائي. من أجل العثور على جميع القيم اللازمة للمعادلة التي تربط الكتلة بثابت بلانك، يجب تشغيل المقاييس بطريقتين مختلفتين. في "وضع الوزن"، تتم موازنة قوة الجاذبية للأسفل التي تمارسها كتلة الاختبار تمامًا بواسطة مجال مغناطيسي تم إنشاؤه بواسطة تيار كهربائي ينتقل في ملف أسفل الملعقة. في "وضع السرعة" تتم إزالة كتلة الاختبار من المقبض ويقوم المحرك الموجود في المقبض الآخر برفع الملف بسرعة ثابتة من خلال المجال المغناطيسي الناتج عن مغناطيس الجهاز فائق التوصيل، والذي يولد جهدًا كهربائيًا في الملف المتحرك.

من الصعب جدًا إجراء تجارب على موازين كيبل، لدرجة أن مجلة Nature أدرجتها في عام 2012 ضمن أصعب خمس مهام في الفيزياء، إلى جانب اكتشاف بوزون هيغز وموجات الجاذبية.

يتم وضع التيار المقاس في وضع الوزن والجهد الحاصل في وضع السرعة في معادلات نظرية الكم التي تربط التيار الكهربائي والجهد والمقاومة بثابت بلانك. باختصار، في مقاييس كيبل، تكون نقطة البداية هي كيلوغرام واحد، ومن خلاله يمكن للجهاز تحديد ثابت بلانك. وبعد ذلك، عندما تكون لدينا قيمة دقيقة لثابت بلانك، يمكننا استخدام الموازين لقياس الكتلة دون الحاجة إلى أي أوزان فيزيائية للمقارنة.

وللتوصل إلى نتائج دقيقة، كان على شلامينجر وزملائه أن يأخذوا في الاعتبار التقلبات المحلية في ضغط الهواء والجاذبية، بالإضافة إلى تهز الأرض والمد والجزر المحيط. "إذا لم تقم بتصحيح النتائج وفقا للمد والجزر، فسيتم إنشاء انحراف بحوالي 100 جزء في المليار"، يشرح شلمينغر. على الرغم من تعقيدها، فإن هذه المنشأة تذكره بأشياء من عصر آخر. عندما كان فريقه مشغولاً بقياس ثابت بلانك، كان عليهم فتح وإغلاق الصمامات بترتيب صارم ومراقبة الضغط باستمرار داخل الخزانات المملوءة بالهيليوم السائل. يقول شلامينغر: "لقد بدا الأمر وكأنني أقود قاطرة من أيام القطارات البخارية، لكن في الحقيقة، قامت هذه التجارب بقياس الأبعاد في ميكانيكا الكم!"

وداعا لGrand-K

واستمرار الأمور يعتمد على نتائج تجارب عام 2016. يجب أن تكون قياسات الكيلوجرام لثلاثة من مختبرات القياس الوطنية الخمسة المشاركة متطابقة تقريبًا؛ ويجب ألا يزيد الفرق بينهما عن 50 ميكروجرام. هذا هو عدم اليقين الحالي فيما يتعلق بكتلة Grand-K. وبعد نشر نتائج الدراسة الأولية، سيبدأ العمل الجاد على التعريف الجديد.

إذا سارت الأمور على ما يرام، فسيتم تحديد الكيلوجرام في نهاية العملية بدلالة ثابت بلانك. وضع المكتب الدولي للأوزان والمقاييس معايير صارمة: ليس فقط أن اختلافات القياس يجب ألا تتجاوز 50 جزءًا في المليار، ولكن في أحد هذه المعايير يجب أن يكون عدم اليقين أقل من 20 جزءًا في المليار. لقد حقق الكنديون هذا المستوى بالفعل. لكي يدخل التعريف الجديد حيز التنفيذ في عام 2018، يجب قبول القياسات الجديدة لثابت بلانك للنشر بحلول 1 يوليو 2017.

وماذا عن جراند-ك؟ وسوف يبقى في خزانته. نظرًا لأن الموازين المطحونة هي أجهزة معقدة للغاية، فيمكن الافتراض أننا سنستمر في رؤية الكيلوجرامات القياسية الفيزيائية. وبدلاً من الاستمرار في إجراء قياسات شاقة على موازين الميزان، ستستمر مختبرات القياس على مدى العقود القادمة في استخدام جيل جديد من النماذج القياسية في عملها اليومي. يخطط المكتب بالفعل لهذه النماذج، ولكن سيتم إجراء معايرتها الدقيقة باستخدام مقاييس Kibble، وليس وفقًا لـ Grand-K.

هل هذه نهاية القصه؟ هل أصبح لدينا أخيرا كيلوغرام "لجميع الأوقات، لجميع الأمم؟" المخزون لا يزال غير جاهز للالتزام.

ويقول: "اعتقد أحد أسلافي في هذا المنصب، الحائز على جائزة نوبل، تشارلز إدوارد غيوم، أن الكيلوغرام الحالي سيخدمنا لمدة 10,000 آلاف عام". "لقد كانت رؤية مفرطة في التفاؤل بالطبع. لست مقتنعًا بأن هذا سيكون الإعداد النهائي، لكنه سيكون جيدًا لبعض الوقت. ربما ليس خلال العشرة آلاف سنة القادمة."

جيد ان تعلم

تغيير الأحجام

في أساس النظام الدولي للوحدات (SI)، المعروف باسم "النظام المتري"، هناك سبع وحدات أساسية. وتستند 22 وحدة أخرى على هذه الوحدات السبع. وفي عام 2018، من المتوقع أن يعيد المؤتمر الدولي للأوزان والمقاييس تعريف معظم الوحدات الأساسية. ستكون هذه أكبر مراجعة للنظام المتري منذ الستينيات. وكجزء من هذه الخطوة، سيتم ربط الوحدات الأساسية السبع بثوابت طبيعية لا تتغير. لن تتغير وحدات المتر والثانية والكانديلا، لكن الوحدات الأربع الأخرى ستحصل على تعريفات جديدة ومختلفة تمامًا عن التعريفات الحالية.

تم تعريف الوحدات بالفعل من حيث الثوابت الفيزيائية

الوحدة: متر          
علامة: m                                    
الحجم المقاس: طول

التعريف الحالي (تأسس عام 1983): المسافة التي يقطعها الضوء في الفراغ في 299,792,458 جزء من الثانية.

ملاحظة تاريخية: عندما اقترحت الأكاديمية الفرنسية للعلوم النظام المتري عام 1791، حددت المتر بأنه جزء من عشرة ملايين من ربع محيط الأرض، والذي تم تعريفه بدوره على أنه خط الطول الذي يمر بين القطب الشمالي وخط الاستواء مروراً بباريس. (بالطبع).

 

الوحدة: ثانيه      
علامة: s
الحجم المقاس: الزمن

التعريف الحالي (تأسس عام 1967): الثانية هي مدة 9,192,631,770 دورة إشعاعية تقابل الانتقال بين مستويين فرعيين من الطاقة في ذرة السيزيوم 133.

ملاحظة تاريخية: التعريف الأصلي هو التعريف المألوف: الثانية هي الجزء 86,400 من "متوسط ​​اليوم الشمسي"، أو الوقت الذي تستغرقه الأرض لإكمال دورة واحدة حول نفسها بالنسبة إلى الشمس. والثاني حصل على تعريفه الحالي، استنادا إلى ميكانيكا الكم، في عام 1967.

 

وحدة: كانديلا      
علامة: cd
الحجم المقاس: تنوير

التعريف الحالي (تأسس عام 1979): الشمعة هي "الإضاءة، في اتجاه معين، لمصدر يصدر إشعاعًا أحادي اللون بتردد 1012× 540 هرتز، وقدرته في هذا الاتجاه 1/683 واط لكل ستراديان" (الاستراديان هو وحدة في النظام المتري للزاوية في الفضاء ثلاثي الأبعاد).

ملاحظة تاريخية: في بداية القرن العشرين، قامت الولايات المتحدة الأمريكية وفرنسا وبريطانيا العظمى بتعريف الكانديلا بالنسبة لإضاءة الشمعة القياسية. وفي عام 20، اختار علماء القياس تعريفًا أكثر دقة يعتمد على إشعاع الجسم الأسود. تم اعتماد هذا التعريف في عام 1933، وتم استبداله لاحقًا بالتعريف الحالي.

 

الوحدات التي لم يتم إعادة تعريفها بعد

وحدة: كيلوغرام       
علامة: kg
الحجم المقاس: الجماعية

التعريف الحالي (تأسس عام 1889): في الوقت الحالي، لا يزال يتم تعريف الكيلوجرام بالنسبة إلى Grand-K، وهي أسطوانة من سبائك البلاتين والإيريديوم محفوظة في خزانة بالقرب من باريس.

التعريف الجديد المقترح: إذا تقدمت الجهود كما هو مخطط لها، فسيتم ربط الكيلوجرام في عام 2018 بثابت بلانك، وهي كمية ثابتة في نظرية الكم تعبر عن كمية الطاقة التي تحملها وحدة واحدة من الضوء، أو الفوتون.

 

وحدة: أمبير        
علامة: B
الحجم المقاس: حاضِر

التعريف الحالي (تأسس عام 1946): التعريف الحالي للأمبير يتضمن، من بين أمور أخرى، "موصلين مستقيمين متوازيين بطول لا نهائي، مع مقطع عرضي دائري مهمل... موضوعين على مسافة متر واحد في الفراغ". هذه هي الشروط التي لا يمكن إنشاؤها بدقة في المختبر.

التعريف الجديد المقترح: سيتم تبسيط الأمبير من خلال ربط قيمته العددية بالشحنة التي تحمل بروتونًا واحدًا (الثابت الأساسي المعروف بالشحنة الأساسية).

 

وحدة: كالفين       
علامة: K
الحجم المقاس: درجة حرارة

التعريف الحالي (تأسس عام 1967): اليوم، يساوي كلفن واحد "الجزء 273.16 من درجة الحرارة الديناميكية الحرارية للنقطة الثلاثية للمياه" - وهو مزيج من درجة الحرارة والضغط الذي يمكن أن يتواجد عنده الجليد والماء وبخار الماء في وقت واحد.

التعريف الجديد المقترح: التعريف الذي من شأنه أن يبني كلفن على قيمة ثابتة لثابت بولتزمان، الذي يربط متوسط ​​الطاقة الحركية لجزيئات الغاز بدرجة حرارتها المطلقة، من شأنه أن يحسن دقة قياسات درجات الحرارة المرتفعة أو المنخفضة للغاية.

 

وحدة: م        
علامة: مول
الحجم المقاس: كمية من المواد

التعريف الحالي (اعتمد في عام 1971): "المول هو كمية المادة الموجودة في نظام يحتوي على جسيمات أولية بعدد يساوي 0.012 كيلو جرام من ذرات الكربون 12."

التعريف الجديد المقترح: سيتم قطع العلاقة بين المول والكيلوجرام، وسيتم تحديد الوحدة من خلال ربطها بثابت أفوجادرو، والذي يشير إلى عدد الجزيئات أو الذرات أو أي جسيم منفصل آخر من المادة الموجودة في المول الواحد.

عن الكتاب

تيم فولجر - الكتابة لمجلات مثل National Geographic وDiscover. كما يقوم أيضًا بتحرير أفضل كتابة أمريكية للعلوم والطبيعة، والتي تنشرها سنويًا دار هوتون ميفلين هاركورت.

لمزيد من القراءة

تعليقات 284

  1. يهودا،
    على عكس ما هو مكتوب هنا، أنا لا أفهم على الإطلاق كيف تنتج آلية مرنة الجاذبية الخفيفة. لماذا تؤدي التصادمات المرنة إلى ثني شعاع الضوء ولكنها لا تنتج انكسارًا مشابهًا لقوس قزح أو ألوان الغلاف الجوي؟
    بالإضافة إلى ذلك، سأكون سعيدًا جدًا بمعرفة مقدار انحناء الضوء، وفقًا لصيغتك: كما توقع نيوتن أو النسبية أم أن الوقت قد حان لشيء مختلف تمامًا؟
    هذا هو detind cili

  2. يودا

    ما هي التجربة ل؟ هل يختلف أحد معك في أن الدفع يخلق الجاذبية؟

    والسؤال هو إذا كان هذا ما يحدث في الواقع، فهل جزيئات المريمية هي التي تربطنا بالأرض.

    لماذا لا نحاول استبدال الجسيمات بالإشعاع؟ سوف تحصل على نفس الجاذبية التي تحصل عليها عند الدفع، ولكن سيتم حل معظم مشاكل الدفع الكلاسيكية.

  3. ألبانزو
    لقد فوجئت بتعليقاتك في تعليقك الأخير.
    أليست النظرية النسبية "مفتاح الجاذبية"؟
    هل النسبية العامة هي نظرية الجاذبية التي عفا عليها الزمن؟
    أفكر بالفعل أنني ربما أحاول أن أفهمها فحسب. ربما يكون الأمر يستحق بناء الدفع بغض النظر عن النسبية العامة وسنرى أين سننتهي، أعتقد أنني سأعمل في نفس الوقت على التخطيط لتجربتي

  4. وهناك أيضا ينتظر يهودا. أعتقد أنها علقت لأنني استخدمت كلمة ممنوع (شكل من أشكال الحكم كان يُمارس في إيطاليا قبل الحرب العالمية الثانية. لا داعي للذعر، فالكلمة لم تكن موجهة إلى أي شخص). نأمل أن يتم إطلاق سراحك قريبا.

  5. ومن الواضح أن النظرية النسبية ليست "مفتاح الجاذبية". منذ يوم نشره تقريبًا، أدرك الناس أنه لا يمكنه وصف العمليات على مسافات قصيرة جدًا (طول بلانك) أو عند طاقات عالية جدًا. النسبية العامة هي نظرية كلاسيكية، وكما أن ميكانيكا نيوتن هي نظرية كلاسيكية لا تصف عالمنا بشكل صحيح ولكنها تقريب ممتاز في ظل ظروف معينة، فإن النسبية العامة هي تقريب ممتاز في مقاييس الطاقة المنخفضة. وعندما أقول منخفضة، أعني "لا تزال أعلى بكثير من الطاقات التي نصل إليها حتى مع المسرعات المتقدمة". لذلك، على الرغم من أن النسبية العامة من الناحية النظرية هي نظرية عفا عليها الزمن للجاذبية، إلا أنها الأكثر تقدمًا والتي تم اختبارها تجريبيًا - ببساطة لأننا لا نملك القدرة على اختبار ظواهر الجاذبية الكمومية بالتكنولوجيا الحالية.

    بالمناسبة، في السنوات الأخيرة، هناك فهم أنه يوجد لدى الرجال السود اختلافات بين الجاذبية الكلاسيكية والكمية، وهي ليست كمية فقط (وبالتالي أقل من دقة القياس) ولكنها أيضًا نوعية وبالتالي يمكن أن تؤدي حقًا إلى سلوكيات مختلفة. ولهذا السبب كانت الثقوب السوداء الكمومية موضوعًا بحثيًا ساخنًا للغاية خلال السنوات الخمس الماضية.

  6. يهودا
    ما أقوله ينطبق أيضًا على الغاز الرقيق جدًا. وستكون هناك ذرات ترتد عن الأرض مثل الحجارة على الماء، فيبطل أثر تلك التي كانت محجوبة.
    وطبعا تتجاهل الباقي..

  7. يهودا،
    عندما يمر الضوء عبر الغلاف الجوي، فإنه يتحول إلى اللون الأزرق بسبب تشتت الضوء في الغلاف الجوي
    لماذا لا تلون جزيئاتك السماء؟ تمت محاكاة جزيئاتك كغاز وهي جيدة بما يكفي لثني الشعاع ولكنها ليست كافية للقيام بالتشتت؟

  8. وفيما يلي أسئلتك:
    سؤال: وماذا يفعل الغاز بالنور؟ الجواب يثنيه حسب كثافته.
    سؤال: أود أن أحصل على إجابة لسؤالي الذي ليس مجرد طرح: الإجابة: من فضلك.
    سؤال: إذا كان كل ما تفعله جزيئاتك هو تأثير مرن، فلماذا ينحني الضوء بها؟
    الجواب: لا يحدث الانحناء بسبب مرونة الجزيئات بل بسبب كثافتها.
    سؤال: وإذا كان الضوء ينحني مع ذلك، فهل ينحني كما تنبأ نيوتن أو كما تنبأت النسبية، وإذا كان كما تنحني النسبية، فكيف تحدث هذه المعجزة؟ الجواب: هذا هو سؤال المليون دولار. وهذا ما أتعلمه الآن.
    يوم جيد وكل عام وأنتم بخير
    يهودا

  9. يهودا،
    وماذا يفعل الغاز بالضوء؟
    أود أن أحصل على إجابة لسؤالي الذي لم يُطرح عبثًا: إذا كان كل ما تفعله جزيئاتك هو التأثير المرن، فلماذا ينحني الضوء بها؟ وإذا كان الضوء ينحني مع ذلك، فهل ينحني كما تنبأ نيوتن أو كما تنبأت النسبية، وإذا كان كما تنحني النسبية، فكيف تحدث هذه المعجزة؟

  10. لقد سبق أن أوضحت أنه في الغاز العادي في الظروف العادية لن تظهر ظاهرة الجاذبية لأن متوسط ​​المسار الحر لجزيئات الغاز قصير جداً بالنسبة للمسافة بين الأجسام. وبالتالي، فإن الجسيمات الخارجية سوف تصطدم بالجسيمات المولدة للجاذبية، وسوف تفقد اتجاهها. ولزيادة المسار الحر نحتاج إلى غاز رقيق عند ضغط منخفض وتكون المسافة بين جزيئاته أكبر.

  11. يهودا،

    من المضحك جدًا أنك تشتكي من اضطرارك إلى تكرار نفس المطالبة عدة مرات. إذا كان لدي سنتًا مقابل كل مرة اضطررت فيها إلى تكرار نفس الشيء مرارًا وتكرارًا لأنك تتجاهلني، فسأصبح مليونيرًا. على سبيل المثال - كم مرة أوضحت لك بالفعل أن الصيغة التي تقترحها تتعارض بشكل واضح مع النسبية العامة؟ إن حقيقة قولك عبارة "لا يوجد تناقض بين صيغتي والنسبية العامة على مسافات قصيرة" لا تساوي شيئًا. وكأنني أقول "لا يوجد تناقض بين الفيزياء الكلاسيكية وفيزياء الكم" أو "لا يوجد تناقض بين الفاشية والديمقراطية". يمكنك أن تقول ذلك بقدر ما تريد - فهو لا يزال هراء.

    الصيغة التي تقترحها تتناقض مع النظرية النسبية بشكل واضح لا يشوبه غموض حتى على المسافات القصيرة. وحقيقة أنك لا تفهم النظرية النسبية ولا تعرف عنها شيئًا لا تمنحك الإذن باختراع أوهام تتصرف فيها كما يحلو لك.

  12. يهودا
    يقولون أن كل مشكلة لها حل بسيط وأنيق...وخاطئ.
    إذا كانت الاصطدامات مرنة، فسوف تصطدم بي الكثير من الجزيئات بعد اصطدامها بالأرض. فكر في الجسيمات القادمة من الأفق بزاوية مسطحة.
    وفقًا لما وصفته، كان من المفترض أيضًا أن يدفعني الهواء فوقي إلى الأرض، لكن هذا لا يحدث. وما أقوله ينطبق على كل من الغاز المضغوط والغاز الرقيق جدًا.

    أدرك لاسيج ذلك، ولذلك جادل بأن التصادمات غير مرنة تمامًا. والمشكلة التي تنشأ بعد ذلك هي الانحباس الحراري الهائل.

    هناك مشكلة أخرى وهي أن النموذج الخاص بك يتطلب شفافية عالية للمادة، وبالتالي فإن شكل الجسم ليس له أي تأثير. لكن - كيف تفسر أن وزن نواة الذرة هو حاصل ضرب بسيط لعدد الجزيئات الموجودة فيها؟

    كيف لا ترى كل التناقضات بين نموذجك والنظرية النسبية؟ وفقًا لنموذجك، ليس للفضاء أي خصائص، ولكن وفقًا للنظرية النسبية، فإن هذه الخصائص هي مصدر الجاذبية والثبات. بحسب توراتك، لا يوجد سبب لتغير معدل الزمن، أو لانحناء الضوء، أو لوجود السرعة ج.
    تشرح النظرية النسبية أشياء أخرى، مثل لماذا تنتج الشحنة المتحركة مجالًا مغناطيسيًا. وتشرح كيف أن هناك أجسامًا أسرع من سرعة الضوء، ونحن نراها أيضًا!

    ومن دون افتراض صحة النظرية النسبية في كل أنحاء الكون، فلا فائدة من الحديث عن الانفجار الأعظم، أو حتى عن علم الكونيات. ومن الأفضل أن نفترض أن النجوم هي نقاط في السماء.

  13. وبسبب ذكر فاينمان، وبخ زوجته في عام 1962 قائلاً: "ذكريني ألا أشارك في المحاضرات والاجتماعات حول الجاذبية بعد الآن، فأنا أيضًا أستطيع أن أكتب صيغًا لا يفهمها أحد".

    لكن إذا كانت الأوتار هي مفتاح الجاذبية، ألا يعني ذلك أن النسبية ليست كذلك؟ أو ربما رأى أينشتاين المستقبل في رؤيته؟

  14. المعجزات
    من المؤسف أنه بعد أشهر من المناقشة مازلت لا تفهم مبدأ دفع الجاذبية البسيط. فالشخص الذي يقف على الأرض يستقبل من جهة الأرض جزيئات أقل من تلك التي يستقبلها من جهة رأسه، ولذلك يُدفع إلى الأرض. حقا بسيطة.
    يوم جيد وكل عام وأنتم بخير
    يهودا

  15. يهودا،
    لا أحد يمنعك من الدراسة. في الحقيقة كان كل شيء هادئًا حتى أجبت على إجابات لم تجب على شيء. السمكة على الطبق لأنه فتح فمه.
    لا يمكنك مقاومة النسبية لأنها غير موجودة بالنسبة لك. لا يوجد فيلم توجد فيه النسبية ولا صيغتك كذلك، وبالتالي ليس لديك أي أساس نظري لاستخلاص الأفكار من النسبية. ليس لديك سبب لتحرك الجاذبية بسرعة الضوء إلا كشرط اعتباطي قررته لأنك تدرك أن الأمر لا يستحق الجدال مع النسبية لأن النسبية تنبأت بنجاح بتنبؤات تحققت منذ 100 عام.
    ما هو توقعك بشأن انحناء الضوء بالقرب من الكتل؟ مثل نيوتن أو مثل النسبية وإذا كانت مثل النسبية فكيف تحدث هذه المعجزة؟

  16. شموليك نسيم وآخرون
    أنت تتدخل في تعلمي للمادة وتصر على مجرد طرح الأسئلة. على سبيل المثال، تم طرح سؤال، هل تتحرك الجاذبية بشكل لحظي مثل نيوتن أم بسرعة الضوء كما في النسبية؟ أجبت أنه من الواضح أن الجاذبية ستعمل بسرعة الجسيمات التي تقارب سرعة الضوء على ما يبدو. وهذا لم يرضي نسيم، فعاد ليطرح سؤالاً عرضياً حول الموضوع، فلا داعي للثرثرة، ومن المؤسف أن يظن أحد أنني مخطئ في مسألة واضحة كهذه وهي سرعة ظاهرة الجاذبية. إذن فهي تتحرك هنا بسرعة الضوء كما في النظرية النسبية، فإذا اختفت الشمس فجأة، سيظل عدد أقل من الجزيئات المندفعة يصل إلى الأرض مقارنة بالجسيمات القادمة من الجانب المقابل، مما يعني أن الجاذبية ستستمر في العمل لمدة ثماني دقائق تقريبًا. إذن، شيء واضح يجب شرحه مرارًا وتكرارًا؟ بالإضافة إلى ذلك، كم مرة يجب أن أقول إن فكرة دفع الجاذبية ليس لها اعتراض على النسبية على مسافات صغيرة. أما بالنسبة للمسافات الكبيرة فلا توجد جاذبية على الإطلاق وفقاً للكون البسيط. تحلى بالصبر قليلاً وسنرى ما أفهمه عن الإسناد العام.
    لذلك لن أجيب الآن حتى أحصل على شرح أكثر جدية لموضوع الإسناد العام
    ليلة سعيدة وسنة جديدة سعيدة
    يهودا

  17. يهودا،
    أنا لا أطلب شيئا والمعجزات لا تجري معك حوارا أصم. لا يتعلق الأمر بالتعرض للإهانة فجأة أو التشكيك في دوافعي.

    لماذا هذه السرعة؟ كيف هي العلاقة بين سرعة الضوء والجاذبية؟ في أي إطار يعتبر الضوء أسرع سرعة لنقل المعلومات؟ لقد تم شرح لك أنه إذا قمت بتغيير نيوتن، فقد فقدت الاتصال النظري بالنسبية (سواء من قبل ألبانزو أو ستيفن واينبرغ) وبالتالي فإن أي خاصية تطلبها من النسبية يجب أن تأتي من شيء آخر لأنه من الواضح أنها لا تأتي من صيغتك . من ماذا؟
    على أية حال، أود الحصول على إجابة لسؤال الدلو، الذي لم يُطرح عبثًا: إذا كان كل ما تفعله جزيئاتك هو تأثير مرن، فلماذا ينحني الضوء بها؟ وإذا كان الضوء ينحني مع ذلك، فهل ينحني كما تنبأ نيوتن أو كما تنبأت النسبية، وإذا كان مثل النسبية، فكيف؟

  18. يهودا
    هذا خطأ يجعلني أرغب في البكاء….. إذا كانت جزيئاتك تتحرك بسرعة معينة، فيجب أن تعتمد الجاذبية أيضًا على السرعة.

    تحتاج أيضًا إلى شرح سبب تساوي كتلة الثبات لديك مع كتلة الجاذبية. أنت تنسب آليات مختلفة لكليهما. أنت في الأساس تتجاهل المشكلة المتمثلة في عدم وجود أي تفسير على الإطلاق لسبب وجود كتلة الثبات.

    أنت تتجاهل الجاذبية.

    أنت تتجاهل التغيير في وتيرة الزمن.

    تتجنب مشكلة الاحتكاك (لماذا لا يوجد الكثير من الضوء؟).

    يتم حل كل هذه الأمور في نظام معادلة واحد للنسبية العامة (في الواقع معادلة واحدة، وهي ليست معقدة للغاية بحيث يصعب فهمها).

  19. إلى شموليك
    تتحرك تغيرات الجاذبية بسرعة الضوء تقريبًا لأن هذه هي سرعة الجسيمات التي تحدد الجاذبية. بسيطة جدا تجعلني أبكي.
    وعندما تقول أنه لا يوجد أي اتصال بيني وبين النسبية، فأنت مخطئ، فالارتباط على الأقل مثل اتصال نيوتن وليس لدي أي اعتراض على النسبية على مسافات قصيرة. أشعر وكأنني أكرر نفسي. دعونا نفهم الإسناد العام وسوف نمضي قدما. لدي شعور أنك تسأل فقط. وبالمناسبة، لقد نقرت على الرابط وحصلت على تعليق عشوائي، وسأتحقق لاحقًا من سبب ذلك.
    سنة جيدة
    يهودا

  20. يهودا،
    يعني أنت لم تجيب على أي شيء.

    القوة "2" لا تحظى بشعبية عندي... إنها نيوتن وعند أينشتاين تظهر نتيجة تقريب معادلات النظرية النسبية في ظروف معينة. لو كنت قد رأيت مقطع اليوتيوب الذي أرسلته، لكنت سمعت ستيفن واينبرغ يقول ذلك. وقال أيضًا إنه لا توجد طريقة للعب بمعادلات النسبية دون تدميرها والفكرة التي تكمن وراءها. ربما يكون الفيزيائيون مخطئين، لكن يبدو لي أنك بحاجة إلى دراسة النسبية جيدًا لتتمكن من قول ذلك. لا تظن

    مرة أخرى سأكرر ما كتبته بالفعل، ويبدو أنك توافق على ذلك: أنت لا تناقض النسبية فحسب، بل تدمر نيوتن أيضًا لأنه على الرغم من أنه لم يقترح آلية للجاذبية، إلا أنك تقترح آلية ميكانيكية للجاذبية وبالتالي يجب عليك توضيح السبب. التغيرات في الجاذبية تتحرك فقط بسرعة الضوء، ما هو المميز في سرعة الضوء معك (تذكر أنه لا يوجد علاقة بينك وبين النظرية النسبية) ولماذا تؤثر الجاذبية على الضوء بالطريقة التي تعمل بها، وعلى وجه الخصوص لماذا ينحني الضوء بالطريقة التي ينحني بها في وجود جاذبيتك. لماذا لا ينتشر الضوء في كل مكان؟
    وفيما يتعلق بالشفق القطبي، مرة أخرى، فإن القمر ليس له غلاف جوي، وبالتالي، إذا نظرت إلى الشمس من القمر، فسوف تراها كدائرة متوهجة في الفضاء المظلم. نظرًا لأن جزيئاتك تتفاعل كثيرًا مع الشمس - وتخلق الجاذبية - وتحني الضوء أيضًا (فقط قرر مثل من) لا يمكنك أن تخبرني أنها لا تلحق الضرر بالضوء. إذا ضربت الضوء، فكيف تحرف الضوء فقط ولا تنتج نفس تأثير الغبار وشعاع الليزر، أي التوهج؟

    نظرًا لأنه يبدو لي أنك ستضطر إلى اللعب بصيغتك، فلاحظ ما كتبه لك ألبانتيزو حول هذا الموضوع: "... يتم ذلك فقط في ظل ظروف معينة. على سبيل المثال، بالنسبة للضوء حتى في المجال الضعيف، هناك سلوك مختلف جذريًا بين النسبية ونيوتن." لذا، عندما تقوم بهندسة صيغتك، كيف تفضل هندستها، مثل نيوتن أو مثل أينشتاين؟

  21. للمعجزات
    حاولت الدخول إلى الكارتون وفشلت. آمل أنها ليست مهينة للغاية. تعتمد صيغتي على فرضية أنه إذا كانت صحيحة، فإنها تتمتع بميزة على صيغة نيوتن التي تم الحصول عليها بالاستقراء. وحقيقة أنها مستمدة من قوانين كيبلر، والتي تم اشتقاقها أيضًا من بيانات تايكو براهي، لا تجعلها صحيحة بالنسبة للكون بأكمله، ولكن فقط بالنسبة للقياسات التي تستند إليها. وفي مسافات كبيرة أو صغيرة أخرى قد تكون الصيغة مزيفة.
    لماذا لا يسمح لي "باللعب" بصيغتي؟؟، مسموح لي! على سبيل المثال، يُسمح لي أن أرى أنها تحل مشكلة مبادرة كوكب هيما دون النسبية. وبدا ذلك شيئًا لطيفًا بالنسبة لي. لا أحاول حل كل شيء معها، أنا فقط أحب العلم وأحاول أن أفهم أكثر عن المواضيع التي أقرأها.
    وبالطبع يمكن أن أكون مخطئا. سأواصل مساعي، هناك الكثير من المواد ولست متأكدًا من أنني سأنجح. لذا يرجى عدم الإزعاج ())
    الرجاء عدم الرد في الساعات القادمة هذا هو المكان الذي تتعلم فيه.
    يهودا

  22. يهودا
    ليس لديك مشكلة مع الصيغة، لأنها صيغة. إنه لا يصف العالم، بل يعطي نتائج قريبة من الملاحظات في ظل ظروف معينة. وكذلك الحال مع صيغة نيوتن، فصيغة نيوتن هي نتيجة تحليل رياضي لملاحظة أن مدارات الكواكب إهليلجية. على وجه الدقة: من قانون كبلر الثاني يترتب على أن القوة مركزية، ومن القانون الثالث يترتب على أنها تعتمد على مربع المسافة (بمعنى ضعيف بالطبع).

    ما لا أفهمه هو، لماذا تعتقد أنه مسموح لك بالتلاعب بصيغتك لحل أي مشكلة تطرأ؟ هذا يذكرني بالكرتون القديم:
    https://www.google.com/url?sa=i&rct=j&q=&esrc=s&source=images&cd=&cad=rja&uact=8&ved=0ahUKEwiH-smKiL_WAhVJwGMKHSfRDj8QjRwIBw&url=http%3A%2F%2Fscripting.com%2Fliveblog%2Fusers%2Fdavewiner%2F2016%2F02%2F10%2F1000.html&psig=AFQjCNG65X2Kp8ysf6o7wbnitIxOJfe0sA&ust=1506385139473887

    ألا يمكنك أن تفهم أنك قد تكون مخطئًا؟

  23. المعجزات
    أعلم أنه ليس لدي أي خطأ في صيغتي، لكن سؤالي هو لماذا بالنسبة لك لا يمكن أن تكون صيغتي في ظروف معينة نتيجة للنظرية النسبية العامة؟. ولدي طريقتان للإجابة:- الأول، وهو لا أفضله، والذي يمكن أن ينبع بالفعل من النظرية النسبية العامة وأنت مخطئ (ربما لا تغضب)، أو (الخيار الثاني) أن أنت لست مخطئًا ولكن هناك أيضًا إمكانية إجراء تغيير بسيط. على سبيل المثال، تفترض عملية الإضافة حدوث اضطراب محتمل في مجال الجاذبية وهو ما يشبه عملية الإضافة في صيغتي. أقوم حاليًا بإنهاء تكرار النظرية النسبية الخاصة بفرضيتيها الأساسيتين وهما سرعة الضوء الثابتة في أنظمة القصور الذاتي وقوانين الطبيعة واحدة في نظام القصور الذاتي. فيما يتعلق بالعام، فأنا أبحث عن المواد ولم أحقق نجاحًا كبيرًا حتى الآن. وحساب الموترات ليس بسيطًا وهو جديد بالنسبة لي. لدي أيضًا مشكلة تتمثل في أن صيغتي يمكنها في ظل ظروف معينة حل مبادرة كوكب هيما حتى بدون الإسناد. ألا يخلق هذا ازدواجية في الإسناد والمصلحة في الواقع تسبب انحرافا خطيرا؟ سؤال يطرح نفسه: ماذا عن فصائل النجوم الآخرين.؟ باختصار …..أنا مهتم!.
    حتى الآن وداعا. وأستمر في إسناداتي.
    لذا يرجى الرد بلطف وصبر.
    سنة جيدة
    يهودا

  24. سوف آخذ استراحة قصيرة من الحفظ وأجيب على أصدقائي.
    شموليك، أنا أستخدم صيغة نيوتن، التغيير الذي قمت به ليس في القوة الثانية التي تحظى بشعبية كبيرة لديك. إضافتي مختلفة، فهي تضرب الصيغة بأكملها بمصطلح معين. ماذا يعني؟ يبدو لي أن الصيغة النسبية مع انحناء الفضاء تشير إلى الفضاء كشيء "نقي" وخالي من العيوب، إضافتي تعطي النسبية معنى "تعكر" معين قد يكون في الواقع أكثر ملاءمة لعلم الكونيات كما أفهمه. صحيح. فالإضافة ستتضرر من الجاذبية اللامتناهية وستخلق تناقضا مع الإسناد الموجود، لكن ربما يكون من الأفضل الزواج من عروس أقل قليلا من الجمال المثالي.
    بالإضافة إلى ذلك، هناك فرق كبير بين الجسيمات الدافعة والمادة المظلمة وهما مختلفان تمامًا. تمتلك جسيماتي جميع الخصائص التي تمتلكها الجسيمات الحقيقية فيما يتعلق بالاحتكاك وما شابه. "تخيل أنني سأطلب "الحق" في السماح للمادة المظلمة بالحق في أن تكون دون احتكاك؟ كنت سأحل مشكلة الاحتكاك بنفسي منذ وقت طويل ولكن في رأيي ليس هذا الحق الممنوح للمادة المظلمة ". عادل" وسيكون من الظلم أيضًا أن أستخدم هذا الحق.
    شيء آخر، جزيئاتي لا تبتلعها المادة وإذا تم ابتلاعها، يجب بالطبع طردها. أفضّل الاصطدام المرن فقط. لكن فكرة إسرائيل مثيرة للاهتمام للغاية وسأفكر فيها.
    وأدهشني أيضًا قولك إنه لا يوجد توهج بالقرب من الشمس، حسنًا، لا داعي للركض إلى الفضاء، يكفي رؤية كسوف الشمس ورؤية التوهج حوله.
    وأريد أيضًا الرد على المعجزات لذا سأنتهي هنا.
    أرجو الرد بلطف وكل عام وأنتم بخير
    يهودا

  25. يودا
    العلم لردود الفعل العلمية. الفيسبوك مخصص لتبادل الخبرات الشخصية. إذا كان لديك رد علمي فأجب. اقطع لنا مسار حياتك. ليست مثيرة للاهتمام للغاية.

  26. يودا
    العلم لردود الفعل العلمية. الفيسبوك مخصص لتبادل الخبرات الشخصية. إذا كان لديك رد علمي فأجب. اقطع لنا مسار حياتك. ليست مثيرة للاهتمام للغاية.

  27. لقد انتهيت للتو من قوانين نيوتن وكيبلر في الميكانيكا وأنا الآن في الجاذبية وبعد النسبية الخاصة وبعد النسبية العامة عندها فقط سأتحقق من التغيير المحتمل في صيغتي. مازال هنالك وقت. أنا حاليا أقوم برعاية حفيدتي. ليلة سعيدة جدا

  28. شموليك،

    العبارة التي تتحدث عنها ليست عبارة ذات اسم مرتبط بشخص ما. ما يظهر في البداية هو حساب ساذج لسعة التشتت لـ 4 جرافيتونات (وهو أمر مربك بعض الشيء، ولكن في الواقع ما يمثله في الواقع هو احتمال اصطدام اثنين من الجرافيتونات ببعضهما البعض وبعد التفاعل سيكون هناك اثنين الجرافيتونات المتبقية التي سوف تتناثر من بعضها البعض بزخم معين وزاوية معينة). وهذا حساب تم إجراؤه بسذاجة من خلال "الترويج" لنظرية الجاذبية الكلاسيكية باعتبارها نظرية كمومية. تبدو الصيغة مخيفة بعض الشيء للأشخاص الذين ليس لديهم خبرة في هذا المجال لأنها تستخدم رموزًا مريحة ومدمجة للغاية والتي هي ببساطة غير مألوفة لعامة السكان (الأقواس المثلثة والمربعة).

    الصيغة الثانية التي يقدمها هي أيضًا بلا اسم على حد علمي، وهي سلسلة مكافئة لنفس العملية الحسابية السابقة. وكما يوضح، فإنه يعطي نفس النتيجة عند الطاقات المنخفضة ولكن عند الطاقات العالية ينظم التبديد. من حيث المبدأ، هذه ظاهرة شائعة جدًا وهي حجر الزاوية في نظرية الأوتار - نظرًا لحقيقة أن السلسلة لها طول محدود، في جميع الحسابات يظهر الحد الأدنى للحجم أو المسافة في التفاعل الذي يلغي اللانهاية (لأنه في نظرية المجال عادة ما تأتي اللانهاية من حقيقة أن الجسيمات يمكن أن تقترب من بعضها البعض حتى مسافة 0 وتتبادل الزخم اللانهائي).

    كما قلت، هذه الصيغ المحددة (قبل وبعد التنظيم) ليس لها اسم أعرفه وأعتقد أنك ستواجه صعوبة في العثور على معلومات عنها، ولكن يمكنك إلقاء نظرة على شيء مشابه. إن سعة فينيزيانو هي سعة تشتت مماثلة (لا نتحدث عن الجرافيتونات ولكن عن الجسيمات الأخرى) وكانت هذه هي المرة الأولى التي يلاحظ فيها الناس مثل هذه الظواهر، مثل التبديد المتلاشي والاتصال بالأوتار. في الواقع، يعتبر الكثيرون أن حسابات غابرييل فينيزيانو هي ولادة نظرية الأوتار. أنا متأكد تمامًا من وجود تفسير لهذه المعادلة حتى على المستوى الذي لا يتضمن الكثير من الرياضيات التي يمكن العثور عليها على الإنترنت.

  29. ألبينتيزو,
    سؤال لا علاقة له بالمناقشة بل بنظرية الأوتار،
    في هذه المحاضرة تتحدث ناع عن صيغة تفسيرها سلسلة: الدقائق 53-55. ما هو اسم الصيغة؟ إذا كان لديك أي شيء تود إضافته حول الصيغة، فإن ويكيبيديا ستكون معقدة للغاية، ويسعدني سماعها
    https://youtu.be/U47kyV4TMnE

  30. عدت إلى المنزل. كان من الممتع أن أكون مع عائلتي وأحفادي. المعدة ممتلئة إلى أبعد الحدود. من فضلك أعطني بضعة أيام لدراسة موضوع الجاذبية والإسناد. ألبانزو وشموليك. كما أنني أخذت تعليقك الأخير بعين الاعتبار. دعونا نفهم روح النسبية. آمل أن أجد كتابًا عن هذا الموضوع من بين المئات التي أملكها.
    يوم جيد وسنة جديدة سعيدة!
    يهودا

  31. يهودا،

    سأحاول أن أشرح: النسبية والجاذبية النيوتونية مختلفتان. إنهما نظريتان مختلفتان بمعادلات مختلفة، وبالتالي فإنهما يفسران ظواهر مختلفة ويفعلان ذلك بطرق مختلفة. إذا أخذت النظرية النسبية ونظرت إليها في منطقة معينة (حيث تكون كثافة الطاقة منخفضة جدًا) لظروف معينة (جسيمات ضخمة)، فستحصل على معادلة نيوتن *تقريبًا*. لكن لاحظ بعض الأشياء:

    1. يتم ذلك فقط في ظل ظروف معينة. على سبيل المثال، بالنسبة للضوء حتى في المجال الضعيف، هناك سلوك مختلف جذريًا بين النسبية ونيوتن.

    2. هذا مجرد تقدير تقريبي. التقريب يعني أن النتائج *مختلفة*، ولكن لكي تلاحظ الفرق عليك أن تنظر عن كثب. هذه تفاصيل مهمة جدًا لأنه بمساعدة أجهزة القياس الدقيقة، من السهل التمييز بين نظريتين، حتى لو كانتا تعطيان نفس المعادلات تقريبًا.

    النظرية التي تقدمها تقارب نيوتن. لكن نيوتن والنسبية شيئان مختلفان، وهي لا تعطي أي تقدير تقريبي للنظرية النسبية. على سبيل المثال، المثالان اللذان ظهرا هنا - علاقتها بالضوء هي مثل علاقة نيوتن (وليست مثل النسبية)، أي علاقة خاطئة. المثال الثاني هو التغير في الزمن مع التغير في المجال - مرة أخرى، بالنسبة لك، الأمر مثل نيوتن، لكن نيوتن والنسبية مختلفان. فيما يتعلق بذلك، فإن الساعات ذات الارتفاعات المختلفة تظهر وقتًا مختلفًا، وهذا بالفعل ما رأيناه في التجربة.

  32. يهودا،
    لو كنتم قد شاهدتم فقط الخمس دقائق اللطيفة التي أرسلتها على اليوتيوب، لكنتم سمعتم ستيفن واينبرغ يقول أن النسبية تنبع من فكرة مركزية واحدة ومن تلك الفكرة تستمد معادلة مجالها. وهذه المعادلات، عند تقريب الأجسام المتحركة ببطء، تتقارب مع معادلة نيوتن، وعلى وجه الخصوص، مع مربع المسافة. ليس هناك خيار سوى قبول القوة الثانية ولا توجد طريقة للعب بمعادلات المجال دون تدمير الفكرة الأساسية وراء النسبية (على حد تعبيره: سيؤدي ذلك إلى هراء). كما كتب ألبانتيزو أشياء مماثلة. لذا، للمرة الألف، لا فائدة من اللعب بالمعادلات للحصول على النتيجة التي تريدها فقط.

    ومن المفارقات، أنتم الذين تعارضون المادة المظلمة، تقترحون مادة مظلمة أخرى لا تفسر الظواهر الموثقة جيدًا فحسب، بل تدمر الفيزياء الكلاسيكية أيضًا. نيوتن الذي تستمد منه معادلته (لماذا منه، لأن هذا ما تعرفه ولهذا ارتديت ملابسك. لا يوجد سبب آخر). في حين أن نيوتن لم يقدم أبدًا تفسيرًا للجاذبية (انظر مقولته الشهيرة) من نظريته، فإنه يترتب على ذلك أن تأثيرات الجاذبية لحظية (وليس بسرعة الضوء) ولكن من حيث المبدأ لا توجد مشكلة في قبول انحناء الضوء بالقرب من كتلة تحتها. (ينحني الضوء لأنه ينحني، لا أعرف لماذا يقول نيوتن. فقط ليس كافيًا بالنسبة للنسبية، في حالة كوكب عطارد). معك، بما أنك تقترح آلية التصادمات الميكانيكية، فلا توجد طريقة لشرح سبب انحناء الضوء بالقرب من الكتلة، لأن اصطدام كرات البلياردو الصغيرة بالضوء لن يؤدي ببساطة إلى انحرافه في الاتجاه الصحيح. النور ليس كرة قدم وجزيئات يهوذا ليست حراس مرمى يعملون معا. عندما يتشكل قوس قزح في السماء، فذلك لأن الماء يرتب نفسه بطريقة فريدة بحيث إذا كانت الشمس خلفك، فإن انعكاس الضوء يخلق قوس قزح جميلاً. معك تتحرك الجزيئات بشكل عشوائي في أي اتجاه ولا يوجد سبب لأن ينحني الضوء دائمًا كما هو في الواقع. نتوقع رؤية انحناء مختلف للضوء في كل مرة، وبشكل أكثر عشوائية

    إذا ضربت جسيماتك الضوء وتشتت الضوء، فأنا لا أفهم لماذا لا يوجد وهج بالقرب من الشمس يملأ السماء بشكل موحد (على غرار ما يفعله الغلاف الجوي). بالنظر من القمر نحو الشمس، أتوقع ألا تكون الشمس نقطة متوهجة في الفضاء المظلم، بل أن تكون توهجًا منتشرًا بشكل منتظم. وسأكون سعيدا لسماع إشارة إلى هذا.

  33. يهودا
    إذا كانت التصادمات مرنة تمامًا، فلن يكون هناك "تجاذب" بين جسمين. فمن ناحية، ستكون هناك جسيمات لن تصطدم بالجسم A بسبب اختباء الجسم B. ولكن، ستكون هناك جسيمات ستضرب النقطة A على وجه التحديد نتيجة لعودتها من النقطة B.
    على وجه الخصوص، لن يتغير تركيزهم.

    وفي حالة الاصطدام بالبلاستيك، في رأيي، سيكون هناك انخفاض في تركيز الجزيئات في منطقة الجسم.

  34. لألبانزو والمعجزات. شكرا لإجاباتك التفصيلية. يبدو أن هناك شيئا من حيث المبدأ لا أفهمه، فلماذا إذا كانت صيغة الكون البسيط تكاد تكون مطابقة للصيغة النيوتونية على مسافات صغيرة، ألا تسمح النسبية بأن تكون بديلا؟. يبدو أن ألبانزو على حق في أنني بحاجة إلى المعرفة الكاملة بالعلاقات. وسؤال للإعجاز، إذا كان تصادم الجزيئات سيكون مرنًا تمامًا، فهل سيكون تركيزها بالقرب من الأجسام أكبر؟ غذاء للفكر. لذا يرجى التحلي بالصبر وأشكركم على إجاباتكم وصبركم، والأهم من ذلك كل عام وأنتم بخير!؟

  35. يهودا،

    أنت تكرر باستمرار أن التصحيح الذي تقترحه للجاذبية النيوتونية لا يتعارض مع النظرية النسبية على المسافات القصيرة. عليك أن تفهم أنه كذلك. على مسافات قصيرة، النظرية التي تقترحها (إذا كنت تستطيع أن تسميها كذلك) تعطي معادلات نيوتن. النسبية لا. عليك أن تجلس للحظة وتركز وتفهم الأمر. وفي النسبية هناك معادلات أخرى تسمى معادلات أينشتاين للمجال. إنها *ليست* معادلات نيوتن، وهي التي تسمح بالتأثيرات النسبية مثل جاذبية الأجسام ذات الكتلة 0 (مثل الفوتون)، وتغيير تدفق الوقت قليلاً، وما إلى ذلك. لا يمكنك أن تقول إن معادلات الجاذبية بالنسبة لك مثل الأس المزدوج لنيوتن، ولا أنها لا تتعارض مع النسبية - لأنها تتعارض صراحة مع النسبية.

    اجلس دقيقة استمع. استوعب ثم أجب. وإلى أن تقبل ذلك، لن يتمكن أحد من الوصول إلى أي مكان. ألا تعتقد أنه قبل أن تقول أن نظريتك تتفق مع النسبية للمسافات القصيرة، يجب أن تتعلم القليل عن ماهية النسبية؟ لو كنت قد بذلت جهدًا لدراسة الموضوع الذي كنت تتحدث عنه كثيرًا لسنوات، لكان من الواضح لك أيضًا أن ما اقترحته حتى الآن يتناقض بشكل مباشر مع النسبية. لذلك، هناك ثلاثة احتمالات: إما أن يكون نموذجك خاطئًا (وهذا هو الاحتمال الصحيح بالطبع، كما رأينا بالفعل من خلال إظهار كمية لا حصر لها من الأخطاء التي اخترت تجاهلها)، أو أن نموذجك صحيح والنسبية خاطئة ( بالطبع هذا احتمال غير صالح لأن النسبية تفي بجميع التجارب التي أجريت منذ 100 عام)، أو أن نموذجك يشرح بأعجوبة نفس الظواهر التي شوهدت في التجربة مثل تغير تدفق الزمن، وتأثير الجاذبية على الفوتونات، وما إلى ذلك. ، ولكن بطريقة مختلفة. وفي هذه الحالة يقع عليك عبء الإثبات، وبدلا من التهرب، عليك أن تجيب على المعجزات نهائيا وتشرح له كيف تفسر "نظريتك" هذه الظواهر.

  36. يهودا
    هل تحاول الحصول على كعكتك وتناولها أيضًا؟ 🙂
    يجب أن يعمل شرحك أيضًا على مسافات قصيرة. وإلا لكان فنجان قهوتي يعمل بمفردي في الغرفة.
    لماذا يكون هناك تركيز عال من الجزيئات في منطقة الكتلة؟ يجب أن يكون الأمر على العكس من ذلك - يتم ابتلاع بعض الجزيئات، أو على الأقل القليل منها داخل الكتلة. إذا وضعت كتلة من البلاديوم في وعاء يحتوي على الهيدروجين، فإن تركيز الهيدروجين في منطقة الكتلة سوف ينخفض.
    ماذا يعني أن الفوتون لديه طاقة تعادل الكتلة؟ تفسيرك ميكانيكي، وليس مبنيا على كل أنواع التحويلات الوهمية للفيزياء من قبل العلماء الجهلاء 🙂
    يهودا – لا أفهم كيف لا ترى التناقض في القول إن مسار الفوتون يتسارع "لأنه يمتلك كتلة". يبدو الأمر كما لو أن فنجان قهوتي ينجذب إلى الأرض لأن له كتلة. هل حقا لا ترى المشكلة؟؟

  37. لذلك انتهينا من الكعك أيضًا. دعنا ننتقل إلى عيون الجاذبية. لماذا ينحني شعاع الضوء في الممر بالقرب من ميسا؟ لنفس السبب الذي تفكر فيه، النسبية. ففي نهاية المطاف، ليس لدفع الجاذبية أي مقاومة على مسافات صغيرة للنظرية النسبية. ولكن بالإضافة إلى ذلك، يمكن أيضًا أن يكون الشعاع منحنيًا بسبب تركيز أعلى من المتوسط ​​لجزيئات الدفع في منطقة كتلة العدسة. والاحتمال الثالث هو أن يكون للفوتون طاقة تعادل الكتلة وهو الذي ينجذب إلى الشمس وينحني الشعاع بأكمله. والاحتمال الرابع هو وجود غاز رقيق حول الشمس أو أي كتلة. ومجموع أربعة ممكن للجر والانحناء، من هو أكثر ومن هو أقل. وأنا لا أستبعد أي احتمال. لذلك لا أرى أي مشكلة مع إيدوس. لاحظ أن دفع الجاذبية لديه أيضًا إمكانية التنافر السلبي إذا مر الشعاع عبر منطقة ضغط منخفض. حتى الآن. سنة جيدة. يرجى الرد بلطف. يهودا

  38. يهودا،
    مثل المعجزات، أنا لا أفهم تفسيرك. لماذا ينحني شعاع الضوء في الكتلة؟
    هل يمكنك أن تشرح مرة أخرى ما الذي يحدث هناك بالضبط؟ كيف تفكر فيما يحدث هناك؟ ما نوع تشتت الضوء الذي يمر عبر التصادم مع هذه الجسيمات؟

    ألبينتيزو,
    لقد فهمت الشرح. شكرا.

  39. شموليك،

    إنها مسألة دلالية قليلاً. أنا شخصيا أدرس الفيزياء الرياضية وأعتقد أنه لا يوجد سبب لتفسير أي عملية تحدث في الطبيعة باستخدام الحدس الكلاسيكي (على سبيل المثال، تصادم الكرات). ولذلك، أعتقد أنه يمكن القول أن الرعاية العامة تفسر الجاذبية (حتى وابل من الأسئلة التي لا نهاية لها، كما قلت). ولكن من الواضح بالنسبة لي أيضًا أن أغلب الناس - حتى بين علماء الفيزياء - يميزون بشكل أكثر وضوحًا بين التفسيرات البديهية والتفسيرات المستمدة من الرياضيات. الجاذبية الميكانيكية (سواء كانت كلاسيكية مثل تلك التي يتحدث عنها يهوذا وإسرائيل، أو الكم كما في نظرية الأوتار) تفسر أصل الجاذبية عن طريق اصطدام الجسيمات، وهو أمر يسهل علينا فهمه وتخيله. حتى أن نظرية الأوتار تشرح ذلك بمزيد من التعمق، لأنها تعطي أيضًا إجابة لسؤال من أين تأتي هذه الجسيمات ولماذا توجد على الإطلاق (إنها ببساطة أوتار مغلقة - مثل حلقات مصنوعة من الأسلاك. ويمكن تمزيق كل وتر إلى قسمين). أوتار، أحدها ينغلق على نفسه، وهذا سيُفهم على يدي على أنه جسيم أطلق جرافيتون - أي جسيم يقوم بتفاعل جاذبية مع بيئته، وشرح نظرية الأوتار جميل جدًا ودقيق جدًا على سبيل المثال، يوضح سبب وجود الجاذبية دائمًا في أي نظام وأن كل جسيمين سوف يؤديان إلى تفاعل الجاذبية، على عكس، على سبيل المثال، الكهرومغناطيسية التي لا توجد في كل نظام وليس بالضرورة أن يقوم كل جسيم بمثل هذا التفاعل).

    أعتقد أن هناك فرقًا جوهريًا بين قانون الجاذبية العالمي الذي لا يعتمد على نموذج الجاذبية، والجاذبية الميكانيكية التي تعتمد على نموذج. بين النسبية العامة والجاذبية الميكانيكية الفرق أرق، لكنه لا يزال موجودا، لأن مصدر التفاعل بين كثافة طاقة الجسيم وبيئته (=قوة الجاذبية) في النسبية العامة ينبع من اعتبارات رياضية مجردة. بالنسبة لي، فهي ليست أقل جودة، لكن من الصعب إنكار القيمة الجمالية للنظرية التي تفسر الظاهرة بوسائل بصرية بسيطة. ولو لم يكن خطأ..

  40. طيب الجميع يعلم الآن أن مبدأ الجاذبية عن طريق دفع الجاذبية يعمل وبما أنه لا يعمل مع جزيئات الهواء تحت الضغط العادي وسيعمل في الهواء الرقيق لذا أتمنى أن يوافق الجميع على صيغتي لجاذبية الكون البسيط وهي صيغة نيوتن مع الإضافة.. هل تقبل المعجزات؟

  41. يهودا
    لم أفهم ما تقيسه. نحن نعلم أن هناك قوى جاذبية بين الأجسام القريبة. هناك تجارب يجب أن تأخذ هذه القوى بعين الاعتبار، على سبيل المثال في قياسات رياح كازيمير.
    إذن أنت تقول أننا سنرى انجذابًا بين الأجساد؟ قام كافنديش بقياس هذا الانجذاب في الهواء الطلق منذ 200 عام. هل تقول أن هذه التجربة لم تتم؟

  42. يهودا
    والتفسير هو من نظرية الكم، وأيضا من النظرية النسبية. حتى في الزجاج تتحرك الفوتونات بسرعة c. لكن الفوتونات تمتصها الذرات في الطريق، وبعد فترة قصيرة تنبعث مرة أخرى. وتستمر هذه العملية وفي النهاية ينبعث فوتون من الذرة في نهاية المسار. وبسبب حفظ الزخم يتغير مسار الفوتون عند الدخول والخروج من الزجاج، وبالتالي تحدث ظاهرة الانكسار.
    في النظرية النسبية، يجب أن يتحرك الجسيم الذي ليس له كتلة ساكنة دائمًا بسرعة c، وإلا فلن يكون له زخم (ومن ثم هناك مشكلة فيما يحدث عندما يصطدم بجسيم آخر).

    أنت لا تقبل افتراضات النظرية النسبية، وبالتالي لا يمكنك استخدام ما كتبته "ضدي".

    بالمناسبة - نرى خطوطًا طيفية في الضوء من الأجسام البعيدة. إذا كانت سرعة الضوء مختلفة في الماضي فيجب أن نرى تحولاً في هذه الخطوط. بالإضافة إلى ذلك، إذا تمت مناقشة سرعة الضوء في الماضي، فيجب أن تكون الكيمياء مختلفة أيضًا، لأن القوة الكهرومغناطيسية تتأثر بشكل مباشر بـ c. وهذا وحده يجب أن يبطل ادعاءك بشأن التغير في سرعة الضوء...

  43. والمعجزة، أن الهواء في تجاربي يجب أن يكون رقيقاً بحيث يكون متوسط ​​المسار الحر لجزيئات الهواء أطول من المسافة بين الأجسام التي تسبب الجاذبية. إذا لم يكن الأمر كذلك، لا
    سيتم إنشاء الجاذبية تقريبًا بين الأجسام. أتساءل عما إذا كنت قد فهمت هذا التلويح باليد. سنة جيدة.

  44. يهودا
    أنت ساحر التلويح بيديك 🙂 ماذا تقصد بأن "الجاذبية النيوتونية" هي تفسير لإيدوش؟ ففي النهاية، أنت تدعي أن لديك تفسيرًا للجاذبية، وترفض بازدراء أي مشكلة في تفسيرك.
    اشرح لي شيئًا بسيطًا جدًا: يتحرك الفوتون عبر الزجاج بسرعة 200,000 كيلومتر في الثانية، وبمجرد مغادرته - يصبح على الفور بسرعة 300,000 كيلومتر في الثانية. كيف تفسر نظريتك هذا؟

  45. شموليك. يمكن إجراء الإيدوش في عدة حالات. أ- مرور الضوء في مناطق مختلفة الكثافة. ب، في الجاذبية النيوتونية إذا كانت الفوتونات تمتلك خاصية المادة. ج- في الجاذبية النسبية. أفضّل A في الشرح، لكن ليس لدي مشكلة مع الاثنين الآخرين. سنة جيدة.

  46. يهودا
    ما هي هذه التجربة؟ تبين أن هناك جاذبية بين الكرات؟ لن تؤثر كمية الهواء على التجاذب بين الكرات، لأن التصادمات مرنة. هل تريد أن تظهر أن هناك الجاذبية؟

  47. وفيما يلي تجربة Sabdarmish A.. داخل معمل حيث الهواء رقيق لإسقاط كرتين. خلال فصل الخريف، من المفترض أن تقترب الأجسام من بعضها البعض بسبب تأثير جزيئات الهواء التي ستلعب دور الجزيئات الدافعة. كيف تبدو الآن؟ تجربة Sabdermish ثانية وأبسط، في حاوية كبيرة تحتوي على هواء رقيق، قم بتعليق كرتين. ستكون المسافة بينهما أصغر من المسافة بين النقاط المعلقة. سيكون هذا دليلاً على العمل المفيد لجزيئات الهواء. لقد قمت بنقل شرائح اللحم إلى عمق الثلاجة. يرجى الرد بلطف. سنة جيدة!

  48. ألبينتيزو,
    بالنظر إلى تجاهل إنجازات الثلاثين عامًا الماضية، لماذا لا يزال تفسير النظرية النسبية - انحناء الزمان والمكان - ليس تفسيرًا لأصل الجاذبية؟ من الواضح أنه يمكن للمرء دائمًا طرح أسئلة إضافية عن السبب، على سبيل المثال، لماذا تشوه الكتلة الفضاء ويستمر في التساؤل عن السبب (حتى تكون الإجابة الوحيدة هي اصمت وأكل رقائقك... على حد تعبير لويس سي كيه). وبنفس الطريقة يمكن للمرء أن يسأل لماذا الأسئلة حول الجاذبية الميكانيكية.

    لماذا الأسئلة دائما إشكالية. يشرح ريتشارد فاينمان السبب
    https://www.youtube.com/watch?v=3D2RaDVkylY

  49. يهودا
    لم أظن للحظة أن ذلك سيقنعك...
    أرجو أن توضحوا لي، كيف يمكن أن نرى مجرات تبتعد عنا بسرعة تفوق سرعة الضوء؟ بشكل عام، لماذا في نظريتك يتحرك الضوء في الفراغ بسرعة c؟ وكيف يمكن للجسيمات أن تتحرك بالفعل بسرعة أكبر من سرعة الضوء؟

  50. معجزات. الانبعاث هو نتيجة لشعاع ضوئي يمر بالقرب من الكتلة ثم ينحني. لا يهم من أين يأتي، ولهذا السبب ثبت أنه حديث من ممر قريب وليس مليارات السنين الضوئية. ومن الممكن أيضًا أن يكون التلوث قد نشأ بسبب شيء آخر، مثل مرور شعاع الضوء في تركيز جزيئات الجاذبية الدافعة. قررت في هذه المرحلة أن أترك شريحة اللحم في الثلاجة. سنة جيدة. يرجى الرد بلطف.

  51. إسرائيل، تجربتي مع الهواء الرقيق في سفينة عندما تعمل جزيئات الهواء كجزيئات دافعة تبدو أكثر إقناعًا بالنسبة لي إذا كان مطلوبًا إجراؤها في حالة انعدام الوزن وكانت أكثر تكلفة. سأحاول التخطيط لتجربة على الأرض.

  52. يهودا
    أنت لم تفهم تماما تفسيرك لماذا تسارع الجاذبية لمجرة على مسافة 8 مليارات سنة ضوئية، لمجرة على مسافة 4 مليارات سنة ضوئية، لا يؤكد أن النسبية العامة تعمل حتى على مسافات كبيرة جدا. وسأكون ممتنا إذا أمكنك توضيح ما شرحته.

    بالمناسبة - في رأيي أن الإشعاع الكوني يؤكد أيضًا النظرية النسبية. خلاف ذلك - لماذا تتحرك كل هذه الجسيمات النشطة بدقة (تقريبًا) بسرعة الضوء في عصرنا؟ إذا تم إنشاؤها منذ وقت طويل، فيجب أن تكون أبطأ - إذًا ليس من المنطقي أن تتسارع على طول الطريق.

  53. يهودا

    لماذا الهواء؟ ويمكن القيام بذلك بوسائل ميكانيكية بسيطة.

    سوف نأخذ قرصين معدنيين كبيرين ومثقبين. الأقراص متقابلة تمامًا مع بعضها البعض.

    سنضع الجهاز في حجرة تتطاير فيها الكرات الصغيرة من كافة الاتجاهات، ويمكننا توجيه سرعتها كما نشاء. صورة لفكرة ليساج.

    يخبرنا ليساج والمنطق أن الأقراص ستغطي بعض الكرات، لذا ستتحرك تجاه بعضها البعض. لقد حصلنا على جاذبية اللازاج.

    المشكلة هي أنه إذا تحرك القرص بالنسبة للخلية، فإن الكرات ستؤثر عليه بقوة معاكسة. هذا هو احتكاك فاينمان.

    الآن بدلا من الأقراص سوف نستخدم البلاستيسين. (من المرغوب والممكن استخدام أجسام مشحونة كهربائياً يمكن أن تخترقها الكرات دون تشويه، لكن البلاستيسين المرن الذي يسمح للكرات بالمرور من خلالها دون تشويه سوف يقوم بهذه المهمة أيضاً).

    لأنه وفقًا لمبدأ البندول الباليستي فوق سرعة معينة، تقل القوة التي تمارسها الكرات على البلاستيسين بشكل كبير، لذلك لن نواجه احتكاكًا هذه المرة. لا ينبغي أن تكون الكرات بسرعة لا نهائية، ولكن عالية بما يكفي للسماح بالجاذبية.

    يدعي إن سي أن الجاذبية ستُلغى عندما تتحرك الأقراص فوق سرعة معينة، وهذا صحيح ولكنه غير ذي صلة. لرؤية ذلك، لنفترض أن DHA يتفاعل فقط مع الجسيمات التي تتراوح سرعتها النسبية بين 1 و1000,000 م/ث. ولذلك عندما يتحرك حول الشمس تبلغ سرعته حوالي 30,000 م/ث. يتم تقليل الجاذبية قليلاً ولكن بالتأكيد لم يتم القضاء عليها. ناهيك عن القمر الذي تكون سرعته أقل بكثير.

    في رأيي، قد نحصل أيضًا على القصور الذاتي: طالما أن القرص يتحرك بسرعة ثابتة، فإن القوة المؤثرة عليه تتوازن. عندما ينتقل من سرعة إلى أخرى، فإن مجموع القوى المؤثرة عليه لا يساوي 0 ولا يتم إعادة ضبطه إلا عندما يصل إلى سرعة ثابتة جديدة. لكنني لست متأكدًا حقًا من ذلك.

    هذا ما لم أفهمه بعد بشأن مجال هيغز: كيف يعرف أن الجسم يتسارع؟ لماذا عندها فقط يقاومها وليس بسرعة ثابتة؟

    وجهت السؤال إلى أستاذ من سان دييغو أوضح لي أن حقل هيغز يشبه المحيط الذي يقاوم السفن المبحرة فيه. قال فقط إنني وجدت التعبير في حجته، لكنه لم يوضح ما هو الحل.

  54. إسرائيل،

    إذا قرأت ردي مرة أخرى سترى أن محتواه لا علاقة له بالمكان الذي سمعت فيه هذا الاقتباس. بالنسبة لي، كان من الممكن أن يقولها طفل عمره 6 سنوات يعاني من تأخر في النمو أو خوان مالديسانا ولن يغير ما قلته. إذا كنت تتوقع مني أن أنبهر لأن الشخص الذي قال هذا هو عالم فيزياء، لدي أخبار لك… 80% من الأشخاص الذين أتحدث إليهم هم فيزيائيون يعملون في الجاذبية ولا أحد منهم فوق الحديث عن الهراء بين الحين والآخر. لا يعني ذلك أنني أدعي أن هذا الادعاء هراء، أعتقد أنني أوضحت تمامًا في ردي أنني أعتقد أنه غير دقيق لأنه يتجاهل أبحاثًا ضخمة ومثمرة للغاية لأنه لم يتم إثباته معمليًا بعد. نعم، إذا أخرجت نظرية الأوتار من الصورة، فأعتقد أن هذا الادعاء صحيح. ولكن بالنسبة لي يبدو من المضحك أن نخرج من الصورة واحدًا من أعظم إنجازات الفيزياء النظرية على الإطلاق. قبل خمس سنوات، هل كان من الصحيح تجاهل النموذج القياسي وكأنه غير موجود والقول بأنه لا يوجد فهم لسلوك المادة والقوى الأساسية في الطبيعة، فقط لأن بوزون هيغز (الذي يعد مكونا أساسيا في الكون) نموذج) لم يتم العثور عليه بعد في التجربة؟ بالنسبة لي، هذا نهج خاطئ، ولكن إذا كان هذا ما تريد تصديقه، فأنت على حق.

  55. ألبينتيزو,
    وما هي سرعة الأقزام؟ فيما يتعلق بالكتلة، اعتقدت أنه على الرغم من وجود جسيمات كتلتها صفر، لكنني كنت أتساءل عما إذا كان هناك حد نظري لمدى خفة الكتلة إذا كانت أكبر من الصفر

    معجزات,
    يوتيوب لطيف. ألا يجب أن تؤثر هذه الحدود أيضًا على جزيئات يهوذا؟

    يهودا،
    إذا كنت لا تصدق ستيفن واينبرغ الذي يقول بصوته أنه في ظل افتراض الزمان والمكان يتم تحديد معادلات النسبية وأن معادلات النسبية ليس لها أي إمكانية إلا الوصول إلى معامل 2 في التقريب المناسب، فلماذا هل تصدق ترجمتي؟ لا أعتقد لذلك لا.
    لنفترض، لماذا ينحني الضوء تحت تأثير الجاذبية الميكانيكية؟

  56. صباح الخير لكل أصدقائي أينما كانوا
    سأرد بإيجاز على أحدث تعليقاتك الذكية
    إسرائيل
    إن النموذج المختبري الذي تتحدث عنه مثير للاهتمام، ولكن هناك أمرين مهمين يجب القيام بهما. أول من استخدم الهواء الرقيق للحصول على جسيمات ذات مسار حر متوسط ​​طويل نسبيًا بالنسبة لمسافة الأجسام المنجذبة. حساباتي للضغط المطلوب هي عشرة أس سالب ثمانية أجواء. ثانياً، سوف تتداخل الجاذبية الأرضية مع التجربة، الخلاصة: يجب إجراء التجربة في محطة فضائية والتي، أثناء دورانها حول الأرض، ستعطي اتجاه دوران النظام الشمسي المصغر الذي يمكن القيام به في التجربة. وإذا كان أي شيء ... ربما دوران الشمس والمجرة هما اللذان يساعدان على التغلب على الاحتكاك...؟
    بالطبع أنا أتفق مع طلبك بأن دفع الجاذبية يجب أن يحظى بالشرف باعتباره النظرية الوحيدة التي تحاول تفسير طبيعة الجاذبية.
    المعجزات
    صدقوني، أنا لا أتجاهل أي ملاحظة قد تظهر انحرافًا عن دفع الجاذبية. بالإضافة إلى ذلك، لا أفهم كيف يكون الأمر، لغرض شرح الجاذبية، أن الناس مستعدون لثني الكون بأكمله ولكنهم غير مستعدين لبعض كرات البلياردو التي تدور حولها....
    ألبانزو
    لقد جلب أقزامك البسمة إلى حلقي ولكن لماذا تختار الأقزام. السير على الله والملائكة. سيكون لديك المزيد من المعجبين. وزير التربية والتعليم لدينا مثلا. وفيما يتعلق بنظرية الأوتار، ليس لدي رأي.
    يوم جيد جدا يا أصدقائي
    وكل عام وأنتم بخير
    يهودا
    http://yekumpashut.freevar.com/

  57. سبب الجاذبية
    الاثنين 18 سبتمبر 2017 الساعة 6:42 مساءً
    snuz2001 snuz2001@aol.comإخفاء
    إلى //////////@mit.edu
    مرحباً سيد بيرتشينجر.
    لقد كنت حاضرًا في محاضرة ألقيتها في جامعة كاليفورنيا قبل بضع سنوات، وإذا كنت أتذكر بشكل صحيح، فقد قدمت كتابة على السبورة تقول:

    لا يعرف العلماء سبب الظاهرة الأكثر وضوحًا في الطبيعة: الجاذبية.

    1. هل ذاكرتي صحيحة؟

    2. هل لا يزال الوضع على هذا النحو؟

    شكر،

    إسرائيل شابيرا.

    تم الإرسال من بريد AOL Mobile

    كان المتلقي في ذلك الوقت رئيسًا لقسم الفيزياء في معهد ماساتشوستس للتكنولوجيا

  58. الجاذبية الميكانيكية ليست النظرية الوحيدة التي تشرح أصل الجاذبية. هناك العديد من النظريات الأخرى التي تفعل ذلك، على سبيل المثال: الفضاء مليء بالأقزام غير المرئية. إنهم يحبون كل شيء قريب، لأنهم أقزام ويصعب عليهم المشي لمسافات طويلة (أرجل قصيرة). لذلك فهم يحاولون باستمرار دفع الأشياء لتكون قريبة من بعضها البعض. نظرًا لأن الجسدين قريبان جدًا بالفعل، فإن ذلك يزيد من حافزهما ويأتي المزيد من الأقزام لمساعدتهما، ولهذا السبب يصبح السحب أقوى.

    وعلى الرغم من أن هذه النظرية أكثر تأخرًا (كثيرًا) من الجاذبية الميكانيكية، إلا أنها خاطئة تمامًا. وبعبارة أخرى، لا أستطيع أن أفهم لماذا يجب على المرء أن "يعطيها الاحترام الذي تستحقه" لشرح أصل الجاذبية إذا كان هذا عقيدة خاطئة. هناك نظريات خاطئة لا حصر لها تشرح أصل العديد من الظواهر. وكما قلت سابقًا - على الرغم من قيمتها الجمالية، فإن الجاذبية الميكانيكية في نهاية المطاف هي نموذج يفشل في محاولة وصف الجاذبية كما فهمناها منذ 350 عامًا، ناهيك عن الجاذبية الحديثة.

    أتفق مع الادعاء بأن النموذج ينتج الجاذبية، وأوافق على أنه إذا تم حل مشكلاته بأعجوبة (على الرغم من إصراري على أن الحل المقترح هنا ليس حلاً حقيقيًا لأنه ليس حلاً فيزيائيًا)، فإنه سينتج جاذبية تتوافق مع القياسات التي قمنا بها كان قبل النسبية العامة. لا أستطيع أن أفهم ما هي قيمة هذا اليوم للبحث (له قيمة ترفيهية، وربما أيضًا قيمة تربوية حول بناء النماذج).

    أنا لا أتفق تماما مع الادعاء بأن العلماء لا يفهمون أصل الجاذبية. هذا الادعاء صحيح فقط إذا تجاهلت المجال البحثي الأكثر فائدة في مجال الجاذبية بشكل خاص (وفي رأيي في الفيزياء بشكل عام) في الثلاثين عامًا الماضية. تعطي نظرية الأوتار تفسيرًا ممتازًا لأصل الجاذبية، فهي تتوافق مع جميع القياسات الحديثة، ولم يتم دحضها نظريًا (مثل الجاذبية الميكانيكية) فحسب، بل تحتوي أيضًا على الكثير من الأدلة النظرية. ومن المهم أن نتذكر، ولهذا السبب قلت لا أوافق تماما بدلا من أن أقول لا أوافق تماما، أن ذلك لا يزال على الورق ولم يتم إثباته في المختبر.
    ومن الخطأ في رأيي أن نقول مثل هذه التصريحات الطنانة عما نفهمه وما لا نفهمه، مع تجاهل الكم الهائل من الأبحاث حول هذا الموضوع، حتى لو كان على الورق فقط في الوقت الحالي.

  59. لا شئ. وليس من المفترض أن نفعل أي شيء حيال ذلك أيضًا. ويكفي أن نبين أنه من الممكن بناء نموذج حاسوبي للجاذبية لساجيت أو حتى منشأة معملية.

    وهذا من شأنه أن يظهر أن النظرية تعمل، مما يسعد يودا كثيرًا. فهل هذا ما يحدث على أرض الواقع؟ على ما يبدو لا، والعلاقات، وما إلى ذلك.

    ماذا يوجد في واشنطن؟ لا أخرج من السرير عند حدوث اهتزازات أقل من 5.

    مساء الخير.

  60. إسرائيل
    يشرح ليساج الجاذبية بافتراض الزخم والمثابرة. إنه يحتاج إلى اصطدامات بلاستيكية لكنه لا يعرف كيف يفسر طبيعتها. فهو يتطلب بحرًا من الجزيئات لكنه لا يعطي تفسيرًا لأصلها. وهو لا يشرح كيف لا يتم تحفيز الأجسام عن طريق إبطاء الجسيمات. ولم يشرح ما هو أصل c والظواهر الأخرى للنظرية النسبية.
    وهو لا يشرح لماذا تقتصر الجاذبية على c، ومن ناحية أخرى، يتنبأ بـ "موجات الجاذبية" من كل مستعر أعظم. وهو لا يشرح كيف أن هناك جسيمات ليس لها كتلة ولكن لها زخم.

    فهل يعطي تفسيرا مبسطا للجاذبية؟ ماذا سنفعل به؟

  61. لماذا لا تلقي نظرة على ليساج؟

    ولم تتطرق بعد إلى ما كتبته:

    "من الممكن بناء نموذج في المختبر حيث سنحصل على أنظمة مشابهة لنظامنا الشمسي، ذات جاذبية يمكننا توجيه قوتها كما نشاء"

    توافق أم لا؟

  62. المعجزات

    دعونا نتفق على أنه لا يوجد تفسير للجاذبية. هل هذا هو السبب الذي يجعلنا نصر على تفسير أقل جودة مما لدينا؟

    ربما كنت تقصد ما ليس لدينا 🙂

    فهل توافق على البيان الذي أحضرته باللغة الإنجليزية؟ لقد أرسلته إلى معهد ماساتشوستس للتكنولوجيا للتأكيد فقط للتأكد. دعونا نأمل أن يستجيبوا قريبا.

    لكن الدفع يفسر مصدر الجاذبية، بدون ذكاء وبدون ذكاء. دعونا نمنحها الاحترام الذي تستحقه باعتبارها النظرية الوحيدة التي تفعل ذلك.

  63. إسرائيل
    أنت تعرف أنني أعرف ذلك….
    في فهمي، تشرح النظرية النسبية العامة الجاذبية بالضبط. الكتلة تشوه الفضاء. الآن سوف تسأل "ولكن - ما هو سبب تشوه الدمسة للمساحة"، وهذا سؤال صحيح. وبعد أن يجيبوا على كل سؤال من هذه الأسئلة، سيأتي سؤال آخر، وسؤال آخر.
    وبالمناسبة، فإن النظرية النسبية (الخاصة) تفسر المغناطيسية، لكنها لا تفسر الجذب الكهربائي (على حد علمي).
    بالطبع هذه ليست القصة بأكملها، لأنها لا تتناسب مع نظرية الكم.

    لكن دعونا نتفق على أنه لا يوجد تفسير للجاذبية. فهل هذا هو السبب الذي يجعلنا نصر على تفسير أقل جودة مما لدينا؟

  64. المعجزات

    لا تتحرك ساعة GPS بشكل أسرع - فالساعات في إسرائيل تتحرك بشكل أبطأ بسبب قربها من جسم ضخم.

    النظرية النسبية لا تفسر سبب الجاذبية، بل تصفها وتحددها كميا. لقد فعل نيوتن ذلك أيضًا. لا أعتقد - جيجال.

    ماذا عن الجملة التالية:

    لا يعرف العلماء سبب الظاهرة الأكثر وضوحًا في الطبيعة: الجاذبية.

  65. إسرائيل
    تعمل ساعة القمر الصناعي GPS بشكل أسرع من الساعة الموجودة على الأرض. وهذه ظاهرة لا يستهان بها، حوالي 45 ميكروثانية في اليوم.

    تفسير النسبية للجاذبية هو انحناء الفضاء بسبب الاختراعات الجماعية، بالإضافة إلى الحركة في الزمكان. تخيل أن الأرض تتضخم بمعدل متزايد - ما ستشعر به يشبه الجاذبية تمامًا.
    أعلم أن هذا تفسير بعيد عن الواقع، لكنه يرضي حدسي. وفي كل الأحوال فهو أقرب إلى الواقع برأيي من الجزيئات الخيالية التي تتحرك بأي سرعة ممكنة وتتناقض مع كل قوانين الفيزياء.

    لمطابقة هذه الجسيمات مع الملاحظات، ببساطة …. تكييفها مع الملاحظات. وكل هذا من أجل ماذا؟ لأننا لم نكتشف جسيمات المادة المظلمة بعد.
    وعندما تظهر ملاحظة تتناقض مع أساس فكرة الجسيمات يتم تجاهلها.

  66. يودا

    من أجل لا شيء.. حقيقة أن الدفع يمكن أن ينتج الجاذبية نظريًا، لا يعني أن هذا أيضًا ما يحدث بالفعل. حتى الحكومة التي لا تحتوي على متطرفين يمكن أن تكون مستقرة نظريًا.

    أعتقد أننا نستمتع.

    أعتقد أنه لا أحد هنا باستثناء يهودا يدعي أن الدفع هو بالفعل سبب الجاذبية. السؤال كما أفهمه هو: هل الدفع يعمل من الناحية النظرية؟

    الجواب هو نعم، طالما أن الجماهير ثابتة. وإلا فإنك ستحصل على احتكاك فاينمان، ولهذا السبب تحدث فاينمان عن ذلك باعتباره عدم أهلية الدفع بشكل كامل.

    إذا تم التغلب على احتكاك فاينمان (اقترحت طريقة)، فإن الدفع يعمل أيضًا مع الكتل الديناميكية.

    وعندما أقول العمل، أعني أنه يمكن بناء نموذج في المختبر حيث نحصل على أنظمة مشابهة لنظامنا الشمسي، ذات جاذبية يمكننا توجيه قوتها كما نشاء. هل هناك غير الشرطة الوطنية الإسرائيلية، تتهربون وتتظاهرون بعدم الفهم ولا تسمعون وعندكم خطأ منطقي ولهذا لم أعد أجيبكم، لا أوافق؟

  67. "بعد كل شيء، تم اختبار النظرية من قبل كل العلماء الجادين تقريبًا في الثلاثمائة عام الماضية، وتبين أنها ناجحة باستثناء المشكلات المعروفة: الاحتكاك، والتسخين، والطموح.."

    لا لا لا. هناك إجماع على أن الجاذبية الميكانيكية هي تفسير أنيق ومثير للاهتمام لأصل الجاذبية النيوتونية (والذي، كما ذكرنا، فشل بسبب عدة نقاط مهمة). لم أسمع قط شخصًا واحدًا يدعي أنه يمكن إعادة إنتاج النتائج النسبية باستخدامه، وهو ما نعرفه بالتأكيد أنه صحيح. بمعنى آخر، هذه نظرية الجاذبية لطيفة جدًا، لكنها تفشل في إعادة إنتاج الجاذبية كما فهمناها قبل 350 عامًا، ناهيك عن المعرفة الحديثة. إذا كان لدى أي شخص طريقة لإعادة إنتاج معادلات أينشتاين الميدانية بمساعدة الجاذبية الميكانيكية (وهي ليست كمية، لأن هذا ما نفعله في نظرية الأوتار كما شرحت من قبل)، فأنا أحب أن أسمعها - حظًا سعيدًا.

    شموليك - من المعتاد بالفعل قياس كتل الجسيمات الأولية بالإلكترون فولت، ولكن لا يوجد سبب محدد للقيام بذلك. من الممكن أيضًا بالجرام، وهو أمر غير مريح لأن كل شخص سيكون لديه كتلة تبدأ بالكثير والكثير من الأصفار. وفيما يتعلق بالكتلة، لا. هناك كمية تسمى كتلة بلانك وهي ذات أهمية كبيرة في الفيزياء، لكنها لا تلعب دورا مماثلا لطول بلانك في نظريات الجاذبية (الطول الذي بدأت منه النظرية الكلاسيكية لم يعد قادرا على وصف الواقع بأمانة لأن التأثيرات الكمومية تصبح مقارنة بقوى الجاذبية). لا يوجد أي عائق أمام أن تكون كتلة الجسيم أصغر من كتلة بلانك. بعد كل شيء، نحن جميعًا نعرف الجسيمات الكمومية ذات الكتلة 0 مثل الفوتون والغلوون والجرافيتون. في الواقع، جميع الجسيمات الأولية لها كتلة أقل من كتلة بلانك.

  68. إسرائيل
    إذا كانت الكتلة نتيجة اصطدام الجسيمات، فليس من المنطقي القول أن الجسيمات لها كتلة. إذا كانت الجسيمات لها كتلة فإن هذه الكتلة تزداد بسرعات عالية، إلا إذا رفضت النظرية النسبية تمامًا.
    لا يمكنك حل مشكلة الجاذبية بالقول "الجسيمات لها كتلة وهذا كل شيء".
    سرعة الجسيمات وثيقة الصلة بتفسير الجاذبية. لا أفهم، اعتقدت أن لدينا تفسيرًا حقيقيًا لظاهرة ملحوظة. حسب ما أنتم أمة فهو مجرد التلويح بالأيدي. يتطلب الأمر الكثير من الوقاحة لرفض كل العلوم الحديثة بهذه الطريقة.

    الوقت يطول ويقصر، راجع أي كتاب عن نظام تحديد المواقع. إذا كنت لا تقبل فرضيات النظرية النسبية، فعليك أن تعطي تفسيرا آخر لهذه الظواهر.

    لم أتحدث أبدًا عن نموذج لاسيج، لأنه تفسير أسوأ بكثير من النظرية النسبية. "الجاذبية الدافعة" لا تختلف من حيث المبدأ عن كون بطليموس - معلقًا معلقًا معلقًا. حتى لو لم يكن لدى نموذج LaSage مشكلة في الاحتكاك، فهذا لا يجعله صحيحًا. لنخترع أخطبوطات فضائية تسحب الأجسام إليها، فتضعف أيديها عند المسافات البعيدة. هل تريد حقًا العودة إلى الأساطير اليونانية لفهم العالم؟

    أساس النظرية - "الكون البسيط" - يفتقر إلى أي سيطرة على الواقع. ألم نمر بهذه المرحلة قبل 120 سنة؟

  69. المعجزات

    قطعاً! والآن اشرح أهمية الأسئلة التالية:

    "كيف تمتلك جزيئاتك كتلة؟" - غير ذي صلة. سوف تحصل على دفع الجاذبية حتى لو كانت الجزيئات عبارة عن دمى شوكولاتة صغيرة وتذيب كتل العسل التي تحاول الدببة تناولها.

    "ماذا لو قصر الزمان وطال" - الزمن يطول ولا يقصر. الدفع لا يدعي ولا يفترض أن يعالج هذه المشكلة، رغم أن له تفسيرا بديلا.

    "ما هي سرعة الجسيمات؟" لا علاقة لها بالموضوع، طالما أنها عالية بما يكفي لإحداث الجاذبية.

    ومازلت لم تجب على السؤال:

    "أنت لا تعتقد أنه من الممكن بناء نموذج في المختبر من شأنه أن ينتج جاذبية لازاج بمواد بسيطة ولكن مع مشكلة واحدة رئيسية وهي الاحتكاك؟ وإذا كانت الأجسام التي تحاكي الكتل في النموذج ليست في حالة حركة فسنحصل على جاذبية يمكننا توجيه قوتها كما نشاء؟

    يقولون لك أن تنظم الهجوم على مطار الأقصر، وتبدأ في التحقيق: ما هي القنابل المصنوعة؟ (غير ذي صلة طالما أنهم يفعلون ما يفترض بهم القيام به، ضرر). كيف ستعرف إذا كان نطاق الطائرات يأتي من الحضر إلى الأقصر؟ (كنا هناك بالفعل في 67 و100 مرة أخرى بعد ذلك). كيف يعمل المصعد على الأجنحة (يعمل، يعمل، ما علاقته بالمهمة؟)

    إذا كنت لا تعتقد أن نموذج ليساج ينتج الجاذبية، فلماذا يدعي جميع العلماء أنه يفعل ذلك؟

  70. المعجزات

    تظهر معظم أسئلتك حول خواص جسيمات اللاساج ويتم الرد عليها جزئيًا على الأقل في مقالة اللاساج:

    https://en.wikipedia.org/wiki/Le_Sage%27s_theory_of_gravitation#Predictions_and_criticism

    ففي نهاية المطاف، تم اختبار النظرية من قبل كل العلماء الجادين تقريبًا في الثلاثمائة عام الماضية، وتبين أنها فعالة باستثناء المشكلات المعروفة: الاحتكاك، والتسخين، والطموح...

    ألا تعتقد أنه من الممكن بناء نموذج في المختبر ينتج جاذبية لازاج بمواد بسيطة ولكن بمشكلة واحدة رئيسية وهي الاحتكاك؟ وإذا كانت الأجسام التي تحاكي الكتل في النموذج ليست في حالة حركة، فسنحصل على جاذبية يمكننا توجيه قوتها كما نشاء؟

    إذًا ما الهدف من كل هذا التحقيق الذي يستغرق ساعتين ونصف الساعة، وما هي الجسيمات التي تتكون منها، وما هي الجسيمات التي تتكون منها الجسيمات، وما هي خصائصها، وما هي خططها، كما لو أن يودا المسكين يجب أن يعرف ..

  71. يهودا
    بالمناسبة - يُعتقد أن كتلة النيوترينو تبلغ حوالي 37-10 جرامًا، فكيف يتناسب ذلك مع ما تقوله؟ يجب أن نرى تصادمات بينها وبين جسيماتك، أليس كذلك؟

  72. المعجزات
    بالنسبة لك ولزوجتك، سأقوم بإعداد، كما طلبت، شريحة لحم بدون دهون، لذيذة ومتبلة... من كتلة داكنة، ههههههه، حسنًا، حسنًا، سوف نتفق.
    وفيما يتعلق بردك، فأنا لا أعاير، فقد أسست فكرة دفع الجاذبية وأدركت أن هناك طريقًا حرًا متوسطًا ومن هنا يأتي كل شيء.
    وبالنسبة لإسرائيل، إذا كان r صغيرًا جدًا بالنسبة لـ a، فإن r مقسومًا على a يميل إلى الصفر وكل الجزء الآخر يميل إلى واحد وتأخذ الصيغة شكل نيوتن وتصل إلى نقطة المخلص (أو الجاذبية) ثم نحن لديهم الجاذبية على مسافات صغيرة. في الغاز في الظروف العادية لا يوجد تجاذب لأن متوسط ​​المسار الحر لجزيئات الغاز صغير بالنسبة للمسافة بين الأجسام. إذا خففنا الغاز، فإن المسار الحر سيزداد، وكذلك الجاذبية.
    أنت لم تفكر في ذلك.
    لم يفكر Le Sage أيضًا.
    ولهذا السبب فهو "الكون البسيط" وليس "نظرية لا سيج"، وليس "دفع الجاذبية" من أجل لا شيء.
    إلى شموليك
    لدي فكرة، ربما يمكنك ترجمة الجزء ذي الصلة إلى التعليق التالي؟ سأعود وأرى ما يدور حوله الأمر.
    سنة جيدة
    يهودا
    http://yekumpashut.freevar.com/

  73. يهودا،
    يقول ذلك صراحة في الدقيقة 26:58. كسر من قبل؟ ردك؟
    الكتلة التي حسبتها، هل هذه هي الكتلة الساكنة؟

  74. يهودا

    إذا، كما تقول، يساوي متوسط ​​المسار الحر للجسيم الذي يبلغ حجمه حوالي 1.55 سنة ضوئية، فكيف نحصل على الجاذبية على مسافات قصيرة، مثلًا مترًا واحدًا؟

    في اللاساج الأصلي، لا يوجد تصادمات بين الجزيئات، وإلا سنحصل على غاز (ولن يرتفع!)، والغاز كما نعلم ليس له جاذبية كما يمكننا إثبات ذلك إذا قمنا بتوزيع ورقتين أمام بعضهما البعض. .

  75. يهودا
    أنا أحب الدهون، لكن زوجتي لا تسمح لي...
    على أية حال - أنت لم تجب على أسئلتي. ما أفهمه هو أن كل ما تفعله هو معايرة الأرقام لمطابقة صيغك مع الواقع. "2" في معادلة نيوتن/آينشتاين له معنى مستمد من افتراضات النموذج.

  76. إلى شموليك
    لقد ذهبت إلى الرابط الذي أرسلتني إليه. لغتي الإنجليزية ليست جيدة، لكني مازلت أفهم شيئًا ما:
    و. على الرغم من مقطوعة باخ اللطيفة، إلا أنني لا أعتقد أن الجمال هو "الدليل" على صحة النظريات. على الرغم من أن مثال الانحراف في القطعة كان صادما. القياسات فقط هي التي ستحدد صحة النظرية.
    ب. لم أكن مقتنعًا بأن القوة الثانية لـ R تنبع من النظرية النسبية. وذكر ستيفن أيضًا أن هذا لا يتبع على وجه اليقين كلمات نيوتن.
    ثالث. إلى سؤالك الصعب عن طبيعة جزيئاتي التي تدفع الجاذبية، الحقيقة. لا أعرف إذا كان ينبغي علي أن أعطيها خصائص منهجية أم كمومية، وأحتفظ بالحق في القيام بذلك لاحقًا، في اللحظة التي أعاملها فيها بأحاسيس نيوتنية. مثل كرات البلياردو الصغيرة كما حددتها. إذا كان هناك قيد في المستقبل سأغيره. لا أعرف أيضًا ما إذا كان عليّ إعطاء خصائص نظرية خارجية لأجزاء من نظريتي. هل يجب علي إجراء حسابات نسبية معهم، لا أعرف. ربما يكون هناك تهرب هنا ولكني لا أعرف حقًا.
    للمعجزات
    عندما نعد فإن الهيدروجين يحتوي بالفعل على بروتون واحد ولدي عشرة أصابع وحتى لو حصلت على 9.8 أصابع في الحساب فسوف أكمل إلى 10. ولكن ما هي كتلة كل إصبع أو كل بروتون. سيكون هناك دائما عدم اليقين. ما هي سرعة الجسم هناك بالتأكيد عدم اليقين. أخبرني أحد الأمثلة من الأكاديمية أن الأمر نفسه ينطبق حتى مع الدوران الذي لا يقبل إلا قيمًا معينة، وإذا جاءت العملية الحسابية بتدوير 1.99، فقد كان من الواضح له أنها كانت دورة. دوران 2. (وربما يمكن أن يكون هناك دوران 1.99؟؟) ليس هناك شك في أنني بحاجة إلى ترقية التعريف حتى تتمكن أيضًا من تلقي الأشياء بيقين مطلق. سوف نفكر في العطلة.
    أعتقد أن سرعة الضوء كانت مختلفة في الماضي، وليس أنا وحدي، بل هناك العديد من الأشخاص الذين يؤمنون بتغيرها. دعونا لا ننسى أن سرعة الضوء الأكبر في الماضي كانت ستغني عن الحاجة إلى كون تضخمي.
    بالنسبة لجسيماتي، هذه هي الكتلة الباقية، لكن حتى في مدونتي هناك تحفظ حول الحجم المذكور أعلاه وطريقة حسابه، خاصة من حيث النسبية المطلوبة أم لا. مسموح لي؟.
    لدي معضلة مع الإسناد. لا أعرف إذا كان علي أن أضعها في الداخل. في بعض الأحيان يمكنني حل مشاكلي النظرية التي تم حلها بواسطة النظرية النسبية. مثال. لنأخذ عملية الجمع (e^(-r/a)) التي أضربها في صيغة نيوتن لأجعلها صيغة الكون البسيطة. إذا اعتبرت (لتذكيرك أن أ يساوي متوسط ​​المسار الحر للجسيم) حجم 1.55 سنة ضوئية، أي حوالي 98,000 وحدة فلكية، فإن هذا من شأنه أن يحل مشكلة المبادرة لكوكب عطارد، دون استهلاك النسبية. فماذا علي أن أفعل الآن؟ أتجاهل ذلك؟ أو ربما تحاول الاستمرار في تدمير كل فكرة تافهة نسبية. لدي شعور بأنني أعالج بنية دقيقة ومن الصعب علي أن أقرر كيفية تنفيذ الخطوة التالية. خاصة عندما تكون هناك ذئاب جائعة تتطلع إلى الاستيلاء على بعض المكدس.
    هذا كل شيء، سأعد لي القهوة ومجرد سؤال للسجل، لا تستخلص استنتاجات خارقة من هذا، ولكن هل تحب شرائح اللحم مع أو بدون دهون ()
    مساء الخير وكل عام وأنتم بخير وأرجو الرد بلطف
    يهودا
    http://yekumpashut.freevar.com/

  77. עברית
    المعجزات
    من السيء للغاية أن نتكلم عن الخلافات مع يهودا. اذهب للقيام بأشياء أكثر إنتاجية. إن الشعور يزداد قوة بداخلي، وربما بداخلك أيضًا، بأن معرفة يهودا وفهمه للفيزياء، في أحسن الأحوال، فضفاضة.

  78. يهودا
    في العلم لا يمكن أن تكون دقيقة؟ ما عدد البروتونات الموجودة في نواة ذرة الهيدروجين؟ لنكن علميين - كم رقمًا دقيقًا نعرف أن النتيجة هي 1.0000000؟

    فيما يتعلق بالثابت c - عليك أولاً أن تفهم أن c ليست خاصية للضوء، بل هي خاصية للفضاء نفسه. بسبب افتراضات النسبية الخاصة، فإن الجسيمات عديمة الكتلة ستتحرك دائمًا بسرعة c. كيف يشتق هذا بالضبط من نظريتك - سأكون سعيدًا بالحصول على شرح موجز. بالإضافة إلى ذلك، يرتبط الاستقرار بمرور الوقت بالروابط داخل الجزيئات، وربما كان على الكيمياء نفسها أن تتغير على مر السنين. لدينا صخور عمرها مليارات السنين على الأرض ولا أعتقد أن أحدًا يعتقد أن الكيمياء كانت مختلفة عندما تشكلت.

    يهودا - إذا قمت بتغيير افتراض نظرية فإن النظرية تتغير. إنه منطق بسيط. أعتقد أن عدم اختراع المادة المظلمة ليس عذرا كافيا لإلغاء قياسات حجم الكون، وعمر الكون، وبشكل عام الانفجار الكبير كله. طنانة بعض الشيء، أليس كذلك؟

    وفيما يتعلق بالمثال الذي قدمته عن تسارع الجاذبية، سنلاحظ: الجسم المرصود حديثًا على مسافة 8.7 سنة ضوئية، والجسم المسبب للتسارع يقع على مسافة 3.7 مليار سنة ضوئية. لذلك - شريحة لحم متوسطة من فضلك.

    لقد كتبت أن جزيئاتك لها كتلة. من الصعب بعض الشيء بالنسبة لي أن أفهم كيف يكون لديهم كتلة، إذا كانوا هم من خلقوا الكتلة. أود أن أعرف مدى سرعة تحركهم.

    لم تشرح لي بعد كيف تؤثر السرعة والجاذبية على الوقت.

    وسؤال آخر - كيف يمكن أن يتحرك فوتون من موجات الراديو الطويلة وفوتون من الأشعة السينية بنفس السرعة بالضبط؟

  79. يهودا،
    انا لا افهم. ما الذي حسبته هو كتلتها الباقية (أو بالنسبة لك الكتلة لا تعتمد على السرعة)؟ هل هي حقًا كرات بلياردو صغيرة جدًا (إذا كان ذلك ممكنًا) أم أنها جسيمات كمومية أولية؟

    ستيفن واينبرغ
    يبدأ القسم الصامت في الدقيقة 18:20 وبشكل ملموس من الساعة 22:20 حتى الساعة 27:00 تقريبًا
    https://www.youtube.com/watch?v=x9Jqgxh6D2s

  80. إلى شموليك
    ليس من الضروري على الإطلاق أن يكون الجهاز الحساس للناترينو حساسًا أيضًا حتى للجزيئات الأصغر من P.G.
    وبالإضافة إلى ذلك، يمكن وزن الجزيئات أيضًا بالطن أو الإلكترون فولت إذا رغبت في ذلك. كما تريد. فضلت الجرام.
    بعد إذنك، لا أستطيع العثور على موقع YouTube لستيفن واينبرغ. أرسل لي الرابط مرة أخرى. من فضلك.
    يهودا

  81. ألبينتيزو,
    لا تعتقد للحظة أنني أحاول جرّك إلى مناقشة، لكن لدي سؤالين حول شيء كتبه يهودا وأود الإجابة عليه.
    كتب: "وفيما يتعلق بحجم الجاذبية التي تدفع الجسيم، قمت ذات مرة بإجراء عملية حسابية وحصلت على أن حجم الجسيم الخاص بي يبلغ 6.3 في 10 أس سالب 37 جرامًا، وهو أصغر بكثير من الحجم المقدر للنيوترينو. لكن الكمية أكبر وهي التي تؤثر".

    هل يتحدث أحد حتى عن الجسيمات الأولية بالجرام، هل لها معنى بالسرعات النسبية؟ لا يتم قياس الجسيمات الأولية دائمًا بـ (G)EV (آسف للعبرية، لا أريد حظرها)
    سؤال آخر: هل هناك حد أدنى للكتلة؟ أي هل هناك حاجز أدنى يستيقظ بلانك تحته ويقول حتى الآن؟

  82. يهودا،
    في هذه الأثناء، لم يتم إصدار الرد الذي يحمل اسمك. وسوف أقوم بالتحديث عندما يتم إصداره
    لم افهم. من الواضح، وفقًا لادعائك، أن التأثير هو *الجاذبية* ويحدث بوسائل ميكانيكية عادية: الاصطدام، مثل كرات البلياردو. يتم إنشاء النيوترينوات في الشمس بكميات هائلة، ولكن لأنها نادرا ما تتفاعل مع المادة، فمن الصعب جدا اكتشافها، ولكن وفرة الجسيمات التي تنتج الجاذبية عن طريق الاصطدام، مطلوب حقا أن تظهر في أجهزة الكشف لدينا. هناك العديد من جسيمات يهوذا الأكثر قتامة من النيوترينوات، وتأثيرها أكثر دراماتيكية، وبالتالي، تتوقع ظهورها في أجهزة الكشف لدينا. هذا لا يحدث.
    إذن لإنتاج الجاذبية عن طريق الاصطدامات في الفضاء نعم، لكن أن نطلب منها الاصطدام بأجهزة كشف في مناجم تحت الأرض محمية من الإشعاع الكوني أليس كذلك؟

    لقد كتبت إلى نسيم "... نحن نختلف حول ما إذا كان أينشتاين القدير قد غيّر الأس من 2 تقريبًا إلى 2 بالضبط"
    لا أعتقد أنك تكلفت نفسك عناء مشاهدة موقع YouTube الذي أنشأه ستيفن واينبرغ خصيصًا لك. يقول لا، بشكل قاطع ومطلق. في ظل افتراضات النموذج لا توجد طريقة للتغيير من 2 بالضبط إلى حوالي 2. ولا علاقة لذلك بعدم اليقين،
    ولكن أن تذهب إلى حيث يقودك قلم الرصاص بعد أن يتم وضع افتراضات النموذج. قم بتغيير الافتراضات، وستحصل على نظرية مختلفة، ولكن سيتعين عليها إعادة إنتاج كل نجاحات النسبية العامة. هل تريد مني أن أعطي رابط يوتيوب مرة أخرى؟

  83. للمعجزات
    كنت أستعد لتناول وجبة الإفطار ولكني أرى أنني سأضطر إلى تناول الغداء على الفور. ولكن بعد رد الفعل الحالي عليك، سأذهب لتناول الطعام.
    إذن كلانا متفق على أنه قبل أن تكون قوة نيوتن الثانية موضع تساؤل. نحن نختلف حول ما إذا كان أينشتاين القدير قد غيّر القوة من 2 تقريبًا إلى 2 تمامًا. باختصار أقول لك أنه في العلم لا يوجد ولا يمكن أن يكون بالضبط. إن القياسات التي ستحاول التحقق من "الدقة" ستفعل ذلك دائمًا مع قدر معين من عدم اليقين، وأنا لا أتحدث فقط عن مبدأ عدم اليقين لأيزنبرغ. هناك دائمًا عدم يقين في تحديد الحقائق في العلم. نقطة.
    ما أريد أن أفعله الآن، ربما في العطلة القادمة يمكننا القيام به بشكل أفضل، هل من الممكن إضافة افتراض رابع وهو "اضطراب الفضاء" إلى الافتراضات الثلاثة الخاصة بمرور الظاهرة. وفي حالتنا ظاهرة الجاذبية. وبالمناسبة، سبق أن قلت إنني لا أقبل افتراض الثابت C وأعتذر لأينشتاين. كل جيل وC. تغير قدره 1 سم في الثانية سنوياً بسبب تمدد كوننا الثمين، وتبريده. لكننا سنرى ما سنحصل عليه في العطلة، ربما سهولة العطلة ورضاي عن الطعام ستسهل علي استخلاص استنتاجات مصيرية في العلوم. لذلك قلت هنا أن افتراض السرعة الثابتة C غير مقبول بالنسبة لي، ولكن على عكسك لست متأكدًا من أنه سيؤدي إلى انهيار النظرية النسبية. لكن علينا أن نتقبل أن القنبلة الذرية التي تسقطها كوريا الشمالية خلال عام ستنتج طاقة أقل مما تنتجه اليوم لأن سرعة الضوء خلال عام ستكون أصغر. وسننتظر التوضيح بعد العطلة.
    وقد وصلنا إلى القسم الأخير من إجابتك - تبريد الجاذبية الذي تم اكتشافه على مسافة 8.7 مليار سنة ضوئية.
    أولا وقبل كل شيء، تهدأ. لم يتم شراء مجموعتك حتى الآن. هناك قدر كبير من عدم اليقين في فوزك.
    ما تظهره هنا ليس تأكيدًا لتسارع الجاذبية لمسافة مليارات السنين الضوئية التي تبعد مليارات السنين الضوئية. ما المقصود بذلك؟ أنا على استعداد لقبول أنه، على سبيل المثال، النظام الشمسي الذي يبعد مليارات السنين الضوئية سيعمل بنفس الطريقة التي يعمل بها نظامنا الشمسي - ليس لدي دليل على ذلك، بل هو مجرد اتفاق ودي يُعطى في لحظة الضعف والجوع من قبل شخص مستجيب يذكرني بمداخن نفسية لشخص جائع ويضعف عقليتي العلمية في مجال تذوق الطعام. من الواضح أن هذه طريقة غير علمية، وأنا أقول أيضًا إنها سخيفة. ولكن هذا ما هو عليه، وهؤلاء هم أصدقائك. دعنا نعود إلى دورة الجاذبية. ما تراه هناك هو نقطة. الجاذبية أم لا هي مسألة أخرى. إذا كان من المناسب لك إضافة كتلة داكنة مثل يد الملك لجعلها "نفايات الجاذبية"، فسوف تفعل ذلك.
    كفى، لقد انهارت، سأذهب لتناول الطعام.
    يرجى الرد بلطف
    سنة جديدة سعيدة وعشاء عيد الميلاد لطيف.
    يهودا
    http://yekumpashut.freevar.com/

  84. شموليك
    يشرفني أنك استخدمت اسمي في ردك المعلق. أعدك بعدم اتهامك بالسرقة الأدبية.
    لم يتم إنشاء نبتون بواسطة الشمس وحدها، ولكن سوف يغفر لك ذلك أيضًا.
    وإجابتي على سؤالك: "لماذا لا تغسل جزيئاتك هذه الكواشف وأجهزة الكشف التي تستخدم للعثور على المادة المظلمة؟" إن جزيئاتي بالتحديد هي التي لها تأثير رائع على أجهزة الكشف و... تظهر الجاذبية! لا يوجد تفسير عملي آخر للجاذبية. عليك أن نعتقد.
    وفيما يتعلق بحجم جسيم الجاذبية الدافع، قمت ذات مرة بإجراء عملية حسابية وحصلت على أن حجم الجسيم الخاص بي هو 6.3 في 10 أس سالب 37 جرامًا، وهو أصغر بكثير من الحجم المقدر للنيوترينو. لكن الكمية أكبر وهي التي تؤثر.
    تحصل على التشجيع من حقيقة أن موجات الجاذبية لم يتم اكتشافها إلا بعد 111 عامًا، فتعالوا وسأشجعكم أكثر قليلاً، فقد تم اكتشاف قوانين كبلر بعد أكثر من ألف عام من فكر بطليموس في مركزية الأرض، وتم اكتشاف سابدارمش بعد 6000 عام من خلق الأرض. العالم، الذي نحتفل بعيد ميلاده هذا الأسبوع. هل هذا سبب للتشجيع؟، لا أعتقد ذلك، لكني اليوم كبير، ومستعد لمنح محبي الكتلة المظلمة بضع ساعات إضافية لاكتشاف أجزائي الضعيفة أو أيًا كان. نتمنى التوفيق للكاشفات التي يتم بناؤها لهذا الغرض،
    نهاركم سعيد وإجازة سعيدة وكل عام وأنتم بخير.
    يرجى الرد بلطف وابتسم!
    يهودا

  85. يهودا
    أنت تنظر إلى صيغة نيوتن كما لو أن نيوتن خمن أن القوة هي 2. لكن الأمر ليس كذلك. لقد خرج نيوتن عن قوانين كبلر، وهذه الصيغ هي نتيجة قياسات تايكو براهي. إذا كانت هذه هي القصة بأكملها، فقد يكون من الممكن القول أن 2 هو رقم تقريبي. لكن جاء رجل اسمه ألبرت أينشتاين وأظهر للعالم أجمع أن صيغة نيوتن مبنية على 3 افتراضات بسيطة (تكافؤ الأنظمة المرجعية، وثبات c في أي نظام من المحاور، ومبدأ التكافؤ). هذا كل شيء، 3 مبادئ بسيطة.
    لكي تقول أن صيغة نيوتن خاطئة، عليك أن تثبت أن أحد هذه الافتراضات الثلاثة خاطئ. أي واحد (أو أي منها) تعتقد أنه خطأ؟

    بالمناسبة - حالة أخرى لتأكيد نظرية النسبية العامة على مسافات كبيرة - تسارع الجاذبية - على مسافة 8.7 مليار (!) سنة ضوئية.
    أنا أحب شريحة لحم متوسطة.

  86. صباح الخير معجزات
    لقد كتبت أنني كتبت أن النظرية النسبية تم إثباتها عن طريق الاستقراء. معاذ الله أن أفعل هذا الفعل الدنيء! ما كتبته يتعلق بصيغة الجاذبية لنيوتن المشهورة، وبالطبع، ربما كان عليّ التأكيد على ذلك. فيما يلي الاقتباس: - "أجد صعوبة في تصديق أنك لا تزال تصر على أن الصيغة المثبتة بالاستقراء على مجموعة صغيرة ستكون صحيحة أيضًا بالنسبة للكون بأكمله؟" نهاية الاقتباس. وهذا ما كتبته، وسأؤكد عليه مرة أخرى وأشرحه: - تم إثبات صيغة نيوتن للجاذبية بالاستقراء على مجموعة صغيرة - الكواكب في نظامنا الشمسي، ومع كل إيماننا بها منذ ولادتنا، يجب علينا لا استخلاص استنتاجات حول الكون كله منه. هناك دائمًا شكوك وعدم دقة في القياسات، لذا عليك أن تفهم أن الاستنتاج من القياسات ليس مجرد صيغة واحدة بل عدد لا حصر له من الصيغ. على سبيل المثال، ربما قوة R ليست 2 بل 2.0000000001؟ وثابت الجذب العام ليس تمامًا كما هو مكتوب في الصيغة، وربما تكون صيغة نيوتن مع حاصل ضرب يهوذا الصغير (e^(-r/a)) هي الصيغة الصحيحة؟ المزيد والمزيد. وبالإضافة إلى ذلك، فإن كل هذه الصيغ اللانهائية صحيحة وتتوافق مع القياسات التي قمنا بها. ولن نتمكن من إلغاء أي منها حتى نقوم بإجراء سلسلة جديدة من القياسات الأكثر دقة أو لمدى أطول. بالطبع، حتى نفعل ذلك، يجب علينا أن نختار أبسطها، من بين كل هذه الصيغ، وهي صيغة نيوتن. (الاختيار وفقًا لشفرة أوكهام. وهو اختيار ملائم!) شرح أكثر شمولاً في مقال مدونتي.
    بالإضافة إلى ذلك، كتبت في ردك أنني قدمت بنفسي مثالاً على أن معادلة نيوتن صالحة حتى على المسافات الكبيرة. أنت على حق، أنت على حق، أنت على حق، لقد أخطأت، أخطأت، لكن ذلك جاء إلي ربما من متعة الكتابة، أو بسبب تعب ليلة هرتسليا الهادئة في الثالثة صباحًا . كان يجب أن أؤكد أنه إذا كانت معادلة نيوتن للجاذبية صحيحة، فإن سرعة الإفلات ستكون 11.2 كيلومتر في الثانية، لكن إذا كانت معادلة أخرى من العدد اللانهائي من الصحيحات الأخرى صحيحة، فقد نصل إلى نتيجة مختلفة ببضعة سنتيمترات في الثانية الى حد ما.
    لذا يرجى الرد بلطف
    وأن تكون سنة خير لنا ولجميع شعب إسرائيل.
    يهودا
    http://yekumpashut.freevar.com/

  87. يهودا،
    في ردي السابق المعلق، استخدمت اسمك عن طريق الخطأ
    سؤال
    وعلى ما تزعم فإن القمر يدور حول الأرض بسبب الجاذبية الدافعة. أي أن بعض جسيماتك المظلمة تلتصق بالأرض، بما يكفي لإنتاج جاذبية للقمر (إذا فهمت بشكل صحيح).
    النيوترينوات، التي يتم إنتاجها فقط في الشمس، وبالتالي فإن كميتها أصغر بكثير من جسيماتك، تمر عبر CdWA دون أن ترمش. ومع ذلك، فإنها "من حين لآخر" تعلق في الذرة وتنتج وميضًا من الضوء، والذي تلتقطه أجهزة الكشف المصممة خصيصًا لهذا الغرض.
    لماذا لا تغسل جزيئاتك هذه الكواشف والكاشفات المستخدمة للعثور على المادة المظلمة؟

    بالمناسبة، لم يحاولوا العثور على المادة المظلمة لمدة 80 عامًا، أقل من ذلك بكثير. إذا لم أكن مخطئًا، فقد اقترح زويكي (أحد الأشرار الكرويين) شيئًا كهذا وقد عززته فيرا روبين كثيرًا من خلال الملاحظات الفردية للمجرات الفردية، ولكن في الآونة الأخيرة نسبيًا فقط تم تصميم أجهزة الكشف خصيصًا لك. أذكرك أن العثور على موجات الجاذبية استغرق 111 عامًا.

  88. يهودا،
    إذا كنت تريد مني أن أتوقف عن الإزعاج، فقط قل لأنه يبدو لي أنك تقلل بشكل رهيب من السؤال حول موجات الجاذبية.
    ومما قرأته، توقع بوانكاريه أنه لا بد من وجود موجات جاذبية تنتقل بسرعة الضوء منذ عام 1905 بسبب هندسة الزمكان التي قدمتها النسبية الخاصة. وبحسب ما هو مكتوب، انضم أينشتاين إلى الفكرة بعد أن نشر نظرية النسبية العامة، رغم أنه كان متشككًا في البداية (لا يزال في أوروبا) وتوصل إلى نتيجة مفادها أنه لا بد من وجود موجات جاذبية.
    ترى يا لها من معجزة، بعد حوالي 111 سنة، تم اكتشاف هذه الموجات عن طريق اصطدام ثقبين أسودين على مسافة مليار سنة ضوئية من هنا (أو بضع مئات الملايين من السنين الضوئية، لا أتذكر)، الحديث عن الامتحان النظري على مسافة كبيرة

    لذا فإن حقيقة أنك لا تفهم ما هي موجات الجاذبية أو بالكاد تفهم سبب وجود أمواج البحر أمر لطيف ولكنه لا يساعدك تمامًا. كما أوضح لك ألبانتيزو (وأيضًا ستيفن واينبرج إذا كنت تكلف نفسك عناء مشاهدة اليوتيوب الذي أرسلته)، فإن النظرية النسبية مبنية على فكرة مركزية تشتق منها العناصر التي يجب أن توجد والعناصر التي لا يجب أن تكون موجودة. موجات الجاذبية يجب أن تكون موجودة ويجب ألا تتجاوز سرعة الضوء وهذا بالضبط ما تم اكتشافه. وفقًا لنيوتن، لا يجب أن تكون هناك موجات جاذبية لأن التأثير لحظي، وبما أنك تقترح أساسًا كونًا نيوتونيًا (أنت لا تقترح بأي حال من الأحوال هندسة مختلفة عن نيوتن)، فلا يجب أن يكون لديك موجات جاذبية، لكنها موجودة بالفعل. هذا تناقض.

    http://www.science20.com/relativity_and_beyond_it/henri_poincare_predicted_the_existence_of_gravitational_waves_as_early_as_june_5_1905-165539

  89. يهودا،
    هل تتحدث عن الطول الموجي للجرافيتون...!؟ هكذا تبدو..
    وكأن تأثير جاذبية الأرض ينتهي عند مسافة معينة...حسنا...
    ولكن، لماذا تعمل الجاذبية بشكل مختلف في المجرات الأخرى؟ (لا أستطيع أن أفهم أساس كلامك)

    أم أنك تتحدث عن القوة التي تقع بين المجرات؟
    أي في المساحات الفارغة من الفضاء. أنت تدعي أن هذه المساحة الفارغة مليئة بجزيئات PG بدلاً من الطاقة/المادة المظلمة، أليس كذلك؟

  90. يهودا
    النسبية أثبتت بالاستقراء؟ ماذا تشرب هناك
    لقد أردت إثباتًا على أن النسبية تعمل حتى على المسافات الذهبية. هل لي أن أسأل لماذا تجاهلت ذلك؟
    لقد قدمت بنفسك مثالاً على أن معادلة نيوتن صالحة حتى على المسافات الكبيرة. هل لي أن أسأل لماذا تتجاهل هذا أيضًا؟

  91. المعجزات
    أجد صعوبة في تصديق أنك مازلت تصر على أن الصيغة المثبتة بالاستقراء على مجموعة صغيرة ستكون صحيحة أيضًا بالنسبة للكون بأكمله؟؟ ذهبت إلى ستيماتسكي في هرتسليا وكانت كل الكتب مصنوعة من الورق فهل يمكن أن نستنتج أن كل الكتب في الكون مصنوعة من الورق؟؟ هل ستستنتج أن موربي كتب قوانينه على الورق لأن هذا هو الحال مع ستيماتسكي؟
    أنا لا أفهمك، نحن نهدر الطاقة على شيء واضح.
    وانظر ماذا فعلت بالإضافة إلى ذلك فأنت تقنعني أن الصيغة التي استخدمتها غير صحيحة وفي الواقع هناك صيغة أخرى وهي الصحيحة فيما يتعلق بالسقوط الحر. جميلة وحتى ممتازة. لكن تخيل أنني سأقول لك أنك مخطئ وأن صيغتي صحيحة!صحيح أنني حصلت على 98 كم في الثانية، لكن 86.8 منها مظلمة ولم أرها شفافة، ويجب أن تكون يتم طرحها من النتيجة. ثم خرج 11.2 كم في الثانية، وهو بالضبط ما ينبغي أن يخرج. السرعة المظلمة تشبه كتلتك المظلمة تمامًا. ماذا تقول معجزات؟، ثمانين عامًا تبحث عن دليل على الكتلة المظلمة في سارن، في أمريكا، وربما أيضًا في كوريا الشمالية، ولم تجدها بالضبط، هل تريد بضع ساعات أخرى للبحث؟، ليس لدي مشكلة مع ذلك.
    ليلة سعيدة ونوم جيد
    لقد كنت مخطئًا - سنة جديدة سعيدة
    يرجى الرد بلطف فهو مجرد علم
    يهودا
    http://yekumpashut.freevar.com/

  92. شموليك
    سؤالك:- لماذا توجد موجات الجاذبية؟
    الإجابة:- لا أعلم حقًا سبب وجود موجات الجاذبية، ولا أفهم التفسير النسبي لموجات الجاذبية ولا أعلم إذا كان دفع الجاذبية قد يدعمه أم لا. بالكاد أفهم سبب وجود الأمواج في البحر. أنا أعتذر. لم أكن في ذلك الفصل في المدرسة الثانوية عندما شرحوا عن موجات الجاذبية.
    لذا من فضلك لا تثير موجات والرد بلطف
    يهودا

  93. يهودا
    يعتمد حساب سرعة الهروب على حقيقة أن طاقة الجاذبية عند اللانهاية هي 0، وأن قوتها عند أي مسافة هي -
    ز * م * م / ص ^ 2. نحن نجري التكامل ونحصل على أن الطاقة الكامنة هي -
    ز * م * م / ص-. ومن هذا نستنتج أن سرعة الإفلات (أو - كما قلت - السرعة التي يصل إليها الجسم بسبب سقوطه من مسافة طويلة) هي
    (sqrt (2 * ز * م / ر
    ضع البيانات وستحصل على 11,200 متر في الثانية. غير صيغة نيوتن وستحصل على نتيجة مختلفة. هذا هو فيزياء الصف الحادي عشر.

    ماذا تفعل يا يهوذا؟ تقول "أنت لا تعرف ماذا يحدث على مسافات بعيدة!!" ثم تقول "لكنني أعرف!"
    ولماذا - لأنهم لم يعثروا على ما هي المادة المظلمة حتى يومنا هذا. هذا كل شيء.
    أنت ترفض النسبية ونظرية الكم. ليس لديك أي تفسير لتأثيرات الزمن النسبية - وهي تأثيرات ليست من فضول العلم - فهي أدوات يستخدمها المهندسون. أنت تتجاهل أن هناك بالفعل دليل على صحة النظرية النسبية حتى على مسافات بعيدة. حسب فهمي، فإن تأثير دوبلر موجود أيضًا في المجرات البعيدة. النسبية ليس لها أي تأثير على هذا التأثير؟

  94. المعجزات
    تقول في ردك: "إن أساس هذا الحساب هو أن صيغة نيوتن صالحة لأي مسافة". نهاية الاقتباس.
    جوابي هو أنه لا توجد ولا يمكن أن تكون صيغة ثابتة لأي مسافة لأن ثبات الصيغة لا يمكن أن يتم إلا عن طريق القياسات وكما نعلم لا توجد إمكانية لقياسات لا نهائية. فصحيح أن نيوتن قال إن الجاذبية تعمل وفق صيغته عند أي مسافة. هكذا قال هل هذا صحيح؟؟، قطعا لا!. هل اختبر على أي مسافة؟، لا!، هل يمكن أن يختبر على أي مسافة؟؟، لا أيضًا! لذلك الجملة المقتبسة التي قلتها لا يمكن أن تكون صحيحة! هل هذا مفهوم؟، الأمر بسيط حقًا.
    دعنا نعود إلى المثال الذي قدمته. قل معجزات، لو تأكدت من صيغة الجاذبية في النظام الشمسي على مسافة أقل من ألف سنة ضوئية، هل تعتقد حقا أنه من الممكن أن نستنتج منها الكون كله دون قياس؟؟؟ وحدد نيوتن صيغته بالاستقراء، أي أنه جمع كل القياسات المعروفة في ذلك الوقت حتى كوكب زحل واستمد منها صيغة الجاذبية الخاصة به حتى مسافة حوالي عشر وحدات فلكية من زحل. بمجرد أن رأى أن الصيغة تعمل على جميع الكواكب المعروفة، كانت بالنسبة له الكون المعروف بأكمله، وكان سعيدًا بإعلان أن صيغته صحيحة بالنسبة للكون بأكمله وكانت دقيقة لأن الكون هو الذي كان معروفًا له . ولكن ليس بالنسبة لنا. إن كوننا المعروف أكبر من كون نيوتن بتريليونات المرات، وبدون قياسات لا يمكنك القول بأن الصيغة صحيحة ولو لسنة ضوئية واحدة. فصحيح أنه تم إجراء قياسات، في المجرات، على مسافات بمليارات المرات، والعجيب أن تكذب صيغة نيوتن الشهيرة في المجرة! إنه يتنبأ بالجاذبية بشكل قليل جدًا، وأنت المعجزة تخبرني أن "صيغة نيوتن صالحة لأي مسافة؟؟؟"
    إسمح لي، من فضلك العلم مستاء.
    أتمنى أن تكون النقطة واضحة
    كل عام وأنتم بخير، وأرجو الرد بلطف. إنه مجرد علم.
    يهودا
    http://yekumpashut.freevar.com/

  95. يهودا،
    لماذا توجد موجات الجاذبية؟
    كتب ألبانتيزو: "بعبارة أخرى، الظواهر النسبية التي يستجوبك عنها نيسيم تم شرحها في النسبية على وجه التحديد من خلال التصحيح الذي يكمن وراء التقريب النيوتوني. لديك تصحيح مختلف، وبالتالي لا يمكنك القول بأنك تفسر الظواهر النسبية بنفس طريقة النظرية النسبية.

    وفقًا لنيوتن، لا يوجد شيء اسمه موجات الجاذبية، وإذا فهمت عملك بشكل صحيح، فكل ذلك موجود في العالم النيوتوني.
    تم قياس موجات الجاذبية، بينما معك (مرة أخرى، إذا فهمت بشكل صحيح)، لا ينبغي أن يكون هناك شيء من هذا القبيل (لأنه لا يوجد شيء من هذا القبيل عند نيوتن). أليس هذا تناقضا مع الملاحظات؟
    https://en.m.wikipedia.org/wiki/Gravitational_wave

  96. يهودا
    أنت لم ترد على المثال الخاص بي.
    بالمناسبة - المثال الذي قدمته لسرعة السقوط المحظورة يوضح تمامًا عكس ما ادعيته 🙂 أساس هذا الحساب هو أن صيغة نيوتن صالحة لأي مسافة.

  97. مجهول
    حرف "s" الذي ظهر كان خطأي، لقد رأى أنه خطأ لمرة واحدة.
    بالإضافة إلى ذلك، سألت لماذا تعمل الجاذبية بشكل مختلف على مسافات بعيدة.
    الجواب على هذا السؤال سبق أن بحثه أفضل الفلاسفة (مثلا ديفيد يوم الإنجليزي) والقرار هو أن ما سنفحصه هو الموجود. إن اتخاذ قرار بالذهاب إلى أبعد من ذلك هو مقامرة.
    سأعطيك مثالا:- v=g*t هذه هي معادلة السرعة التي يتم الحصول عليها في السقوط الحر على الأرض، مثلا بعد ثانية واحدة تكون 9.8 متر في الثانية وبعد ثانيتين تكون السرعة 19.6 متر في الثانية . ما رأيك في السرعة بعد 10,000 ثانية؟ السرعة في نظرك يجب أن تكون 98 كم في الثانية. لكن هناك مشكلة في ذلك، لأن سرعة السقوط الحر إلى الأرض لا يمكن أن تزيد عن سرعة الهروب التي تبلغ 11.2 كيلومترًا في الثانية. من هنا عليك أن تفهم أن ما ينجح على المدى القصير لن ينجح بالضرورة على المدى الكبير.
    ومشكلة انتشار الجاذبية في الفضاء هي أنهم تجاهلوا اضطرابًا محتملاً يمكن أن يؤثر على تقدمه في الفضاء مثل التعكر الذي يؤثر على انتشار شدة الضوء في الفضاء. ومن ثم، يجب أن تحتوي صيغة الجاذبية على مصطلح يعبر عن هذا الاضطراب e^(-r/a) حيث e هو الرقم الطبيعي، وr هي المسافة بين الكتل التي تخلق الجاذبية، وa هو متوسط ​​المسار الحر لدافعي الجاذبية الجسيمات (الاصطدام بينها يقلل من تكوين الجاذبية). لذلك، عند المسافات r صغيرة جدًا بالنسبة إلى a، يميل المصطلح e^(-r/a) إلى 1 وضربه في صيغة نيوتن يعطي صيغة نيوتن، من ناحية أخرى، على مسافات r كبيرة جدًا بالنسبة إلى a، يميل الحد e^(-r/a) إلى الصفر وهكذا عن طريق ضربه في صيغة نيوتن للجاذبية.
    حاليا لا أعرف حجمها، أعتقد أنها أكثر من سنة ضوئية.
    وفيما يتعلق باقتراحك لتغيير اسم "الكون البسيط" سأفعل ذلك عندما أغير اسم سابدارمش
    والمعجزات،
    سيكون من الجميل في الواقع أن نجتمع بصحبة العلماء لقضاء أمسية ممتعة بجوار النار، مع بعض الملاحظات الفلكية.
    وإلى إسرائيل
    لا يبدو لي أن أحداً فكر في صيغتي.
    سنة جيدة
    سابدارمش يهودا

  98. بالمناسبة، يا يهودا، هل تفكر في تغيير اسم نظريتك إلى شيء أكثر جاذبية، مثل، على سبيل المثال، اسمح لي باقتراح "نموذج تسونامي للجسيمات". ما رأيك؟ ؟

  99. يهودا
    شكرا.
    وفي الوقت نفسه، لا أستطيع أن أفهم لماذا تعمل الجاذبية بشكل مختلف على مسافات بعيدة. ففي النهاية، هذه هي نفس المواد، وهذه هي نفس الطاقات، وهذه هي نفس الظروف الفيزيائية الموجودة في الفضاء القريب.

  100. أيها الإسرائيليون، قلوا ما شئتم عن نظرية يهودا (أو.س، كما قرر أن يطلق على نفسه الآن.. لن أتفاجأ إذا قرر فجأة الظهور بقبعة النعش، وسترة جلدية، ولحية عصرية... : ))))
    لكن فكرته رائعة... ☺

  101. إلى ألبينزو
    شكرا على اجابتك. يبدو أنه ليس لدي خيار سوى أن أتعلم كيف أن النسبية كما نعرفها تعطي مجالًا تقريبيًا ضعيفًا لمعادلة نيوتن ومن ثم استخلاص استنتاجات حول صيغتي. سأدخل في الموضوع وأجيب. سيكون لدي شيء لأفعله في العطلة.
    شكرا مرة أخرى ألبانزو وعطلة سعيدة.
    يهودا

  102. يهودا،

    النظرية النسبية كما نعرفها تعطي في تقريب المجال الضعيف معادلة نيوتن. يمكنك أن ترى هذا بنفسك في أي كتاب علاقات. إذا كان نموذجك يقترب من معادلة مختلفة، فهو ليس نفس نموذج النسبية. من الممكن أن تعطي النماذج المختلفة نفس التقريب. على سبيل المثال، نظرية الأوتار - هذا نموذج مختلف للجاذبية (مختلف تمامًا، إنه كمومي على الإطلاق)، لكنه ينتج تقريبًا معادلات أينشتاين (أي النظرية النسبية العامة)، وهي تعطي تقريبًا معادلات نيوتن.
    في حالة وجود نموذجين مختلفين يعطيان نفس التقريب، يمكن القول أن جميع الظواهر (في المنطقة التي يكون فيها التقريب صحيحا) ستكون نفسها، على افتراض أنها محلية (لا تعتمد أيضا على أماكن أخرى حيث يكون التقريب صحيحا). قد لا يكون التقريب صالحًا). لكن معك هناك شيء مختلف - معادلة الحركة في المجال الضعيف *مختلفة*. إنها تعطي نفس التقريب مثل النسبية العامة فقط عند الحافة، ولكن جميع الظواهر النسبية هي بالضبط الظواهر التي لم يتم تفسيرها بهذا التقريب (إذا تم تفسيرها بهذا التقريب، فإنها ستكون جزءًا من الميكانيكا النيوتونية). بمعنى آخر، الظواهر النسبية التي يسألك عنها نيسيم مفسرة في النسبية على وجه التحديد من خلال التصحيح الذي يقع خارج التقريب النيوتوني. لديك تصحيح مختلف، لذلك لا يمكنك القول أنك تفسر الظواهر النسبية بنفس طريقة النظرية النسبية.

    في الختام - النظرية النسبية تشرح بعض الظواهر وتعطي أيضًا صورة تقريبية لمعادلة نيوتن. لديك نموذج آخر، والذي يقارب أيضًا معادلة نيوتن. وهذا لا يعني أنه يوضح الأشياء التي تشرحها الرعاية. يجب عليك إظهار ذلك بشكل صريح، أو مواجهة نموذجك الذي يتعارض مع الملاحظات.

  103. بدأت أشعر بالتعب قليلاً، خاصة من الاضطرار إلى كتابة التفاصيل مرارًا وتكرارًا. أولئك الذين ما زالوا مهتمين يمكنهم رؤية التوضيحات في نفس المقالة منذ خمس سنوات:

    https://www.hayadan.org.il/astronomers-reach-new-frontiers-of-dark-matter-130112/comment-page-17/#comment-327356

    خروج منه:
    "في الخلاصة، كان الغرض من هذه المناقشة برمتها هو الوصول إلى نقطة مفادها أنه من الممكن أن تكون هناك جسيمات مشابهة لجسيمات لازاج تتمتع بالخاصية التالية: عندما تصطدم بالمادة، فإنها تنقل إليها كمية الحركة دون إضاعتها على شكل حرارة، ولكنهم فوق سرعة معينة يمرون في المادة دون أن يؤثروا عليها أو عليها.

    اذهب إلى الطريق السريع.

    Autostrada هو نموذج Lesage في بعد واحد من الطول. فبدلاً من الجسيمات الموجودة في جميع الاتجاهات وبجميع السرعات، نركز فقط على تلك الجسيمات التي تتحرك على طول خط مستقيم معين، في كلا الاتجاهين وبجميع السرعات. سنركز على سرعات تصل إلى 500 م/ث.

    يمكننا مقارنته بطريق سريع يضم 100 حارة، 50 في كل اتجاه، شمالًا وجنوبًا. تتحرك السيارة بسرعة 10 م/ث، 20 م/ث، …. ما يصل إلى 500 م / ث. لغرض المناقشة، نقوم بتركيز السيارات التي لها نفس السرعة على نفس المسار. لذلك، في المسار رقم 1، ستكون جميع السيارات بسرعة 10 م/ث، ثم 20 م/ث حتى الخط 50 حيث توجد سيارات بسرعة 500 م/ث. نفس الشيء في الاتجاه المعاكس.

    1. إذا رسمنا خطًا وهميًا عبر الطريق السريع، فإن الزخم الإجمالي للسيارات التي تعبر الخط يبدو 0 لأن السيارات من كلا الاتجاهين تقابل بعضها البعض.

    2. إذا استخدمنا بدلاً من الخط كيانًا، على سبيل المثال مجال القوة الكهربائية، أو من أجل التوضيح شراعًا أو مادة بلاستيكية خاصة يمكن للسيارات المرور من خلالها دون تشويهها، فيبدو أننا حصلنا أيضًا على 0 زخم على البلاستيسين.

    3. لنفترض حسب ما استنتجناه سابقاً أن البلاستيسين حساس فقط للسرعات التي تصل إلى 100 م/ث. السيارات التي تمر بالبلاستيك بسرعة أكبر من 100 م/ث تكون شفافة بالنسبة للبلاستيك وهو شفاف بالنسبة لها. أقل من هذه السرعة النسبية، ستؤثر السيارات بقوة على البلاستيسين.

    4. دعونا نرى ما يحدث عندما نضع البلاستيسين على الطريق السريع:

    جميع السيارات التي تبلغ سرعتها 110 م/ث فما فوق، في كلا الاتجاهين، تكون شفافة بالنسبة للمركبة

    5. تؤثر السيارات التي تبلغ سرعتها 100 م/ث أو أقل قوة على النبض، ولكنها تقابل بعضها البعض. القوة الإجمالية المطبقة على النبض تساوي 0 وتبقى في مكانها. (يحدث نفس الشيء لأي شراع في الهواء الساكن. فجزيئات الهواء السريعة تقابل بعضها البعض).

    6. ماذا سيحدث إذا وضعنا شراعًا مستويًا أو شراعًا ثانيًا يتحرك بسرعة 10 m/s باتجاه الشمال بالنسبة للطريق السريع؟

    السيارات على الطريق رقم 1 المتجهة شمالًا، والتي كانت تتحرك بسرعة 10 م/ث بالنسبة إلى المستوى A، ستكون سرعتها 0 بالنسبة إلى المستوى B. أولئك الذين تحركوا في الطريق 2 بسرعة 20 م/ث بالنسبة إلى أ، سيتحركون بسرعة 10 م/ث بالنسبة إلى ب، وما إلى ذلك. سيتم رؤية جميع السيارات الموجودة على جميع الطرق في اتجاه الشمال في المستوى B وهي تسير بسرعة أقل بمقدار 10 م/ث مما يراها في المستوى A. من ناحية أخرى، سينظر B إلى السيارات المتجهة جنوبًا على أنها تسير بسرعة أعلى بمقدار 10 m/s من المستوى A الذي سيقيسه.

    ومع ذلك، لاحظ أنه بما أن كل سيارة تبلغ سرعتها 100 م/ث أو أعلى في أي اتجاه تكون في الواقع شفافة للبلاستيك، فإن مجموع القوى على المستوى B هو أيضًا 0. ومن ثم، ستبقى بنفس السرعة البالغة 10 م/ث باتجاه الشمال. إذا عدنا إلى الشراع، حتى بسرعة 10 م/ث لن نشعر بأي رياح، وبنفس المنطق أيضًا بسرعة 200 م/ث، أو 350 م/ث، ولا يهم أي اتجاه. مهما كانت السرعة التي نضع فيها الشراع أو الشراع بالنسبة للطريق السريع، فسوف يظلون عند ذلك، عندما يكونون بالنسبة لهم في نظام "الهواء الراكد".

    سنأخذ قسطًا من الراحة لاستيعاب الأفكار والأسئلة، وسنناقشها لاحقًا."

    وهذا رد مئير عميرام على الفكرة:

    "إسرائيل،
    أنا أتابع. لقد فهمت كيف تغلبت على حجة فاينمان. من العبقرية.
    لكنني لا أحب LS.

    أعتقد أن حجتي تعمل بشكل منطقي، رغم أن هذا لا يعني بالطبع أن هذا ما يحدث في الواقع.

    ليله سعيده للجميع.

  104. شيما إسرائيل,
    لدى Yoda بالفعل براءة اختراع لخدعة "لم أفهمها"، ولا يمكنك استخدامها أيضًا!
    إذن لدينا الآن نسختان من دفع الجاذبية، ونسختان من الأشخاص الذين لا يفهمون العيوب الموجودة فيهم.

    تعال...

  105. يهودا
    النسبية العامة تعزو الجاذبية إلى تأثير الكتلة على الفضاء. يدعي تفسيرك أن الفضاء ليس له خصائص.
    نحن نعلم بالملاحظة أن النظرية النسبية صالحة أيضًا لمسافات شاسعة، حتى حافة الكون المرئي.

  106. إسرائيل
    أنا أعرف بالضبط ما هو الارتجاج الباليستي. لا يمكنك استخلاص الكثير من القياس، وبالتأكيد ليس في حالتنا.
    وحتى لا يكون هناك احتكاك، لا بد من وجود شيئين. الأول هو أن هناك حاجزًا أمام السرعة التي ينتقل بها الزخم. لماذا يوجد مثل هذا الحاجز؟ وهل هو ثابت لكل مادة وفي كل حالة؟
    والثاني هو أنك تحتاج إلى عدد لا حصر له من الجسيمات في كل سرعة. إنه ليس شيئًا ماديًا، وحتى لو كان هناك حل للشيء الأول - في فهمي، فهو يستبعد الفكرة.

  107. للمعجزات
    يمر الوقت بشكل مختلف بسبب تفسير النسبية. الكون البسيط لا يستبعد النظرية النسبية على مسافات صغيرة، تجربة ميكلسون مورلي مقبولة لدى الكون البسيط ولا تتعارض معها، ويفترض احترام النظرية النسبية مكانها.
    سبق أن شرحت لشموليك في الرد 123.
    اسبوع جيد
    يهودا

  108. المعجزات

    عن أي تناقض تتحدث؟ أنا أقول أنه اليوم في المختبر، باستخدام الوسائل الموجودة، يمكنك بناء نظام ينتج عنه كل ما وصفته تقريبًا، أي الجاذبية عديمة الاحتكاك المثالية تقريبًا وربما القصور الذاتي أيضًا.

    هل قرأت المدخل الخاص بالبندول الباليستي؟

  109. نورث كارولاينا آسف، اعتقدت أنك إذا كتبت:

    الحل هو أن الجسيمات الموجودة في نطاق سرعة معين فقط هي التي تتفاعل مع الجسم.

    لذا، إذا ابتعدت عن (أو اقتربت من) جسم ضخم فسوف أعاني من جاذبية أقل (مقارنة بحالة السكون بالنسبة للجسم الضخم). وذلك لأن هناك جسيمات لم تتفاعل مع الجسم الضخم وتفاعلت معي أيضاً، وبالتالي فإن الجسم الضخم لم يحجب كل ما وعد بحجبه.

    لقد كنت تقصد جاذبية أقل، والآن أفهم أنك تقصد احتكاكًا جديدًا.

    ولكن ربما يفهمني شخص آخر بشكل أفضل ويمكنه أن يشرح لي ما هي الاحتكاكات الأخرى التي تتحدث عنها.

    نسيم، يهودا، هل تفهمين ربما ما يعنيه نورث كارولاينا؟

    عيد سعيد.

  110. إسرائيل
    وأوضح ألبينزو ما هي المشكلة هنا، وأنت تتجاهل ما كتبه.
    لهذا السبب سألتك سابقًا عن الخصائص الفيزيائية للجسيمات. كل ميزة يمكنني التفكير فيها تنطوي على تناقض. إذا لم يكن لديهم أي خصائص فيزيائية فكيف تؤثر على المادة العادية؟

  111. إسرائيل،

    لقد أجبت بالفعل على سؤالك.. وهذه هي المرة الأخيرة التي أجيب فيها: لقد قمت بحل مشكلة الاحتكاك ولكنك أدخلت "احتكاكًا" آخر في الباب الخلفي قد يكون بنفس السوء.. بعد كل شيء، أوضحت أنه إذا تحرك شيء بشكل أسرع من "سرعة رد الفعل القصوى" لن تواجه الجاذبية على الإطلاق، وعدم تجربة الجاذبية على الإطلاق ليس شيئًا ضئيلًا تمامًا، أليس كذلك 🙂

    من أجل التغلب على الاحتكاك الذي أدخلته، يجب أن تكون "سرعة التفاعل القصوى" أكبر بعدة مرات من سرعة الضوء (لأن الضوء لن يواجه الجاذبية)، ثم تعود إلى المشكلة المكتوبة في ويكيبيديا في الدفع دخول الجاذبية في قسم سرعة الجاذبية..

    باختصار، أنا أفضل نظرية الجاذبية الدفعية الأصلية لأن..
    و. إنه أبسط
    ب. لا تحتاج إلى جزيئات لا نهائية بكل السرعات الممكنة
    ثالث. هناك احتكاك في كليهما، ولكن على الأقل في الأول يوجد، رايل يفهم الاحتكاك.

    وأخيرًا، هناك 5 جسيمات ذات جاذبية دفعية تتحرك بسرعة 100 كم/ساعة من اليمين إلى اليسار. يتم تمثيل Kdva بالرمز X.

    قبل :سسسسسسسسسسسسسس

    وهذه رسمة لهم بعد أن عبروا الطريق الذي توقف فيه أحدهم.

    بعد دقيقة:سسسسسسسسسسسسسسسسسسسسسسسسسسسسسسسسسسسسسسسسسسسسسسسسسسسسسسسسسسسسسسسس
    بعد دقيقتين:سسسسسسسسسسسسسسسسسسسسسسسسسسسسسسسسسسسسسسسسسسسسسسسسسسسسسسسس

    نشاط ترفيهي لعيد رأس السنة: قم بطلاء الحفرة بألوان مبهجة، وأرسلها إلى النظام واكتب مدى سرعة حركتها. من بين الحلول، سيتم رسم تفاحة وعسل.

  112. المعجزات
    إذا كنت تقصد متوسط ​​السرعة بالنسبة إلى الكتلة - الشبكة - فهي 0، تمامًا كما أن متوسط ​​سرعة جزيئات الهواء في حالة السكون بالنسبة للجسم هو 0.

    ولكن على عكس الهواء، فحتى عندما تتغير سرعة الكتلة، تظل سرعة الجزيئات المؤثرة 0، بينما في الهواء يكون متوسط ​​سرعة الجزيئات بالنسبة للجسم هو سرعة الجسم بالنسبة للهواء (سرعة الرياح).

    هل ترى خيارا آخر؟

  113. إلى ألبينزو
    أولا وقبل كل شيء، شكرا لك على تعليقك.
    هناك فرق كبير بين صيغتي مع إضافة العدد الطبيعي كما وضحت في ردي السابق على شموليك، والتي تم الحصول عليها عن طريق الاستنباط من تطور فكرة دفع الجاذبية، وصيغة نيوتن التي تم الحصول عليها عن طريق الاستقراء وكما قال ديفيد يم، فهو صحيح حيث يتم قياسه. الصيغتان متشابهتان جدًا مع بعضهما البعض إلا إذا ذهبنا بعيدًا،
    لقد قلت بشكل صحيح، وأنا أقتبس، أن النظرية النسبية هي نموذج - مجموعة من الافتراضات المتعلقة بسلوك الجاذبية. يستلزم هذا النموذج مجموعة من المعادلات، والتي في تقريب المجال الضعيف تعطي الصيغة النيوتونية التي أعرفها. الآن يُطرح السؤال "القاتل": هل من الممكن أن يكون هناك نموذج مختلف قليلاً للنظرية النسبية - مجموعة من الافتراضات المتعلقة بسلوك الجاذبية. إطلاق مجموعة من المعادلات والتي في تقريب المجال الضعيف تعطي صيغتي الكونية البسيطة والتي تكاد تكون مطابقة للنيوتونية؟؟
    بالنسبة لي، في حدسي الضعيف، يبدو الأمر كذلك، ولكن أود أن أسمع رأيك المستنير.
    شكرا مرة أخرى، وشكرا لكم على صبركم.
    اسبوع جيد!. مع خالص التقدير
    سابدارمش يهودا
    http://yekumpashut.freevar.com/

  114. يهودا

    هل يمكنك تلسكوب وفيديو النجوم المتغيرة بسرعة مثل CY Aqr؟

    أو عيوب ALGOL التي تحدث كل يومين أو نحو ذلك، الأوقات مذكورة في:

    https://freestarcharts.com/algol-eclipse-dates-times-september-2017

    لدي إمكانية الوصول إلى التلسكوبات عبر الإنترنت ولكنها جميعها في نصف الكرة الجنوبي. تصلني دائمًا رسالة مفادها أن ألغول موجود تحت الأفق.

    شكر

  115. يهودا،

    أنت لا تفهم. لقد شرحت لك ذلك من قبل، وسأحاول أن أشرحه مرة أخرى.

    النسبية ليست صيغة. أنت تتصرف كما لو أن النظرية النسبية هي صيغة وإذا كانت صيغتك التي اخترعتها (سأشرح بعد قليل ما هو المقصود بـ "اخترع") مشابهة، فإنها ستفسر نفس الظواهر التي تفسرها النظرية النسبية. وهذا ببساطة غير صحيح على الإطلاق.

    النظرية النسبية هي نموذج - مجموعة من الافتراضات المتعلقة بسلوك الجاذبية. يستلزم هذا النموذج مجموعة من المعادلات، والتي في تقريب المجال الضعيف تعطي الصيغة النيوتونية المألوفة. الظواهر النسبية مثل الفرق في تدفق الوقت في مجالات الجاذبية المختلفة لا يتم تفسيرها من خلال الصيغة، ولكن من خلال النموذج. إذا توصلت إلى صيغة مختلفة (أي لا تشتقها من نموذج ولكن تقرر ببساطة أنها تصف الجاذبية) ليست مثل معادلات المجال في النسبية العامة، فهي غير متسقة مع النموذج. ولا يهم إذا كان الفرق فقط في المسافات الكبيرة، أو قصيرًا، إذا كان أسيًا أو متعدد الحدود، إذا كان أسودًا أو أبيضًا. بمجرد أن تكون معادلة القوة الخاصة بك مختلفة، فهذا يعني أنها غير مشتقة من النموذج النسبي (لأنها لو كانت مشتقة من النموذج النسبي، فلن تكون مختلفة). ولذلك، فإن جميع الظواهر التي تفسرها النسبية *لا* تفسرها صيغتك. إذا تم تفسير بعض الظواهر (أو جميعها) بأعجوبة بطريقة أو بأخرى من خلال صيغتك، فسيكون عليك عبء الإثبات لإظهار كيفية حدوث ذلك. بمعنى آخر، عليك أن توضح كيف يتدفق الوقت بشكل مختلف على ارتفاعات مختلفة فوق الأرض، ولا يمكنك القول إن السبب في ذلك هو أن صيغتك المبتكرة تشبه معادلة القوى المشتقة من العلاقات عند هذه المسافة، لأنها ببساطة ليست المعادلة. القوى المسببة لهذه الظاهرة، ولكن الدور الذي يلعبه الزمن في القماش الريماني الزائف (والذي هو بالضبط أساس نموذج النسبية).

  116. شموليك
    لقد سألت: "كيف يمكن قبول النظرية النسبية من ناحية وعدم قبول تقريبها الذي ينتج عنه نيوتن" من ناحية أخرى؟
    الجواب: غير دقيق. على مسافات صغيرة تصل إلى بضعة آلاف من الوحدات الفلكية أحصل عليها، وليس ذلك فحسب، بل إن صيغتي أيضًا تشبه صيغة نيوتن وهي تقريبية للنظرية النسبية! المقصود: - أنه عند المسافات الصغيرة تتقارب صيغتي مع صيغة نيوتن. لسوء الحظ، لا يمكن نسخ الصيغ في التعليقات، ولكن سأحاول التوضيح:-
    صيغة الجاذبية للكون البسيط هي صيغة نيوتن للجاذبية مضروبة في المصطلح (e^(-r/a) حيث e هو العدد الطبيعي، وr هي المسافة بين الكتل التي تخلق الجاذبية، وa هو متوسط ​​المسار الحر للكون البسيط. جسيمات جاذبية دافعة (الاصطدام بينهما يقلل من تكوينها)، لذلك، على مسافات r صغيرة جدًا بالنسبة لـ a، يميل المصطلح (e^(-r/a) إلى 1 وضربه في صيغة نيوتن يحصل على صيغة نيوتن، من ناحية أخرى ومن ناحية، على مسافات r كبيرة جدًا بالنسبة إلى a، فإن الحد (e^(- r/a) يميل إلى الصفر، وبالمثل فإن حاصل ضربه في صيغة نيوتن للجاذبية يميل أيضًا إلى الصفر.
    وبما أنه لا يوجد فرق بين نيوتن والكون البسيط على مسافات صغيرة، فهناك أيضًا تطابق مع النسبية. أما على المسافات الكبيرة فتختلف النقطة لأنه لا توجد في الواقع جاذبية وفقًا للكون البسيط، وبالتالي لا يوجد توافق مع نيوتن أو النسبية. ولا ننسى أن نيوتن أيضًا ليس له تطابق مع البيانات المقاسة في مجال حركة المجرات إلا إذا أضفنا المادة المظلمة والطاقة مثل الملك. يتم حل مشكلة الدوران عن طريق الكون البسيط مع اختلافات الضغط الموجودة فيه بسبب جوهره كغاز.
    أتمنى أن أكون قد فهمت. إذا لم تتمكن من الوصول إلى مدونتي، فربما سيكون هناك المزيد من الفهم.
    اسبوع جيد
    سابدارمش يهودا
    http://yekumpashut.freevar.com/

  117. نورث كارولاينا.

    لم أستطع أن أفهم لماذا تتحرك "الثقوب" بالسرعة التي ذكرتها. ما يتحرك هو الجسيمات، ويمكن أن تتحرك بسرعات أعلى بكثير من سرعات الشبكات (إلا إذا كنت تقصد أن الثقوب عبارة عن تراكب لسرعة الجسيمات + الشبكات).

    لكنني لا أرى أي صلة، لنفترض أننا حصلنا على جاذبية أقل بسبب التأثير الذي وصفته. وطالما أننا لم نقم بقياسها كميا، فلا يمكننا أن نعرف مدى تأثيرها حقا.

    مثال: إذا كانت قوة الجاذبية بين الأرض والقمر تريليون نيوتن عندما يكونان في حالة سكون بالنسبة لبعضهما البعض، ولكنها تقل بمقدار 10 نيوتن عندما يكونان في حالة حركة كما في الواقع، فإن التأثير لا يذكر.

    المهم والذي تحدثت عنه هو أنه مهما كانت سرعة تحرك الشباك فإنها لن تواجه "رياح" جسيمات اللاساج وهي احتكاك فاينمان. وهذا على عكس الشبكة التي تتحرك في الهواء مثلاً، حيث أنك ستواجه الريح من الجو.

    يأخذ؟

  118. إسرائيل،

    دعونا نجعل الأمر بسيطًا ونركز على مثال الطريق السريع... وسأكون ممتنًا إذا تمكنت من الإجابة على الأسئلة أدناه.

    1. أقف بجوار طريقك السريع، وألقي نظرة على الثقوب التي "تم إنشاؤها" عندما تمر السيارات عبر الفلتر أ. أشاهد الحفر تتقدم على طول الطريق السريع. لذا فإن أسرع ثقب سأراه هو ثقب بسرعة 100 كم/ساعة (هذه هي السرعة القصوى للجسيمات التي تتفاعل مع الفلتر).
    هل تتفق مع هذه الجملة؟ نعم او لا؟ إذا لم يكن الأمر كذلك، يرجى التوضيح.

    2. إذا كانت الإجابة بنعم، فلنفترض أنني أرى الفلتر B يتحرك على الطريق السريع بسرعة 120 كم/ساعة (في نفس اتجاه الثقوب). هل توافق على أنه من وجهة نظري، لا يمكن لأي ثقب يحدث في المنخل "أ" أن يحدث ذلك؟ (يبدو الأمر تافها بالنسبة لي، لكني أتأكد فقط..) نعم أم لا؟

    3. إذا وافقت على القسم 2، يجب أن توافق على أنه، من وجهة نظري، باعتباري شخصًا يقف على جانب الطريق السريع، فإن المرشح B لن يواجه الجاذبية من الاتجاه A. نعم او لا؟
    إذا لم يكن الأمر كذلك، فيرجى توضيح كيف يتمكن شيء لا يلبي ثقوب المنخل أ من تجربة جاذبية المنخل أ

    شكرا لك وآسف ومساء الخير

  119. يهودا،
    فكيف يمكن قبول النظرية النسبية من ناحية وعدم قبول تقريبها الذي ينتج عنه نيوتن من ناحية أخرى؟ لا توجد خيارات هنا. وكما قال واينبرغ، فإن صيغة نيوتن للجاذبية تنبثق حتماً من تقريب النسبية. فكيف تقبل النسبية دون قبول نيوتن؟ كيف تناسب؟

  120. إسرائيل
    أنت تتجاهل المشكلة مرة أخرى 🙂 أنت تريد أنه عند أي سرعة، سيكون متوسط ​​سرعة الجزيئات مساويًا لهذه السرعة - أليس كذلك؟ وأوضح ألبانزو سبب عدم نجاحه.

    يمكنك تكييف نموذج الجسيمات مع الواقع من خلال المزيد والمزيد من الإضافات، كما يفعل يهودا، ولكن لماذا هذا التشابه؟ لفكرة المحاكاة. وصف أفلاطون نوعين من التماثيل. الأول نسخة من الواقع، كتمثال دقيق للإنسان بكل أبعاده. والثاني مثل قيام الإغريق ببناء تماثيل كبيرة، أو مثل تاج محل: يتم تشويه الأجزاء العليا بحيث تبدو من وجهة النظر بالحجم الصحيح. الجاذبية الدافعة، مع الإضافات لحل الاحتكاك، تشبه المنحوتات الكبيرة - تشويه لنموذج ليتناسب مع الواقع.

  121. إسرائيل،

    أنت لم تفهم المشكلة التي أصفها.. تخيل للحظة "الثقوب" الموجودة في طريقك السريع، هذه هي المسافات بين السيارات التي أحدثها الفلتر الأول. تتحرك هذه الثقوب على الطريق السريع بين السرعة 0 والسرعة القصوى - على سبيل المثال 100 كم/ساعة. هذه هي الطريقة التي حددتها، إذا كانت السيارة سريعة جدًا فلن تتعطل، وبعد ذلك ببساطة لن يكون هناك أي ثقوب عند سرعة أعلى من 100 كم/ساعة.

    الثقوب هي الجزء المثير للاهتمام، لأنها تسبب الجاذبية... ومن لم يواجهها لا يعاني من الجاذبية.

    هذا يعني أن لديك عددًا لا يحصى من الثقوب بسرعة تتراوح من 0 إلى 100 وحتى ثقبًا بسرعة أعلى. تتحرك هذه الثقوب في اتجاه الفلتر الثاني. ولكن بعد ذلك، إذا كان الغربال الثاني يتحرك بعيدًا عن الأول بسرعة 120 كم/ساعة، فلن يتمكن أي ثقب من الوصول إليه، أليس كذلك؟ (في الواقع، إذا فكرت في الأمر بعناية، فسوف "تحصل" على ثقوب، مما يعني أنها على الأرجح ستواجه الجاذبية في الاتجاه الخاطئ..)

    حتى لو كانت سرعة الغربال 5 كم/ساعة فقط، فهناك مشكلة في 5% من الثقوب (تلك التي تكون بسرعة 0 إلى 5 كم/ساعة).

  122. مجهول
    سؤال: ما هي المسافات الكبيرة في علم الكون؟ الجواب: الجاذبية لمسافة كبيرة هي لمسافات أكبر بكثير من المجموعة الشمسية حيث تبلغ المسافات عدة عشرات من الوحدات الفلكية وهي أقل من ألف سنة ضوئية وسنة ضوئية و والمزيد منها عبارة عن مسافات متوسطة وهذه هي المسافات إلى النجوم الأقرب إلى الشمس. يقع أقرب بروكسيما سنتوري على بعد 4 سنوات ضوئية، أي ما يزيد قليلاً عن ربع مليون وحدة فلكية. أعظم المسافات في علم الكونيات توجد في المجرات التي يبلغ نصف قطرها عشرات الآلاف من السنين الضوئية، وتبلغ المسافة بينها ملايين السنين الضوئية. إذا حاول المرء على مسافات كبيرة استخدام صيغ نيوتن للجاذبية، فإن الكثير من المادة مفقودة في المجرات. ولذلك يُزعم أن هناك الكثير من المادة المظلمة الإضافية مختبئة في المجرة.
    وهنا ننتقل إلى سؤال شموليك الذي يصر على صحة إضافة الكتلة المظلمة. صحيح جدا
    عندما تضيف كتلة داكنة فإن كل شيء يعمل. لكن الناس ينسون أن الصيغ ليست مهمة لأنه إذا كانت البيانات المقاسة في الميدان، وإذا كانت البيانات لا تتطابق مع ما تنبأت به الصيغ، فيجب التخلص من الصيغ!لا توجد صيغة مقدسة إلى الحد الذي يجعلنا نغير البيانات المقاسة . أثبتت صيغة نيوتن للجاذبية نفسها بشكل جيد على مسافات صغيرة. ويظهر تزوير صغير لها في مبادرة كوكب عطارد، والتي يتم تصحيحها عن طريق التصحيح النسبي للنظرية النسبية. لكن على مسافات كبيرة يجب البحث عن احتمال آخر فمنذ ثمانين عاما لا نجد الكتلة المظلمة.
    عزيزي فالنيسيم أقول إن الظاهر أنك لم تقرأ ردي السابق فأقول مرة أخرى:-
    أشرح اعتماد الزمن على الجاذبية بمساعدة... النظرية النسبية. ليس لدي أي اعتراض على النظرية النسبية، وأما سؤالك الثاني فأنت تسأل: منذ متى لم يتم العثور على شيء يثبت عدم وجوده؟
    لم أزعم ذلك قط، لكن البعض يزعمون، على سبيل المثال، أنهم يزعمون أن تجربة ميكلسون مورلي تثبت عدم وجود الأثير لأن الضوء لا يستخدم الأثير.
    وعزيزتي إسرائيل، لدي إمكانية الوصول إلى التلسكوبات ذات التتبع، وسأكون سعيدًا بالمساعدة، فقط ضع في اعتبارك أن بعض أعضاء الجمعية لا يزالون في الولايات المتحدة، والبعض الآخر يواصل الذهاب إلى صربيا لحضور مؤتمر النيازك.
    شاب شالوم
    يهودا

  123. المعجزات

    صدر الرد على الزيادة الجماعية، هل رأيتم؟

    لنبدأ بحقيقة أنني لا أنوي الدفاع عن نموذج ليساج، كما ذكرت عدة مرات. اهتمامي هو أكثر النسبية.

    إن مسألة الجسيمات التي تتحرك بسرعات لا نهائية ليست حاسمة بالنسبة لجاذبية لاساج، لأنه إذا تجاوزت سرعة معينة، كما أوضحت في مثال البندول الباليستي، فإنها لن تتفاعل مع الجسيمات. لذلك من الممكن الاكتفاء بسرعات عالية لجسيمات ليست لا نهائية.

    تعتبر السرعات اللانهائية حاسمة فقط عند محاولة دمج المفهوم ليشمل غير المحلية.

    لا أفهم لماذا يجب أن يكون التوزيع موحدًا للحصول على جاذبية لازاج، ولكن يقال أن هذا هو الحال. وماذا في ذلك؟

    أنظر إلى نموذج الأوتوسترادا، وهو عبارة عن لاساج في بعد واحد. هل ترى أنك سوف تحصل على الجاذبية هناك؟ هل ترى أن حركة الشباك لا تغير موضعها بالنسبة لوزن السيارات وبالتالي لن يكون هناك احتكاك بل ستظل هناك جاذبية؟ هل ترى أنه فقط أثناء تغيير السرعة ستكون هناك قوة متوازنة على الشباك وبالتالي ستحصل على القصور الذاتي (ما زلت غير متأكد من هذا، لكنه ليس بهذه الأهمية).

    والأهم بالنسبة لي: هل ترى أن الموجة التي تتقدم في مثل هذا النموذج المفتوح تتقدم دائمًا بسرعة معينة بالنسبة لكل جهاز قياس، ولا يهم ما هي سرعة جهاز القياس؟

  124. إسرائيل
    كيف تتغلب على المشكلة التي أثارها ألبينزو؟ أن أذكر - يتبع من تفسيرك أن هناك عددًا لا حصر له من الجسيمات في جميع الكائنات، في كل مكان.

  125. نورث كارولاينا

    انت تكتب:

    لذا، إذا ابتعدت عن (أو اقتربت من) جسم ضخم فسوف أعاني من جاذبية أقل (مقارنة بحالة السكون بالنسبة للجسم الضخم). وذلك لأن هناك جسيمات لم تتفاعل مع الجسم الضخم وتفاعلت معي أيضاً، وبالتالي فإن الجسم الضخم لم يحجب كل ما وعد بحجبه.

    أفترض أنك عندما تكتب "الابتعاد عن (أو الاقتراب)" فإنك تعني "في حركة نسبية".

    لنأخذ الطريق السريع كمثال: تسير السيارات في كلا الاتجاهين وبجميع السرعات.

    تضع مرشحًا ضخمًا على الطريق السريع، فيحجب بعض السيارات. وعلى مسافة معينة منه تضع فلتر آخر فيقوم بحجب بعض السيارات أيضاً. وفقا ليساج سيكون هناك تجاذب بين المرشحين، هل تقبل؟

    دعونا نقدم تيكون إسرائيل. أحد المرشحات يتحرك بالنسبة للآخر. فهو الآن يحجب السيارات بسرعات لم يحجبها من قبل، لكن السيارات الأخرى التي تم حظرها من قبل تتمكن من المرور لأنها لا تستجيب للفلتر. يأخذ؟

    سنرى ما هو التأثير على الفلتر الثاني.

    بالنسبة لها لا يوجد تغيير في الوضع. لا يزال هناك عدد أكبر من السيارات التي تصطدم به من جانب واحد أكثر من الجانب الآخر، لذا ستؤثر عليه قوة مساوية في اتجاه الغربال الأول، أليس كذلك؟

  126. يهودا،
    لم تجيبيني. ويقول شون كارول إن السبب (واحد على الأقل) الذي يجعل الفيزيائيين يعتقدون بوجود المادة المظلمة هو أنه بإضافتها إلى الحسابات لا داعي لتغيير النظرية والنتيجة التي تم الحصول عليها من الحسابات تتفق تماما مع نتائج التجارب
    ويقول إن العكس تمامًا يحدث عند محاولة إصلاح صيغ الجاذبية. ردك؟

    في محاضرة ستيفن واينبرغ (جميلة جدًا لدرجة أنني استغرقت بعض الوقت للعثور عليها مرة أخرى) يصف أنه بينما بالنسبة لنيوتن فإن الاعتماد على 1 مقسومًا على مربع المسافة هو أمر اعتباطي حيث تم وضع المربع لأنه يناسب النتائج (وفي الواقع بالنسبة إلى دقة القياسات التي كانت موجودة في زمن نيوتن، كان من الممكن بنفس القدر مكان 2.01 والحصول على نفس النتيجة)، فإن معادلات أينشتاين غير قابلة للتلاعب. كل شيء ينبع من الفكرة الأساسية وهي أن الفضاء منحني نتيجة وجود الكتلة، والمعادلات التي تنبع من الفكرة الأساسية جامدة. وأي محاولة للعب بها ستؤدي إلى هراء وإبطال الفكرة الأساسية للنظرية. من معادلة أينشتاين للأجسام التي تتحرك ببطء، ظهر له مربع نيوتن، لكن يجب أن يكون مربعًا وليس 2.01 (كتب ألبانزو هذا من قبل إذا لم أكن مخطئًا. ألبانزو، أنت في صحبة جيدة)
    https://www.youtube.com/watch?v=x9Jqgxh6D2s
    يبدأ القسم الصامت في الدقيقة 18:20 وبشكل ملموس من الساعة 22:20 حتى الساعة 27:00 تقريبًا

    أي أنك إذا قبلت النظرية النسبية، فيجب أن توافق على أنه لا يمكن تغيير نيوتن (أو الجدال رياضيًا مع واينبرج)، بل وأكثر من ذلك، أخبرك شون كارول أنه جرت محاولات لتغيير نيوتن وهذا أدى إلى التناقض مع النتائج التجريبية، لذا فإن ما تبقى على الأرجح هو المادة المظلمة لأنها إضافة جماعية دون تغيير الفكرة الأساسية

  127. نسيم إسرائيل وآخرون
    سأبدأ بالقول إن نظريتي تشرح الجاذبية على مسافات صغيرة تصل إلى بضع وحدات فلكية. ليس لديها أي تناقضات (تقريبا) مع النظرية النسبية على المسافات الصغيرة، وتقبل كل ما تستنتجه النظرية النسبية. في هذه المرحلة، أتقبل تجربة ميكلسون-مورلي وكل استنتاجاتها باستثناء نتيجتين مهمتين لا أعتقد أنهما نابعة من التجربة، وهما أن سرعة الضوء كانت في الماضي كما هي اليوم، و والشيء الثاني هو أن الموقع غير موجود. الضوء لا يستخدم موقع نقطة. كما أنه لا يستخدمني ونسيم وإسرائيل وألبانزو وبشكل عام موقع المعرفة. ومع ذلك فإنه لا يؤثر على وجودنا أو عدم وجودنا. وكذلك على الموقع. يمكن أن يستمر نظام تحديد المواقع العالمي (GPS) في العمل بسبب... النظرية النسبية، كما هو مذكور أعلاه أيضًا مع كوكب عطارد ومبادرته، كل شيء موجود بسبب النظرية النسبية. ولذلك لا مجال للسؤال عن كيفية تفسير شيء تفسره النظرية النسبية. أشرح ذلك بمساعدة النظرية النسبية.

    النظرية النسبية مع نظام تحديد المواقع العالمي (GPS) وغيرها، والمبادرة الكورية الشمالية والقنابل الذرية تعمل وتثبت بشكل جيد على مسافات أقل من ألف سنة ضوئية (النظام الشمسي)

    لكن نظريتي لديها مشكلة مع الجاذبية على مسافات كبيرة، وكذلك الحال بالنسبة لنيوتن والنسبية. لا تقل لي أن إضافة الكتلة والطاقة المظلمة إلى البيانات لا يمثل مشكلة. ولذلك بما أن النظرية النسبية تتحدث عن الجاذبية في الكون بأكمله (على ماذا؟ هل قام أحد بقياسها هناك؟) حتى لمسافات كبيرة تصل إلى مليارات السنين الضوئية، فأنا أختلف معها في هذه الحالة. بالنسبة لي، يعمل الأمر بشكل مختلف على مسافات كبيرة، تقريبًا بدون جاذبية.
    آمل ألا يُطرح السؤال بعد الآن: كيف تفسر نظريتي النسبية على مسافات صغيرة؟

    لكن، لا تعتقد أنني أحب النظرية النسبية، فأنا أحب غاليليو، ولا أحب الارتباط النسبي للسرعات والاستنتاجات الأخرى. لكنها تعمل وأنا أتواصل مع نظرية الإسناد حتى بدون "الحب" ولكني أحب أن أقرضها.
    ولهذا السبب كنت سعيداً جداً بالفكرة اللطيفة والمثيرة للاهتمام التي طرحتها صديقتنا إسرائيل وهي أننا قد "نعض" في النظرية النسبية وأنا أستعد للعطلة القادمة للتفكير في الاحتمالات التي تنفتح حول الموضوع.
    وبالمناسبة يمكن لنظريتي أن تفسر بالفعل حركة بدارية كوكب هيما دون إسناد. لمن يريد الدخول للمدونة
    شاب شالوم
    يرجى الرد بلطف
    يهودا
    http://yekumpashut.freevar.com/

  128. الأمر صعب بعض الشيء هنا مع حالات الانتظار التي لم يتم الإفراج عنها، فلنأمل أن تختفي.

    نمط الرصاصة عبر البلاد ليس كاملاً ودقيقًا، كما بينت في المقال الأصلي. ولم يتم طرحه إلا لتوضيح الفكرة الرئيسية المتمثلة في مرور المقذوفات عبر البندول الباليستي، وهو مثال مثالي. والتي لا تحتاج إلى أن تكون جزيئات للحكيم. ويمكن شحنها بالجسيمات من خلال مجال مغناطيسي (مما يزيل احتكاك كلفن الذي أشار إليه نسيم) أو ببساطة الإشعاع. كل شيء سوف يندرج تحت فئة poshing.

    هل الضغط صحيح؟ لا أعرف (ولكنك لا تعرف ذلك أيضًا!). هل هو مهم بشكل خاص؟ ليودا آمنة.

    أنا مهتم بالنسبية، والآلية التي اقترحتها ربما يمكنها تفسير عكس سرعة الضوء وليس من خلال مسلمة.

    من الصعب إدخال التفاصيل الشخصية في كل مرة، هل يعرف أحدكم حيلة تجعل الكمبيوتر يحفظها؟

    ليلة سعيدة للجميع (سوف تمر؟).

  129. يهودا،
    وقال إنه وفقًا لإشعاع الخلفية، كانت التنبؤات المتعلقة بالمادة المظلمة دقيقة، في حين فشل تعديل صيغة الجاذبية مقابل الملاحظات. واضح وسلس. أي أن هناك أدلة قوية جدًا هنا على صحة نظرية نيوتن خارج المسافات القصيرة للنظام الشمسي. هل لم تعد تدعي أن قانون الجاذبية بحاجة إلى التغيير لأنه لم يتم اختباره بما فيه الكفاية؟

  130. يهودا، لقد رأيت للتو تعليقك.

    الرأس التركي يعمل بشكل صحيح. إذا قرأت الرابط الأول الذي قدمته للمقال منذ خمس سنوات مع يوفال و ر.ه.، فستظهر فيه الاستنتاجات التي استخلصتها.

    عيد سعيد.

  131. من قائمة الانتظار:

    ويوضح روفين نير في كتابه "الجاذبية" أن الكتلة التي تدور بسرعة حسب ليساج تشبه شبكة دوارة من شأنها أن تحجب المزيد من الجزيئات وبالتالي تزيد الكتلة.

    لم أفهم لماذا الجاذبية النيوتونية غير صحيحة. لم يشرح نيوتن أبدًا أصل الجاذبية، وهذا ما حاول ليساج فعله.

  132. زيادة الكتلة: وفقًا لرؤوفين نير، فإن القرص الدوار يحجب المزيد من الجزيئات، تمامًا مثل الغربال الدوار. لذلك، وفقًا ليساج، فإن المزيد من الجسيمات المحجوبة يعني كتلة أعلى.

  133. وظيفتي ليست الدفاع عن نظرية ليساج، فمنتجنا يودا يفعل ذلك بإخلاص.

    وظيفتي هي حل مشكلة الاحتكاك. المكافأة: أيضًا مشكلة مصدر القصور الذاتي (فقط عن طريق التسارع ستحصل على قوة على الجسم) وسبب عكس سرعة الضوء. وربما أيضا احتمال عدم المحلية.

    هل هذا ما يحدث حقا؟ على ما يبدو لا. الميزة: يمكن اختبار ذلك تجريبيًا، وهو الآن مجرد مسألة تحسين التكنولوجيا الحالية.

  134. إسرائيل
    لقد فهمت ما تقوله. لكنك تستخدم الجاذبية النيوتونية لإثبات أن الجاذبية النيوتونية غير صحيحة.
    وماذا يحدث إذا كانت النظرية النسبية؟ لماذا تزداد الكتلة مع السرعة؟

  135. المعجزات

    إن القذيفة التي تمر عبر نفق في إسرائيل بأقل من سرعة الإفلات سوف تلتقط حركة توافقية بسيطة وتنقل إليها زخمًا يتناسب مع سرعتها بالنسبة إلى إسرائيل.

    والشيء المذهل هو أنه أبعد من سرعة الإفلات حتى بجزء واضح من السرعة، فإنه لن ينقل الزخم على الإطلاق. الميكانيكا النيوتونية.

    إذا طبقنا المبدأ على جسيمات لاسيج، يمكنك التفكير في بُعد واحد لطريق سريع ذو اتجاهين تسير عليه السيارات في كلا الاتجاهين بسرعات مختلفة عديدة، لكنك كشراع منتشر عبر الطريق السريع، لا تتفاعل إلا مع نطاق ضيق من السرعات. وبالتالي فإن القوة المتوازنة المؤثرة عليك هي صفر، مثل القوة المؤثرة على أي جسم مملوء بالهواء في حالة السكون.

    إذا كنت تسير بسرعة مختلفة، فإن القوة المتوازنة المؤثرة عليك ستظل صفرًا، لأن بعض السيارات عالية السرعة ستختفي من منظورك ("ما وراء سرعة الهروب") وستظهر في سياراتها سيارات جديدة لم تشعر بها. مكان.

    لذلك، على عكس الهواء الذي يمارس قوة (الريح) عليك، إذا كنت في حركة بالنسبة إليه، بغض النظر عن مدى سرعتك بالنسبة للطريق السريع، فإن القوة المؤثرة عليك ستكون مساوية للصفر.

  136. إسرائيل

    أولا، عطلة سعيدة.

    في ردك علي كررت ببساطة ما قلته سابقًا عن الاحتكاك. لكنني لم أقل أن ما كتبته عن الاحتكاك خاطئ، لكن في محاولتك لإصلاح الاحتكاك أفسدت شيئًا آخر - الآن دفع الجاذبية المحسن الخاص بك لا يفعل ما يفترض أن يفعله - إنتاج الجاذبية.

    تظن أنك سباك، وجئت لإصلاح صنبور متسرب عند أحد العملاء، وقد أصلحت المشكلة بالفعل! لكن الآن الصنبور مغلق - لا يخرج منه ماء على الإطلاق حتى لو فتحته... وبعد ذلك عندما يخبرك بذلك، تقول: "هذا ليس دقيقًا، يمكنك أن ترى أن الصنبور لا يتسرب. " حقا جميلة..

    على كل حال أنصح بشرب الكثير من الماء، وإذا كان لديك الوقت فراجع ما كتبته، وإذا لم يكن كذلك فلا بأس أيضاً... وكل عام وأنتم بخير.

  137. إلى شموليك
    ذهبت إلى الرابط الذي وجهتني إليه، وأوضح شون كارول أشياء معروفة وذكر أيضًا الضعيف الذي يتم البحث عنه دون جدوى. كما أنه لا يستبعد احتمال أن يكون هناك شيء ما في الجاذبية ليس صحيحا. لذلك لم يخبرني كثيرًا.
    يهودا

  138. إسرائيل
    مقدار الزخم المنتقل هو جزء لا يتجزأ من القوة مع مرور الوقت. في حالة المقذوف الموجود في الذيل، عند سرعة معينة لن تزداد القوة لأن القص سيحدث. وفوق هذه السرعة، سينخفض ​​مقدار الزخم بشكل ملحوظ. لن يتم إعادة ضبطه بأي سرعة.

    فيما يتعلق بالقذيفة من خلال النفق. أعتقد أن الأمر لا علاقة له بالسرعة، بل بالوقت. تخيل أن قذيفة بعيدة جدًا تمر عبر الحفرة. في البداية، انجذب إلى الأرض. يزداد زخم المقذوف وكذلك زخم الأرض. بعد الانتقال، ينعكس الوضع وفي النهاية ستتوقف الرصاصة. وهذا يعني أنه سيتم إنتاج البندول.

  139. المعجزات

    وبينت أنني أعتقد أن الجسم الذي تكون سرعته أعلى من سرعة الإفلات، لن ينقل أي زخم إلى الأرض إذا مر عبر نفق فيها.

    الأمر نفسه ينطبق على الأجسام التي تمر عبر بندول باليستي، لكن يكفي أن نقول إنها تنقل كمية حركة أقل فوق سرعة معينة.

    ولماذا نذهب بعيدا؟ أي مقذوفة تفضل أن تضرب ذيلك الوهمي: تلك التي تنتقل بنصف سرعة الصوت أم تلك التي تنتقل بنصف سرعة الضوء؟ من سيلحق المزيد من الضرر بالطائرة؟

  140. يهودا
    قبل 150 عامًا، رأوا شذوذًا في مدار كوكب عطارد. ألا تعتقدون أننا لن نرى اليوم شذوذاً في مدارات الأقمار الصناعية التي يبلغ ارتفاعها عدة مئات من الكيلومترات؟ لا تكن ساذجًا 🙂 اقرأ قليلاً عن اكتشافات كاسيني وشاهد دقة المعلومات التي وصلنا إليها في دراسة زحل. كما كتبت - قلة الاحتكاك تتعارض مع نظريتك.

    كيف تفسر التغير الزمني في السرعة والجاذبية؟ كيف تفسر التأثير الكهروضوئي؟ التعاليم التي ترفضها تشرح هذه الأشياء. نحتاج إلى ما هو أكثر بكثير من العذر "لكنهم لم يجدوا المادة المظلمة!!!" لرفض مثل هذه التعاليم الناجحة.

    فيما يتعلق بالشحنات - النيوترون لا يتكون من بروتون وإلكترون. يتكون كل من البروتون والنيوترون من 3 كواركات، والإلكترون عبارة عن جسيم من نوع اللبتون. الكواركات لها شحنات +2/3 و-1/3 من الإلكترون. لماذا بالضبط هذه الأرقام؟

  141. للمعجزات
    الأقمار الصناعية تسقط بين الحين والآخر، لا تنسوا أن لديها وقودًا يصحح مسارها أحيانًا، أعتقد أن تلك التي تطير إلى الشرق ستدوم أقل.
    فيما يتعلق بنظرية الجاذبية التي "تتطلب" إنتاج كتلة وطاقة مظلمة، أين نظرياتك فائقة النجاح؟ نيوتن لا يفسر شيئًا، وأنا أيضًا أشرح حركة المجرات دون إضافات جماعية.
    وفيما يتعلق بالنظرية النسبية وتجربة مايكلسون مورلي فهي مقبولة بالنسبة لي بكل استنتاجاتها النسبية للمسافات القصيرة. لا يبدو لي أن هناك أدلة نسبية للمسافات الكبيرة لا تتطلب منك اختراع أشياء مجنونة. لكن استمر في الإيمان بالكتلة المظلمة والطاقة المظلمة. لا تقلق بشأن المعجزات، فعندما يجدون أن القمر الصناعي الشرقي يعيش أقل من القمر الغربي، سيكون هناك بالفعل شخص من وكالة ناسا سيقول أن ذلك ليس بسبب الاحتكاك المفرط، فهذه المحركات ساخنة (أو باردة) أو ذلك لقد نقلوا أكثر (أو بالأحرى أقل) من هناك وهذا هو السبب. لن يتخلوا عن نيوتن بهذه السرعة.
    فيما يتعلق بالبروتون والإلكترون، لا أعرف ماذا تعني الشحنة الكهربائية حتى أتمكن من شرحها، لكن سأحاول على أي حال - لدي شعور بأننا أخذنا شيئًا ما من البروتون والنتيجة هي بروتون وإلكترون، لذا يجب أن يكونوا متساوين، فإذا كان لدي 30 طالبًا في صفي وأخرجت طفلًا واحدًا، فسيتبقى لدي 29 طالبًا ويخرج واحد واحد. الواحد هو بالضبط 30 ناقص 29. لكن كما تعلمون،
    إسرائيل
    الاستعداد لقراءة الرابط الذي أرسلته لنا. مثير للاهتمام.
    هذا كل شيء، وتأتي وجبة السبت.
    لذا يرجى الرد بلطف. أعلم أن الناس لا يحبون مهاجمتي لنيوتن وأينشتاين.
    يوم جيد وسنة جديدة سعيدة! ويرجى الرد بلطف
    يهودا
    http://yekumpashut.freevar.com/

  142. إسرائيل
    لكي لا ينقل الجسيم أي زخم، يجب أن تكون سرعته لا نهائية (بافتراض الاستمرارية). ومن ناحية أخرى، فإن الجسيم ذو السرعة اللانهائية لن ينقل الزخم إلى أي جسم، لذلك يمكن تجاهله.
    لقد عدنا إلى البداية...

  143. لا بد لي من الطيران للعمل. أنا لا أدعي أن نموذج Lesage صحيح. يعاني من مشاكل كثيرة، ليس الاحتكاك الثاني (كلفن) الذي أثاره نسيم سوى واحدة منها.

    أنا أدعي فقط أن المشكلة الرئيسية (احتكاك فاينمان) يمكن حلها باستخدام الفكرة التي طرحتها. لكن يجب أولاً أن نفهم مبدأ البندول الباليستي، ولماذا تتوقف الأجسام التي تمر عبره عن نقل الزخم إليه فوق سرعة معينة.

    يمكنك أيضًا القراءة على:

    https://www.hayadan.org.il/astronomers-reach-new-frontiers-of-dark-matter-130112/comment-page-11/#comment-326491

  144. يهودا
    تدور الأقمار الصناعية فوقنا في كلا الاتجاهين منذ عقود (أعتقد أكثر من 30 عامًا). أجد صعوبة في تصديق أن ظاهرة الاحتكاك الخاصة بك لم تكن لتكتشف خلال هذه الفترة. فكر في أنهم قبل 150 عاماً كانوا يعلمون أن هناك انحرافاً في ميل كوكب عطارد، حجم الانحراف أقل من نصف ثانية في السنة!

  145. يهودا
    أنت بحاجة إلى سبب وجيه لنظرية جديدة، خاصة إذا كانت ترفض النظريات الموجودة والتي حققت نجاحًا كبيرًا. النظرية النسبية تنبأت بالعديد من الملاحظات التي أكدتها، فهي نظرية بسيطة، وهي تفسر ظواهر لا تفسرها نظريتك مثلا. هناك مثالان بسيطان هما اعتماد الوقت على السرعة واعتماد الوقت على الجاذبية.
    أما نظرية الكم فهي أكثر تعقيدا، ولكنها تتنبأ بالمخرجات التجريبية بدقة تصل إلى 12 رقما. لقد تنبأت بالعديد من الجسيمات التي تم اكتشافها، ومرة ​​أخرى - تشرح الملاحظات التي لا تستطيع التوراة الخاصة بك اكتشافها.

    هناك أشياء أبسط بكثير من المادة المظلمة لا يمكن تفسيرها. على سبيل المثال - لماذا تكون شحنات الإلكترون والبروتون متماثلة تمامًا من حيث القيمة المطلقة؟ فهل تعتقد أن هذا سبب وجيه لعدم الأهلية؟

    ألبينزو - هل أنا على حق بشأن أجهزة الشحن؟

  146. ألبانزو، إسرائيل، نسيم وآخرون.
    "أ، أنا لا أفهم لماذا الحاجة المطلقة لعدد لا حصر له من الجسيمات؟ بعد كل شيء، إذا افترضنا أننا نأخذ كمية محدودة من الجسيمات التي 99.999999 في المئة من الجسيمات سوف تتحرك بين سرعة 0 وXNUMX V، يمكننا أن وما زلنا نقول بدرجة عالية جدًا من الدقة أن متوسط ​​السرعة هو V ونستخلص استنتاجات قريبة جدًا من الواقع. نحن لا نطالب بالاحتكاك تمامًا، بل نطالب بالاحتكاك الذي سيبقي كوكبنا الثمين على قيد الحياة لمليار سنة أو سنة أخرى.

    ب. سؤال جوهري. إذا كنا نتحدث عن نظرية جديدة أو فكرة جديدة مثل تلك التي تأتينا بها إسرائيل، فهل استخدام نظرية موجودة (النسبية أو الكم) أمر لا بد منه؟، مجرد سؤال.
    سنة جديدة سعيدة يا ألبانزو ودعونا نأمل أن القطط السوداء قد انتهت.
    سنة جديدة سعيدة للجميع أيضا
    يهودا
    http://yekumpashut.freevar.com/

  147. عزيزتي إسرائيل
    من الممتع في التعليقات أن تنتظر فجأة بعد أن تصادف شيئًا لم تفكر فيه من قبل، وعلى الفور يبدأ رأسي التركي في استخلاص استنتاجات "طائرة".
    لذلك عندي لك شيئين:
    أ. ألن يكون هناك توزيع بولتزماني أيضًا فيما يتعلق بدفع الجاذبية للجسيمات بسرعات مختلفة؟ أي لنفترض أن متوسط ​​السرعة سيكون مليون كيلومتر في الثانية بالنسبة لـ 999,000 أو 1,009,000 سيكون هناك عدد أقل قليلاً من الجسيمات، وبالتالي سيكون هناك تغيير في الجاذبية التي تنشأ في كلتا الحالتين أعلاه؟
    ولنفترض أن الجاذبية لا تنشأ إلا من تحرك الجزيئات بسرعة 900,000 ألف كيلومتر في الثانية، وكل شيء آخر يمر دون أن نشعر به، فإذا تحرك الكوكب بسرعة عشرة كيلومترات في الثانية، اعتمادًا على اتجاه الحركة، فإن عدد الجسيمات الموجودة فيه فالمنحنى الغوسي بهذه السرعة سيكون أقل من ذلك في حالة واحدة، وأكثر من ذلك في الحالة الثانية فإن الجاذبية ستزداد أو تقل حسب الدوران حول الشمس بالطبع. مثير للاهتمام!

    ب. الأمر الثاني وهو الأهم:- نحن نتحدث باستمرار عن نفس السرعة تجاه الأجسام وبغض النظر عن مدى سرعتها، ألا يذكرك هذا بالنظرية النسبية. ربما بعد كل شيء، غاليليو يتحكم في الفضاء بمساعدة تصحيح إسرائيل شابيرا؟، ربما لهذا السبب تكون سرعة الضوء واحدة في كل اتجاه وليس بسبب ما قاله صديقنا ألبرت؟ هل فهمت كيف يعمل الرأس التركي بسرعات مختلفة؟
    لذا يرجى الرد بلطف. أعلم أن الناس لا يحبون الإساءة إلى روح نيوتن الرقيقة وحتى روح قريب ألبرت.
    يوم جيد وسنة جديدة سعيدة!
    يهودا
    http://yekumpashut.freevar.com/
    ملحوظة: في هذه الأثناء، رأيت رجلاً جديدًا يأتي إلى الحي. سنة جديدة سعيدة ألبانزو!

  148. إسرائيل،

    شيء لتفكر به. ليس عليك الإجابة إذا كنت لا ترغب في ذلك، هذه هي المرة الأولى التي أرى فيها تفسيرك للادعاء (الذي سمعتك تدعيه من قبل) بأن مشكلة الاحتكاك يمكن حلها مع الجاذبية الميكانيكية. أرى مشكلة في الحل الخاص بك مما يجعله غير ذي صلة على الإطلاق، ولكن ليس لدي أي نية للدخول في جدالات، لذلك سأشرح فقط ما هي المشكلة كما أراها ويمكنك أن تفعل بها كما يحلو لك.

    إن مجرد توزيع الجسيمات بسرعات مختلفة بين اللانهاية وسالب اللانهاية لن يحل المشكلة. يجب أن يكون التوزيع موحدا. على المستوى الرياضي، يتم الحصول على ذلك من حقيقة أن إضافة السرعة إلى الجسم في مجال الجاذبية سيؤدي إلى تغيير التوزيع، وللحصول على تلك الجاذبية، يجب أن يكون التوزيع ثابتًا بالنسبة للإزاحات (في الزخم بالطبع). على المستوى البديهي، يمكن فهم ذلك من حقيقة أنه لكي لا تتغير الجاذبية نتيجة لسرعة الجسم (= لا يحدث احتكاك)، فبالضبط لكل جسيم إضافي يجمعه الجسم في اتجاه اتجاهه. السرعة، يجب أن يكون هناك جسيم يتوقف عن المساهمة في الجاذبية الميكانيكية (يصبح "شفافًا" بالنسبة له كما حددته). مرة أخرى، يتطلب هذا توزيعًا موحدًا للسرعات. من السهل إجراء تجربة باستخدام التوزيع الغاوسي ورؤية أنه عند السرعات العالية سيكون هناك ببساطة عدد أقل بكثير من الجسيمات التي تكون سرعتها النسبية للجسم في النطاق المناسب، وبالتالي ستكون الجاذبية أضعف.

    لكن التوزيع الموحد لديه مشكلة حرجة، وهي أن العدد الإجمالي للجسيمات فيه لا نهائي. ليس عددًا كبيرًا جدًا، وليس "كثيرًا" - لمنع الاحتكاك بالطريقة التي تقترحها، فأنت بحاجة إلى عدد لا حصر له من الجسيمات الموجودة في جميع السرعات وفي كل اتجاه. إذا أخذت عددًا منتهيًا - مهما كان كبيرًا - فإن التكامل على توزيعه يجب أن يتقارب، وبالتالي يجب أن يتلاشى إلى 0 عند الحواف، وستشعر على وجه الخصوص بالاحتكاك (لأن التوزيع لن يكون ثابتًا على الإزاحات، و دائمًا ما يكون هناك جسيمات "خلفها" أكثر من "الأمام" بشكل مقاربي، وهو بالضبط ما يخلق الاحتكاك).

    كلاسيكيا، هذا هراء. كلاسيكيا لا يوجد شيء مثل عدد لا حصر له من الجسيمات. هذا ليس نموذجًا فيزيائيًا صالحًا، وبالتأكيد ليس نموذجًا ميكانيكيًا (الذي يدعي تفسير الظاهرة من خلال التفاعلات الحركية). في هذا الصدد، إنها خدعة بلياردو لا تعمل إلا عندما يكون هناك عدد لا نهائي من الكرات على الطاولة. أخرج حتى واحدًا - ولن تنجح الحيلة بعد الآن.

    الكم لديه الكثير ليتحدث عنه. إنها ليست تافهة، ولكن الكم هناك آليات لخلق وتدمير الجسيمات التي يمكن أن تجعل المفهوم أكثر مادية. لكن بالطبع إذا كانت الجسيمات كمّية، فإنك ستواجه مشكلتين جديدتين. أولا، الاعتبارات الميكانيكية الكلاسيكية لم تعد صالحة. الفكرة الكاملة عن الجاذبية الميكانيكية هي أن هناك جسيمًا يتحرك في المدار، وإذا كان هناك شيء بينك وبين الجسيم الذي يخفيك، فإن الجسيم لن يؤذيك. لذلك، عملياً ستشعر بضغط سلبي من هذا الاتجاه وستنجذب إلى الجسم المخفي. لكن لا يوجد كم للجسيم المداري. وحتى لو نظرت إلى الجسيمات التي بدأت حياتها في اللانهاية وتريد الوصول إليك (في طريق غير محدد)، فبالطبع عليك أن تأخذ في الاعتبار النفق، وبشكل عام فهو مقصّر في شرح مدى النموذج الميكانيكي يحتاج إلى التغيير من النهاية إلى النهاية ليكون متسقًا مع ميكانيكا الكم. سأذكرك أن النماذج الكمومية للجاذبية هي من أصعب المشكلات التي واجهناها في الفيزياء. هذا الادعاء في حد ذاته ليس حجة ضد الفكرة، ولكنه مجرد تذكير بعدد آلاف المشاكل التي تنشأ عند محاولة التوفيق بين الصديقين معًا، مشاكل سيتعين عليك حسابها إذا ادعيت أن جزيئات الجاذبية الميكانيكية هي الكميات.

    المشكلة الثانية ليست مشكلة تمامًا كما ستكون: لماذا؟ سأذكرك أن الجرافيتونات هي أيضًا جسيمات تخلق قوة الجاذبية ميكانيكيًا (عن طريق نقل الزخم بين جسمين). الفرق هو أنها ليست أثيرًا، بل هي جسيمات يمكن العثور عليها في حالة مرتبطة في الفضاء (مثل المادة)، أو تنبعث تلقائيًا من الأجسام التي تؤدي تفاعلات الجاذبية. وبعبارة أخرى، فإن التناظر الكمي للجاذبية الميكانيكية موجود بالفعل منذ بضعة عقود. إذا كانت الجسيمات الموجودة في النموذج الخاص بك هي الكميات، فليس من الواضح بالنسبة لي ما يقدمه النموذج غير موجود في النماذج الحالية التي تتضمن الجرافيتونات (بغض النظر عن حقيقة أن نموذجك "أصغر سنًا" بكثير ولم يتم اختباره مقارنة بهذه النماذج التي تمت دراستها لعقود من الزمن وحققت تقدمًا نظريًا كبيرًا).

  149. نورث كارولاينا.

    نوع، ولكن ليس دقيقا.

    لا تنس أن هذه عبارة عن عدد لا حصر له من الجسيمات التي تتحرك بجميع السرعات من سالب ما لا نهاية إلى ما لا نهاية، ولكنها تتفاعل مع الجسم بسرعات معينة فقط (مثل الأرض ذات النفق الذي تنقل إليه المقذوفات 1-11 الزخم من عدد لا حصر له من المقذوفات بسرعات من ناقص ما لا نهاية إلى ما لا نهاية، وكل شيء آخر "شفاف").

    الآن ماذا سيحدث إذا كانت السيارة ذات سرعة مختلفة؟ سوف يستجيب لـ 11 قذيفة أخرى وسيكون الباقي شفافًا.

    ولذلك، مهما كانت سرعة تحرك الرصاصة، فهي في نفس الوضع بالنسبة لجميع المقذوفات.

    وفي نظام لازاج، بغض النظر عن مدى سرعة تحرك الكوكب، فهو يكون عند 0 سرعة بالنسبة لجميع جزيئات لازاج، تمامًا كما تكون سرعة الورقة 0 بالنسبة للهواء الساكن، على الرغم من أن جزيئات الهواء تتحرك بسرعة عالية. بالنسبة للورقة، ولا يوجد أي منها في الواقع عند 0 سرعة بالنسبة لها.

    وكما أن الورقة الساكنة بالنسبة للهواء لا تحتوي على احتكاك فاينمان مع الهواء ولكن فقط إذا كانت تتحرك، كذلك فإن الكواكب أيضًا ليس لديها احتكاك مع جسيمات لاسيج: بغض النظر عن سرعة الكوكب، فهو عند 0 السرعة النسبية لجميع الجزيئات.

    يودا عطلة سعيدة، دعونا نأمل أن تكون القنابل الوحيدة التي نواجهها هي قنابل الظهر.

  150. عزيزتي إسرائيل
    لقد رأيت ردكم على المعجزات المتعلقة بهيكل القنبلة الذرية الذي لدينا في حي شمال كوران في هرتسليا، المدينة المقدسة، ويمكننا أن نتعلم منهم حقًا عن جسيمات لو سيج - دفع الجاذبية - كون بسيط. واستخلاص استنتاجات حول الجاذبية المحبوبة ويمكن أن تكون النتائج مثيرة للاهتمام بالنسبة لمصير الكون البسيط والكون بشكل عام.
    لم أفكر في الأمر بهذه الطريقة. في الواقع، غذاء للفكر!
    سنة جديدة سعيدة يا إسرائيل يا صديقي
    يهودا
    http://yekumpashut.freevar.com/

  151. للمعجزات
    دعونا نجري حسابًا تقريبيًا لاحتكاك الأقمار الصناعية مع دوران الأرض وضده
    وبشكل عام، يطير القمر الصناعي بسرعة ثمانية كيلومترات في الثانية. تبلغ سرعة الأرض 40,000 ألف كيلومتر في 24 ساعة، أي 0.46 كيلومتر في الثانية، وبالتالي فإن فروق الاحتكاك بين الحالتين ستكون مثل 8.46 إلى 7.54، أي حوالي 12 بالمائة، وبما أن الاحتكاك يكون حسب مربع السرعة، فهو يقع في حوالي ربع. ليس كثيرا، لكنه "مقبول"، أي أنه إذا استمر القمر الصناعي العادي 100 عام، فإن القمر الصناعي الإسرائيلي سيستمر حوالي 75 عاما، بالمناسبة، سبب إرسال المركبات الفضائية نحو الشرق هو الاستفادة من مسافة 0.46 كيلومتر. في الثانية عند إرسال الصاروخ إلى الفضاء.
    معجزات يوم جيد
    يهودا

  152. يهودا
    وبسبب القيود، فإن الأقمار الصناعية التي تطلقها إسرائيل تدور في الواقع في الاتجاه المعاكس لدوران الأرض. وفقًا لما وصفته، كان ينبغي أن يزيد الاحتكاك بشكل كبير. لا أعتقد أن هذا هو الحال.

  153. المعجزات
    إن دوران الشمس حول محور يرجع إلى اتجاه دوران السحابة البدائية التي تشكل منها النظام الشمسي. الشمس والكواكب لها نفس الاتجاه العام للدوران. إذا تحركت مثل أي شخص آخر، فإن فرصك في البقاء على قيد الحياة أكبر مما لو تحركت بشكل مختلف. وهذا ما قلته أيضًا عن الأقمار الصناعية.
    مساء الخير
    يهودا

  154. المعجزات، مرة أخرى، عطلة سعيدة لك أيضا!
    دعونا نرى ما كتبته: إجابتي - بين قوسين (). لنبدأ:
    «الريح يحركها مصدر خارجي وهو الشمس» (صحيح).
    "جزيئاتك مثل الغاز في الفضاء المفتوح" (صحيح أن جزيئاته شديدة النفاذية)
    "" وليس فيها ريح "" (غير صحيح، هناك اختلافات في الضغط والرياح)"
    "أي أن متوسط ​​سرعتهم هو 0" (هذا صحيح، عادةً في المتوسط).
    "أنت تقول مرارًا وتكرارًا أن الجزيئات تتحرك في اتجاه حركة الكوكب" (هذا صحيح، الجزيئات تحرك الكوكب وتحرك نفسها في اتجاه الكوكب)،
    "لكنني أوضحت لك أن الأمر لا يسير على ما يرام في حالة حيث يدور قمران صناعيان حول نجم ويمران عبر نفس النقطة" (هذا ليس صحيحًا لأن الجسيمات لها نفاذية عالية، ولكن الأقمار الصناعية ستسقط في النهاية إلى واحد على الأقل من النجوم) منهم، الذي يتحرك ضد اتجاه الدوران الذاتي للكوكب).
    "وعلى أية حال، لا يوجد كوكب يتحرك في مدار "ثابت" (الثابت هو مسألة تعريف بالنسبة إلى ماذا؟، الأمر معقد).
    "الأرض تدور حول الشمس، والشمس تدور حول مركز مجرة ​​درب التبانة، ومجرة درب التبانة لا تقف ساكنة أيضًا (بالنسبة إلى إشعاع الخلفية الكونية)" (صحيح)
    . وبعبارة أخرى، تمر الكواكب عبر جزيئات "جديدة". (صحيح)
    لتلخيص المعجزات، فما هو الاستنتاج، تحقق مما إذا كنت تفهم مبدأ خلق الجاذبية عن طريق دفع الجاذبية. يتفق الجميع تقريبًا على أن الجاذبية خلقت. هناك من يزعم أن الأمر لن يدوم طويلا بسبب الاحتكاك، وهنا نختلف
    يرجى الرد بلطف
    سنة جيدة!
    يهودا
    http://yekumpashut.freevar.com/

  155. يهودا
    أولا - عطلة سعيدة!
    يحرك الرياح مصدر خارجي وهو الشمس. جسيماتك تشبه الغاز في الفضاء المفتوح وليس لها رياح، أي أن متوسط ​​كتلتها هو 0. أنت تقول مرارًا وتكرارًا أن الجسيمات تتحرك في اتجاه حركة الكوكب، لكنني أوضحت لك أن هذا لا يضيف إلى الموقف حيث يدور قمران صناعيان حول نجم ويمران عبر نفس النقطة.

    وعلى أية حال، لا يوجد كوكب يتحرك في مدار "ثابت". تدور الأرض حول الشمس، وتدور الشمس حول مركز مجرة ​​درب التبانة، كما أن مجرة ​​درب التبانة لا تقف ساكنة أيضًا (بالنسبة إلى إشعاع الخلفية الكونية). أي أن الكواكب تمر عبر جزيئات "جديدة".

  156. للمعجزات
    سأكون مختصرا في تحليل ردك
    قلت: "الريح تدفع القارب في البحر، وفي هذه العملية تفقد الريح قوتها" نهاية الاقتباس
    وماذا في ذلك، في هذه الأثناء، مرت الريح التي فقدت قوتها، وجاءت ريح جديدة، منعشة وبكل قوتها، تواصل إبحار السفينة. هذا هو بالضبط ما يحدث مع الجاذبية التي تدفع الجسيمات التي تصطدم بالأجسام وتمر عبرها بسرعة الضوء (على ما يبدو) وعلى الفور تستمر الجسيمات الجديدة في الوصول بكامل قوتها.
    عزيزي نسيم، مع كل التقدير لكلامك، لا فائدة من استمرار الجدال بيننا، أرى أن الجميع يحصنون موقفهم وواثقون فيه. نحن قبل رأس السنة الهجرية، لذا دعونا نقول وداعًا هنا، وإذا قمت بزيارة هرتسليا خلال العطلة، سأكون سعيدًا بلقائي معك ومناقشة دفع الجاذبية، وربما نتوصل إلى بعض الاستنتاجات
    أتمنى لكم ولجميع المجيبين على العلوم ولجميع شعب إسرائيل سنة جديدة سعيدة ومباركة
    كل خير!
    يهودا

  157. يهودا
    تدفع الرياح القارب في البحر، وفي هذه العملية تفقد الرياح قوتها. وبدون طاقة الشمس لن تكون هناك رياح بسبب الاحتكاك.

    العلاقة بين دوران الرياح ودوران الكواكب فضفاضة للغاية. تدور الرياح بسبب مزيج من الحركة الخطية والحركة الدورانية للأرض، مما يخلق ما يسمى بقوة كوريوليس. تكتسب الأعاصير الطاقة في البحر لأن البحر يجعل الهواء دافئًا ورطبًا، ولأن البحر مسطح ويسهل حركة الهواء.
    هناك ظاهرة مثيرة للاهتمام في الطيران تسمى "اتصال الرياح" - كلما صعدت إلى أعلى، يتحرك اتجاه الريح في اتجاه عقارب الساعة، وهذا بسبب انخفاض الاحتكاك مع الأرض.
    تدور الكواكب لأنها تشكلت من جسيمات جاءت من بعيد، ولها طاقة حركية أولية، مما يعني أنها تحتوي على مكون سرعة غير شعاعية. إذا أخذت الكثير من الجسيمات وأجريت عملية تكامل، فسوف ينتهي بك الأمر إلى نظام عبارة عن قرص دوار، لأنه في النهاية يوجد زخم دوراني وهو متجه (أعلم أنه تفسير واهٍ بعض الشيء).

    صحيح أن هناك القليل من الاحتكاك في الفضاء، اسأل ناسا ماذا حدث لـ Skylab في ذلك الوقت بسبب هذا الاحتكاك.
    لكن الاحتكاك الناتج عن جزيئاتك أكبر بكثير، وبالتالي سيتسبب في تحلل مداري أسرع بكثير.
    ومع ذلك، فإن حساباتك خاطئة ولا تصف أي شيء مادي. وطالما أن الكوكب في حالة حركة، سيكون هناك احتكاك ولن يتم إنشاء التوازن أبدًا. على العكس من ذلك، كلما اقترب الكوكب من الشمس، زادت قوة الجاذبية، وزادت سرعة الدوران (الطولي) وبالتالي الاحتكاك.
    في رأيي، من الممكن إعلان "كش ملك"، أليس كذلك؟

  158. إن حل Yesh.R.A.L للاحتكاك جميل، لكنه سيفسد الجاذبية.

    الحل هو أن الجسيمات الموجودة في نطاق سرعة معين فقط هي التي تتفاعل مع الجسم.

    لذا، إذا ابتعدت عن (أو اقتربت من) جسم ضخم فسوف أعاني من جاذبية أقل (مقارنة بحالة السكون بالنسبة للجسم الضخم). وذلك لأن هناك جسيمات لم تتفاعل مع الجسم الضخم وتفاعلت معي أيضاً، وبالتالي فإن الجسم الضخم لم يحجب كل ما وعد بحجبه.

  159. المعجزات
    استجابة مثيرة للاهتمام.
    سأبدأ من نهاية إجابتك - مثال المركب الشراعي لا يتطابق تمامًا مع الشعاع الموجود في النظام الشمسي، ولكن مثلما يتحرك المركب الشراعي في البحر على الرغم من "الاحتكاك" بالرياح والأمواج، كذلك سوف يتطابق أيضًا يتحرك الكوكب على الرغم من الاحتكاك. مصدر الطاقة هو مصدر الجسيمات الدافعة الجاذبية، والتي مصدر طاقتها ربما/ربما هو الانفجار الأعظم ولديها الطاقة المطلوبة.
    يجب أن يكون هذا كافيًا للحركة الدورانية وليس هناك حاجة لشرح ما سيحدث إذا استمر الاحتكاك. لأنه لا يوجد احتكاك
    لكنك أثرت سؤالا مثيرا للاهتمام فيما يتعلق بكل من التفسير النيوتوني والتفسير البسيط للكون: لماذا تدور الكواكب أصلا ولماذا في نفس الاتجاه؟. إن الذي يعطيهم هذه الدفعة وفي رأيي المتواضع هو الذي يعطيهم الدفع هو دوران المجرة نفسها وإلا فإن الكواكب من المفترض أن تسقط في الشمس. ويحدث أمر مماثل مع الرياح الموجودة على الأرض والتي دورانها هو ما يعطي الحركة الدورانية للرياح (الأعاصير القمعية والأعاصير) وطالما أنها تتحرك في البحر المفتوح فإن سرعتها تحافظ على سرعتها بل وتزداد رغم الاحتكاك الكبير مع البحر. بحد ذاتها. هذا هو الوضع في تفسير دفع الجاذبية.
    خذ في الاعتبار أن الاحتكاك ينشأ (ولو بدرجة أقل) في الحركة أيضًا وفقًا لنيوتن، وأيضًا وفقًا لنيوتن هناك احتكاك في الحركة في الفضاء لأن الفضاء "الفارغ" ليس فارغًا تمامًا (ذرات الغاز، النيازك، الكويكبات، (الرياح الشمسية والأشعة الكونية والنيوترونات وما إلى ذلك) مما سيجعل الكواكب في حالة توازن غير مستقر والاحتكاك القليل سيؤدي في النهاية إلى سقوطها في الشمس إذا لم تكن هناك آلية لإصلاح ذلك.
    لكن وفقًا للحسابات التي أجريتها، حتى السقوط البسيط باتجاه الشمس سيخلق جاذبية أكبر لأننا في مدار داخلي أكثر وبالتالي بمساعدة المجرة وربما الشمس، سيتم ترجمته إلى حركة دورانية كافية .
    لكن... أنت على حق، مشكلة اتجاه تأثير الاحتكاك "مزعجة" بعض الشيء، ولكن أيضًا بالنسبة للتفسير النيوتوني.
    ولكن مرة أخرى، كما يقول رئيس وزرائنا الحبيب: "لا يوجد احتكاك لأنه لم يكن هناك أي احتكاك على الإطلاق، ولكن حتى لو كان هناك احتكاك يمكن حله (ربما)" في رأيي بمساعدة الشمس والمجرة ولكن. .. ربما انا على خطأ.
    مكان للفكر. لكن هذا ليس سببًا للتوقف عن الإيمان بالكون البسيط،
    يرجى الرد بلطف. إنه مجرد علم.
    يهودا
    http://yekumpashut.freevar.com/

  160. يهودا
    حساباتك في نهاية المقال خاطئة. قوة الجذب المركزي شعاعية بينما قوة الاحتكاك طولية. لا يتم التخلص من الاحتكاك عن طريق تقليل نصف القطر، وفي النهاية سوف يسقط الكوكب في الشمس.

    ما علاقة المراكب الشراعية في البحر؟ هناك مصدر طاقة خارجي يسبب الرياح (يسمونها "الشمس").

  161. للمعجزات
    وتمر بها الجزيئات تقريبًا دون أن تصطدم بالأجسام التي تمر عبرها. وهي تتصرف مثل جسيمات النيوترينو التي يمكنها عبور الأرض بأكملها دون أن تدرك أنها مرت عبرها ولكن تقريبًا. بعضها مكابح، لذا لا يهم أين تتحرك الأقمار الصناعية، فإنها جميعًا ستصطدم بالجزيئات، وبنفس القدر تقريبًا ستتعامل مع الجسيمات التي تدفع الجاذبية كجزيئات غازية صغيرة ذات نفاذية عالية جدًا. جوجل أو ويكيبيديا حول دفع الجاذبية، لو سيج. أو على مدونتي تم شرحه جيدًا
    مساء الخير
    يهودا
    http://yekumpashut.freevar.com/

  162. لسنوات كنت مقتنعا أن مساحة منزلي 312 مترا مربعا.
    أفهم الآن أن احتمال أن تبلغ مساحة منزلي 312 مترًا مربعًا يميل إلى الصفر.
    أنا لا أزن 72 كيلوغراما أيضا.
    وطولي 176 سم.
    لقد سقطت الأرض من أجل لا أكثر.

  163. للمعجزات
    لماذا لا تتقاطع الأقمار الصناعية أو تتحرك في اتجاهين متعاكسين؟ ففي النهاية، هذه هي الطريقة التي أرسلناهم بها إلى الفضاء. علماً أن جميع أقمار المجموعة الشمسية تتحرك في نفس الاتجاه، ويبدو أن تلك التي وصلت من اتجاه مختلف لم تدم، والسؤال هو ما إذا كانت الأقمار الصناعية ستحافظ على حركتها في الفضاء لمليارات السنين. لاحظ أنه في أحد اتجاهات دورانهم يزداد الاحتكاك، ولكن في الاتجاه الآخر سيستفيدون فعليًا من "الرياح الخلفية".
    وأما السؤال الثاني عن الحركة اللانهائية في خط مستقيم، فهو أنه في الاتجاه من الشمس إلى الخارج يكون للأجرام تباطؤ، بل وانحراف معين عن الخط المستقيم. حاولت حسابها ولم أستطع، لقلة الرياضيات. ويكون التباطؤ بحسب سرعة الأجسام. ولهذا أعتقد أنه يجب إرسال مركبة فضائية للتحقق من هذه الظاهرة.
    هل هناك من يتطوع للقيام بالحساب؟
    يرجى الرد بلطف. إنه مجرد علم.
    يهودا
    http://yekumpashut.freevar.com/

  164. المعجزات
    فالاحتكاك الذي تتحدث عنه هو احتكاك مختلف عن احتكاك فاينمان الناتج عن حركة الكواكب، وسيكون موجودا حتى لو كانت الكواكب في حالة سكون. وهذا هو الاحتكاك الذي تحدث عنه اللورد كالفن في مقالته عن ليساج.

    لمعرفة سبب عدم أهمية ذلك، فكر في نفق يمتد بين القطبين الشمالي والجنوبي (لا يوجد غلاف جوي) ويتم الوصول إليه بواسطة مقذوفات بسرعة تتراوح من 1 إلى 20 كيلومترًا في الثانية. من سينقل أكبر قدر من الزخم إلى الأرض، وهل سيكون هناك فقدان للطاقة بسبب الاحتكاك؟

    الجواب على السؤال الثاني سلبي، وعلى الثاني: المقذوف رقم 11 (سرعة الإفلات). 12-20 رصاصة، وأي رصاصة ذات سرعة أعلى لن تنقل الزخم، ومن وجهة نظر الدولة فهي "شفافة".

    بدلًا من ذلك، فكر في كتلة من البلاستيسين معلقة من خيط على شجرة وقم بإطلاق مقذوفات عليها بسرعات مختلفة. من سيحرك الكتلة أكثر؟

    كما كان من قبل، سترى أنه فوق سرعة معينة ينقل الرصاص زخمًا أقل، وفوق سرعة معينة يتوقف عن نقل الزخم على الإطلاق.

    وحتى جزيئات النيوترينو السريعة تمر عبر آلاف الشموس دون جهد واحتكاك، ودون أن تترك أثرا على الشمس أو تتباطأ.

    وقد سبق أن ذكرنا النيوترونات البطيئة التي تفجر اليورانيوم بينما تمر منها السريعة دون مشكلة.

    وهذا مبدأ طبيعي، فلا يوجد سبب يمنعه من العمل في حالة جزيئات اللاساج، وهو يحل مشكلة احتكاك فاينمان.

  165. يجال
    تحتوي الخلية السرطانية على نفس الحمل الجيني تقريبًا مثل بقية خلايا الجسم. وبهذه الطريقة "يعرف" الجهاز المناعي أنه لا يهاجم خلايا الجسم، وبالتالي الخلايا السرطانية أيضًا.

  166. يهودا
    إجابتك لا تشرح كيف يمكن لقمرين صناعيين عبور نقطة في الفضاء في اتجاهات مختلفة. كما أنه لا يفسر كيف يمكن لجسم أن يتحرك في الفضاء لمسافات غير محدودة في خط مستقيم.

  167. إسرائيل
    إذا لم تحصل على الاحتكاك فلن تحصل على الجاذبية أيضًا. وليس كتلة.
    وبمجرد اصطدام جسيم سريع جدًا بالنيوترون على سبيل المثال، سيتلقى النيوترون كمية كبيرة من الطاقة، وبالتالي الحركة البراونية.

  168. نسيم، والدي، ياريف، يهوذا، إسرائيل

    بالحديث عن السرطان، آسف، لا علاقة له بالموضوع:
    سؤال، كيف يمكن للخلية السرطانية أن "تعرف" كيف تخدع جهاز المناعة في الجسم (لحجب المواقع البينية للأجسام المضادة بحيث لا يمكن تحديدها للتخلص منها ؟؟؟؟ إنها مجرد خلية غبية أصيبت بالجنون والتكاثر) كثيراً.
    ما هو؟ بكتيريا أو فيروس يمر بتريليونات الطفرات حتى يتمكن أحدها من خداع الخلايا التائية.

  169. للمعجزات
    الرابط الذي أرسلته لي منذ حوالي عشرة أو عشرين تعليقًا يتحدث عن تنظيف الكيلو جرام الصحيح، لذلك قاموا بتنظيفه كثيرًا وتحتاج إلى إضافة بضعة ميكروجرامات إليه، لا أفهم ما هو بالنسبة لأعيننا. وهذا يوضح صعوبة تحديد الكيلو جرام الذي قد يمتص الأوساخ ونحو ذلك.
    وفيما يتعلق بالاحتكاك كما أوضح لك إسرائيل (شكرا على المساعدة يا صديقي)، فلا شك أن نشاط الجسيمات كان سيخلق الجاذبية وبالتالي الدوران أيضا. لكن الجاذبية ستؤثر على جميع الأجسام الموجودة في النظام الشمسي وأيضًا... على الجسيمات نفسها، وبالتالي سيكون لها أيضًا لحظة دوران حول الشمس، وبالتالي سيكون متوسط ​​حركتها مثل حركة الكوكب في منطقتها ، لذلك لن يكون هناك احتكاك. لقد ضرب ريتشارد فاينمان مثالا على احتكاك شخص يتحرك ضد المطر، بالتأكيد ستضرب وجهه المزيد من قطرات المطر، لكن المثال ليس جيدا لأنه لو افترضنا أن الشخص أيضا تهب عليه الريح فإنه يتحرك بسرعة من القطرات وسوف تتبلل بالتساوي من الأمام والخلف. وتضرب إسرائيل مثالاً ينبغي دراسته بعمق، وهو ماذا سيحدث لو أن سرعة معينة فقط من الجزيئات كان لها تأثير ولم يكن للبقية تأثير. بصراحة، لم أفكر في ذلك. سيكون هناك شيء يجب القيام به في عطلة نهاية الأسبوع.
    باختصار، مشكلة الاحتكاك التي أفسدت نظرية الجاذبية الدافعة ليست قاتلة للنظرية، لذا سأستمر فيها. في انتظار الدليل الفائز الذي سينزع إيماني بالكون البسيط
    الرجاء الرد بلطف، SA هو مجرد العلم
    يهودا

  170. لنفترض للتبسيط أن الجسيمات عبارة عن دببة صغيرة من الشوكولاتة، وأن النجوم والكواكب عبارة عن شبكات ضخمة تنقل معظم الجسيمات ولكن ليس كلها.

    هل ستحصل على ليزر الجاذبية عندما تكون الألواح في حالة راحة؟ إيجابي. ويقول فاينمان هذا أيضًا.

    هل ستواجه احتكاكًا في طراز Lesage الأصلي؟ إيجابي أيضًا، وهذا ما يقوله فاينمان.

    هل ستحصل على الجاذبية في نظام مفتوح حيث تتحرك الجسيمات بجميع السرعات من سالب ما لا نهاية إلى ما لا نهاية ولكنها تتفاعل مع الشبكات بسرعات معينة فقط ولكن ليس خارجها؟ إيجابي.

    هل سيكون هناك احتكاك؟ سلبي. عند أي سرعة للشبكة، يكون متوسط ​​السرعات التي تصطدم بها الجسيمات ذات الصلة بالشبكات هو 0، مثل الهواء حول ورقة تضربها جزيئاتها بجميع السرعات وتوازن أي منها.

  171. إسرائيل
    هذا كثير من التلويح باليد. قانون تصادم الجسيمات ليس اصطدام رصاصة بقطعة هلام. في فهمي، التشبيه الجيد للجسيمات هو كرات البلياردو.

  172. من فضلك إيراف كيف أعرف؟ وماذا يهم؟ تظهر جميع التفاصيل في الإدخال الخاص بنموذج Lesage على ويكيبيديا. وظيفتي هي حل مسألة الاحتكاك فقط، وإذا قرأت الشرح عن البندول الباليستي تبين أن الجسيمات السريعة تنقل كمية حركة أقل من الجسيمات البطيئة (مثل النيوترونات البطيئة التي تنفجر نواة اليورانيوم بينما تمر منها السريعة دون مشكلة )، ففي رأيي أن الشرح يحل مشكلة الاحتكاك.

    وأيضا ثبات سرعة الضوء في جميع الأنظمة المرجعية..

    وأيضا مصدر الجمود..

    وأيضا إمكانية عدم المحلية..

  173. للمعجزات
    نعتذر، في هذه الأثناء وصل "الدليل".
    حسنًا، لقد قمت بطرح الحالة التي نتحدث عنها في مقال KG فقدان الوزن. لذا، أولاً وقبل كل شيء، لقد كنت مخطئًا، فالأمر لا يتعلق بخسارة الكيلوجرام كتلته، بل وزنه. تبقى الكتلة ثابتة.
    التفسير هو هذا. وفقا للكون البسيط، ونظرية الجسيمات الدافعة للجاذبية ولاسيج، فإن من يحدد الوزن هما الجسيمان اللذان يدفعان الجاذبية. إذا كان الكون يتوسع، فإن الجسيمات الدافعة للجاذبية تملأ مساحة أكبر، وبالتالي فإن الجسيمات الدافعة ذات الجاذبية الأقل ستؤثر على الكتلة لكل وحدة زمنية، وبالتالي ستحدد وزنًا أصغر مع مرور الوقت. من السهل القيام بالحسابات لأن ثابت هابل معروف إلى حد ما. لقد أجريت الحسابات وتبين أن كل كيلوغرام سيفقد قيمة تقل قليلاً عن نصف ميكروغرام لكل كيلوغرام سنويًا على الأرض. عجبا لنظرية. اللي بيحصل فعلا في الميدان، في نقص في الوزن كده بالظبط، لكن محدد إنه بسبب نسبي إلى كيلو تاني. لا أعرف كيفية علاج ذلك، لكن إذا لم يكن هناك فقدان للوزن، فلماذا تغيير التعريف؟ أقترح إجراء تجربة وأخذ أي كتلة ووزنها على موازين الضغط الحساسة ومعرفة ما إذا كانت تفقد حوالي 0.45 ميكروجرام لكل كجم سنويًا. يتذكر. الموازين العادية التي تستخدم الأوزان لن تساعد لأن الأوزان أيضًا تفقد وزنها ولن تفعل ذلك سوى موازين الضغط الحساسة.
    إذا تغير الوزن بعد سنة فقد خسرت. إذا لم يتغير الوزن. وهذا من شأنه أن يشكل ضربة قاسية للنظرية لأنه من المستحيل ألا ينخفض ​​الوزن. وهكذا ستتلقى مني هدية جميلة كما وعدتك وبفرح.
    يرجى الرد بلطف
    يهودا
    http://yekumpashut.freevar.com/

  174. إسرائيل
    احتمال وقوع حدث بالفعل هو 1. لقد رميت للتو عملة معدنية - وحصلت على "شجرة". أعلم أن لدي شجرة، لذا فإن الاحتمال هو 1.
    إذا لم ألقي نظرة على العملة بعد، فإن احتمال أن أرى "شجرة" في المستقبل هو 0.5. لكنه يتحدث عن المستقبل.

  175. للمعجزات وغيرها
    الجزء الثاني'
    لقد كنت سعيدًا ذات مرة (نعم، كنت سعيدًا!) عندما اكتشفت أن الاحتكاك الناجم عن دفع الجاذبية سيمنع الكواكب من الحركة لملايين السنين، ثم (لسوء الحظ) راودتني فكرة أنه لا يوجد احتكاك لأن الجاذبية تدور الجسيمات الدافعة أيضًا حول الشمس. لا يوجد سبب لعدم القيام بذلك. لكن سيكون هناك احتكاك عند الابتعاد عن الشمس مثل بايونير. تشرح نظريتي شذوذ بايونير. أولئك الذين لا يقبلون الجاذبية الدافعة وجدوا تفسيراً آخر للحرارة المنبعثة من محركاتها الذرية. ليكن. أعتقد أنه سيكون من الحكمة إرسال مسبار صغير للقيام بذلك. تحقق مما إذا كان هناك رعشة غير مبررة في الحركة بعيدًا عن الشمس.
    أظهر لي ألبانزو إمكانية أنه في تجربة المحور سيكتشفون ما إذا كان هناك احتكاك في حركة الجسيمات مع الجزيئين اللذين يدفعان الجاذبية. إذن هنا، الرياضيات هناك تتجاوز فهمي ولكنها أيضًا تتجاوز فهم الكثيرين. أنت بحاجة لمقابلة أحد العلماء هناك والسؤال عما إذا كان هناك اضطراب في حركة البروتونات.
    نظريتي أيضًا لا تؤمن بالنقاط المفردة، فأنت تريد أن تصدق حقك. للتذكير، لم يتم قياس قوانين الفيزياء أبدًا بالقرب من نقاط منفردة، وكل ما تم تحديده بشأن سلوكها هو مجرد فرضية لا أساس لها من الصحة، مبنية على قوانين لم يتم إثباتها هناك ولم يتم قياسها إلا على مسافات كبيرة
    باختصار...لقد تعبت، وقدمت الكثير من الاحتمالات لإثبات دحض الكون البسيط، وللأسف التجارب مكلفة بعض الشيء.
    لذلك ربما يكون جميع الرافضين على حق. أنا أعمل فقط مع الأدلة. أرِنِي
    يهودا

  176. المعجزات وغيرها
    لدي نظرية ولا يهمني إذا رأى شخص ما أنها تتعارض مع البيانات الموجودة في المجال، سأكون سعيدًا وسيزيل الصداع الذي أشعر به من جميع أنواع الأشخاص الأذكياء (بدون استهتار)
    أنا لم أقل أن العلماء أغبياء، هذا ما تقوله. الجميع يفكر بشكل أفضل وفقًا للمعرفة المتاحة في ذلك الوقت، وبالتالي فإن بطليموس أيضًا يستحق جائزة على ملاحمه وبالنسبة لي لا بد أنه عبقري على الرغم من أنني أعلم أنه كان مخطئًا تمامًا. نظريتي تشرح الكون بدون المادة المظلمة والطاقة المظلمة. أصريت على استخدامه تتعطّر. تريد استخدام الجاذبية في الكون كله؟ بناء على ماذا؟ بعد كل شيء، حدد نيوتن ما حدده بالاستقراء، ولا ينبغي للصيغة أن تتصرف على الإطلاق حتى على المسافات التي لم يتم قياسها. وأي محاولة لاستخدامها على مسافات كبيرة تؤدي إلى ضرورة القول "لم نقم بالقياس بشكل جيد وهناك مادة أكثر في الكون" نظريتي لا تتعارض "بشكل أساسي" مع النظرية النسبية وعلى المسافات القصيرة فهي متوافقة تماما مع النظرية النسبية ولكنها لا تتوافق مع النظرية النسبية على مسافات كبيرة من الكون لأنها أيضا لا تؤمن بوجود الجاذبية حتى على مسافات بضع سنوات ضوئية. علاوة على ذلك، يعمل فرق الضغط بشكل جيد على مسافات كبيرة.
    لم أعترف أبدًا بأنني لا أملك أي معرفة رياضية. لدي معرفة رياضية كافية لاحتياجاتي (عادة).
    لم يكن أرسطو يفكر مثل الحكيم ولم أعيش في عصره. أنت فقط الخلط بين رأسك.
    وبشكل عام، لاحظ أنه يجب علي دائمًا الاعتذار عن كل الحثالة التي توجهها إليّ أنت والآخرون. لذلك تعبت. أرني دليلاً قاطعًا على أن نظريتي لا تعمل وسأقدم لك هدية ممتعة كشكر لك. لا تكرر التغريدات التي يغردها الجميع بشكل متكرر. هناك أشياء أخرى كتبتها، سنتركها لما بعد العشاء.
    يرجى الرد بلطف. إنه مجرد علم
    يهودا

  177. يهودا
    لا تهتم بنظريتك؟ ما أجمل أن نبدأ الصباح بنكتة 🙂
    وبعد ذلك تظهر هنا مدى عدم فهمك للناس. أنت لا تفهم أن كل عالم، مثل أي شخص آخر، يتوق إلى العثور على أخطاء في الجميع وأن يكون بطل اليوم. وبدلاً من ذلك، أنت تعيش في عالم حيث يكون جميع العلماء عبيدًا لسيد محترم يمنعهم من مخالفة كلماته.
    لقد قيل لك أن العلماء وجدوا تفسيرًا لشذوذ بايونير. بدلاً من قبول الأمر كما هو، فإنك تصفهم بالحمقى والكاذبين (ربما بكلمات ألطف قليلاً ولكن بشكل واضح جدًا).
    يشرحون لك أن هناك الكثير من الأدلة على وجود المادة المظلمة، وهي أدلة لا تستطيع نظريتك تفسيرها.
    يشرحون لك أن نظريتك تتناقض بشكل أساسي مع النظرية النسبية، وهذا أمر لا يذكر من جانبك.
    يخبرونك أن مسألة الاحتكاك تستبعد نظريتك تمامًا، وتعترض على أنها تعزز نظريتك بالفعل.
    أنت تعترف أنه ليس لديك أي معرفة رياضية (مثلي)، لكنك غير قادر على فهم أنها أداة أساسية في فهم الفيزياء. تعتقد أن الكون "بسيط". آخر من فكر هكذا هو أرسطو... يا ترى لماذا؟

  178. المعجزات
    لا يهمني نظريتي وخلافا لرأيك. القياسات فقط هي التي تحدد لي، أرني قياس واحد تجاهلته؟؟ ومن يتجاهل القياسات هم العلماء الذين قرروا تغيير البيانات المقاسة في المجرات لتتناسب مع الصيغة بمساعدة مادة وطاقة وهمية. وإذا أظهرت أن هناك احتكاكًا في حركة بايونير، فإنهم يروون لي قصصًا عن إحماء بايونير إلى المدى الدقيق والاتجاه المطلوب. ولكنني هنا لست الوحيد الذي يدعي أن سرعة الضوء تتغير، فهناك عدة غيرها، وسيكون هناك غيرها الكثير.
    إسرائيل
    لقد سألتني إذا أتيحت لي فرصة مشاهدة النجوم المتغيرة (النجوم الرأسية في السماء) من خلال التلسكوب، فالإجابة هي لا. لكن في الملاحظة القادمة سأطلب من عضو الجمعية أن يريني ذلك. عادةً ما تكون مدة دورتها أيامًا، لذلك لن يكون من الممكن رؤية تغيراتها في ليلة واحدة. لماذا هو مثير للاهتمام لكيوبيد المذكورة أعلاه؟
    يوم جيد
    يهودا

  179. أن الاحتمال هو 1 هو بالتأكيد مسألة رأي. الألغام مختلفة. وحقيقة أن الضوء ليس موجة هي أيضًا مسألة رأي.

    تقبل النسبية استنتاجات نظرية ماكسويل، لكنها ترفض نموذج الأثير الساكن، وهي محقة في ذلك. وهي مرتبطة بالنظرية اليهودية حيث تحدث كل من ليساج وماكسويل عن النموذج الهيدروديناميكي للجسيمات. ولكن في حين أن نظرية ليساج هي في الأساس نظرية نوعية، فإن ماكسويل يقدم وصفًا كميًا دقيقًا مدعومًا بمعادلات للظواهر.

  180. إسرائيل
    الاحتمال هو 1. هذه ليست مسألة رأي.
    لا أفهم سبب اختلاف Weichen. الضوء ليس موجة. معادلاته هي معادلات موجية، لكن هذه ليست الحقيقة.
    وعلى أية حال، هناك ارتباط وثيق بين المجال الكهربائي والمجال المغناطيسي، والارتباط هو النظرية النسبية. لذلك ليس من المستغرب أن تظهر سرعة الضوء هناك.

    كيف يرتبط كل هذا بالشريعة اليهودية؟ التوراة التي لا تصلح للمراقبة. قلت ذلك أيضا.

  181. كيف يختلف عن Weichen؟ ولم لا؟

    جيجل هيشن وجيجيل ماكسويل. تعرف على عدد النتائج التي تحصل عليها لكل منها. إذا كان هويتشين مخطئًا فهذا خطأ هامشي، وإذا كان ماكسويل مخطئًا فهو خطأ فادح.

    لاحظ أيضًا أن ماكسويل لم يأت لشرح الملاحظات الموجودة مثل بطليموس وويتشين. بنى ماكسويل نموذجه واكتشف - لدهشته - أنه يربط بين ثوابت الكهرباء والمغناطيسية وسرعة الضوء.

    إن احتمال أن يؤدي مثل هذا النموذج المعقد والدقيق بالصدفة إلى سرعة الضوء هو احتمال منخفض للغاية في رأيي.

  182. إسرائيل
    - التمييز بين النموذج والواقع. يعد نموذج ماكبويل رائعًا، ولا تزال معادلاته مستخدمة حتى اليوم.
    المشكلة هي الإسقاط الذي تم من النموذج إلى الواقع.
    وكيف تختلف عن نظرية موجة هويتشين؟ نموذج ممتاز لا يزال يستخدمه حتى يومنا هذا كل أخصائي بصريات وكل طبيب عيون وكل مصور.

  183. إسرائيل
    كان لنيوتن نموذج رائع لجزيئات الضوء وكان لدى هويتشان نموذج رائع لموجات الضوء. كانت أفلاك التدوير وأفلاك بطليموس مذهلة أيضًا.
    وكانت الأساطير اليونانية والتوراة رائعة أيضًا في عصرهما، لأنها وصفت ما اعتقدوا أنه واقع.
    وهم في رأيي أفضل بكثير من نموذج يهودا يديدينو، لأنه لم تكن هناك في زمنهم ملاحظات تدحض النماذج.

  184. المعجزات

    لذا فإن نظرية ماكسويل بعشرات الرسوم التوضيحية ومئات المعادلات خاطئة.. شوين.

    واشتقاق سرعة الضوء (المعادلة 136) مجرد صدفة محظوظة... فليكن.

    هل تتذكر السيد نحشون من المتتبع؟ كانت لديه طريقة لتخمين نتائج اليانصيب: كان يحسب النتوءات ويضربها في طول الجدران ويضع الجيران في الداخل. لقد نجح دائمًا.

    وماذا عن معادلات ماكسويل؟ هل هم أيضا نتيجة الحالة العمياء؟ أو ربما مجرد السحر؟

    هل نظرت إلى النموذج؟ هل رأيت مدى تعقيدها ودقتها؟ حقا هذا الساحر ماكسويل..

  185. يهودا
    هل من الممكن أن توجهوني إلى حيث قلت أعتقد أنه لا يوجد تغيير في سرعة الضوء؟
    ولنفترض أن هناك تغيرًا بمرور الوقت في سرعة الضوء، فما علاقة هذا بنظريتك؟ ففي نهاية المطاف، أنت آخر من يعتقد أن الملاحظات تؤكد النظرية...

    إسرائيل
    هل هناك حالات أعطت فيها النظرية الخاطئة نتائج صحيحة؟

  186. إسرائيل شابيرا
    شكرا للمساعدة. فالنيسيم يقول أنه بما أنه لا يوجد قرار حازم، فإنني أخذت باحتمال أن يكون هناك تغير في سرعة الضوء كدالة للوقت (التاريخ)، بينما تفضل احتمال عدم وجود تغيير. لقد أوضحت طريقة لكيفية اختبار ذلك في منشأة LIGO. ولا يهم حقًا إذا كنت أكثر ذكاءً وأفهم الفيزياء أو لا أفهمها. هناك رقم وعليك أن تقرر ما هو الصحيح.
    اتمنى لك يوم جيد
    يهودا

  187. المعجزات

    نموذج موقع ماكسويل

    https://en.wikisource.org/wiki/On_Physical_Lines_of_Force

    ومنه استنتج سرعة الضوء من ثوابت الكهرباء والمغناطيسية، وهو ما يذكرنا كثيرًا بنموذج لاساج أو الكون البسيط، والذي يكون فيه أيضًا كونًا هيدروديناميكيًا (سائل أو غاز)، ويوجد فيه أيضًا جزيئات ، فروق الضغط، الدوامات، التيارات، الحمير الوحشية..

    إذن استنتاجات النموذج - سرعة الضوء، معادلات ماكسويل - صحيحة، ولكن النموذج نفسه خاطئ؟

    ?

    ??

    ؟؟؟!؟؟؟

  188. يهودا
    لقد كتبت أن سرعة الموجة مرتبطة بدرجة الحرارة. وشرحت لك أن هذا صحيح بالنسبة للموجات الطولية، وإذا أردت أستطيع أن أشرح لك السبب. لكنك لست مهتمًا بفهم الفيزياء، بل مهتمًا بتطوير نظريتك رغم الفيزياء 🙂

    إذا سبق لك أن درست الفيزياء، فستعرف أن الحركة العرضية لموجات الضوء هي بالضبط ما يحدد سرعة الضوء (مرة أخرى، معادلات ماكسويل). إذا كنت أتذكر بشكل صحيح، في كتاب بيركلي للفيزياء، يتم حساب سرعة الضوء كنتيجة لتقدم موجة كهربائية مستعرضة، ويتم إنشاء المجال الكهرومغناطيسي العداد بسبب المجال المغناطيسي المستحث (عمودي أيضًا على اتجاه الحركة).

    يمر الضوء عبر الغاز، وكذلك عبر الماس. ما علاقة هذا بالأمر؟ ووسط الضوء ليس جزيئات الغاز.

    لم ألغ أي تغييرات في السرعة. على العكس من ذلك، قلت إنني لا أرى سببًا خاصًا لعدم إمكانية حدوث ذلك (كتبت سابقًا، لأن سرعة الضوء تعتمد على خاصيتين للفضاء (mio-0 وepsilon-0).

    قل - جزيئاتك لا تتحرك في الفضاء؟

  189. المعجزات
    للضوء خصائص عديدة، لكن هل الطول أو العرض هو الذي يحدد سرعة الموجة؟ ويحددون ما إذا كانت ستغير سرعتها عند درجات حرارة مختلفة؟ في أوقات مختلفة؟؟ ما هو الشيء الموجود في الموجات الطولية الذي يتغير سرعته عند اختلاف درجات الحرارة وهذا الشيء غير موجود في الموجات المستعرضة؟
    ثانياً، اعذروني على جهلي، لكن ألا يتحرك الضوء أيضاً في معركة الغاز؟ باختصار، أنت تقرر إلغاء خيار تغيير السرعة، وأنا أقرر قبول خيار التغيير. ما سيتم تحديده هو تجربة ستختبر لعدة ساعات في منشأة LIGO. يجب أن تظهر سرعة الضوء تغيرًا ببضعة أنجستروم في الثانية لمسافة حوالي 1600 كيلومتر يتم قياسها بالليزر في LIGO، وهي الحسابات الأكثر دقة على موقعي. موقع إلكتروني. إذا كان لدى أي شخص اتصالات في منشأة LIGO سأكون ممتنًا له.
    الشيء الثالث - قلت أن مساحتي ليس لها أي خصائص على الإطلاق. للأسف أنت مخطئ والفضاء الخاص بي بمجرد تعريفه على أنه غاز، لديه كل خصائص الغاز، وبالتالي سيكون له درجات حرارة وضغوط واختلافات في الضغط، وبالتالي بالطبع الرياح وعلامة التعاطف مع إعصار إيرما في فلوريدا، إذًا هناك أيضًا أعاصير في كوني، على سبيل المثال إعصار أندروميدا الذي يدور بجانب إعصار بيل الحليب، ولهذا لا تحتاج إلى الجاذبية أو اختراع الكتلة المظلمة والطاقة، فقط فرق بسيط في الضغط، وكل شيء يدور، ولكن هذا لفترة أخرى. وإذا كنت غاضبًا مما كتبته، فتذكر أنه بالفعل الواحدة بعد منتصف الليل، لذا يرجى التراجع والابتسامة والرد بلطف، إنه مجرد علم.
    على أية حال، أنا ذاهب إلى النوم!
    سابدارمش يهودا
    http://yekumpashut.freevar.com/

  190. يهودا
    سرعة الموجة في الغاز مرتبطة بدرجة حرارة الغاز، هذا صحيح. ولكن... هذا عندما يتعلق الأمر بالموجات الطولية فقط. الضوء، لتذكيرك، هو موجة عرضية.

    يهودا - لا توجد موجات عرضية في الغاز...

  191. المعجزات
    تتناسب سرعة الموجة مع جذر درجة الحرارة المطلقة للغاز الذي يتحرك فيه. أعلم أن سرعة الضوء شيء خاص، كما أنه من الصعب على العلماء أن يقبلوا أن الفضاء الفارغ من الكون هو في الواقع غاز، (في نظرية الكون البسيط ينبع هذا من جوهر نظرية الجسيمات) ولكن حتى ذلك ثبت خلاف ذلك، وأفترض أن الضوء لديه هذه الخاصية أيضًا، أي أن سرعة الضوء تتناسب مع جذر درجة حرارة خلفية الكون، وبالتالي إذا انخفضت درجة حرارة الخلفية فإن سرعة الضوء ستنخفض أيضًا. أستطيع أن أعرف مدى تمدد الكون لأنني أعرف ثابت هابل، وبالتالي يبرد الكون تبعاً لذلك، وستنخفض سرعة الضوء أيضاً. لقد قمت بالحساب وحصلت على أن سرعة الضوء تنخفض بمقدار كيلومتر واحد في الثانية سنويًا.
    من ناحية أخرى، في الوقت الذي كان عمر الكون، على سبيل المثال، 380000 ألف سنة، كانت درجة حرارة الكون حوالي 3000 درجة كلفن، أي أكثر بقليل من 1000 مرة من درجة حرارة خلفية الكون اليوم، وبالتالي فإن درجة حرارة الكون تبلغ حوالي 33 درجة كلفن. وكانت سرعة الضوء تعادل تقريباً XNUMX ضعف سرعة الضوء اليوم، أي حوالي عشرة ملايين كيلومتر في الثانية. بهذه السرعة ربما لم يكن من الضروري تحديد التوسع التضخمي للكون كما هو مذكور في الرابط الذي قدمته في ردي السابق.
    وأما ما كتبته من إعجاز عن كلام صديقنا ماكسويل، فأنا أتساءل عما إذا كانت هناك علاقة مقابلة بين النفاذية المغناطيسية، ومعامل العزل الكهربائي، ودرجة حرارة خلفية الكون. إذا كان هناك اتصال، فقد يمنع حدوث خلافات بيني وبين ماكسويل.
    يرجى الرد بلطف
    سابدارمش يهودا
    http://yekumpashut.freevar.com/

  192. يهودا
    وفقا لماكسويل، تعتمد سرعة الضوء على خاصيتين للفضاء: النفاذية المغناطيسية ومعامل العزل الكهربائي. وإذا اعتمدوا على الزمن فإن سرعة الضوء سوف تتغير.

    لكن - ما علاقة هذا بنظريتك؟

  193. للمعجزات وغيرها
    شكرا لك على الرد الكريمة
    وفيما يلي مقتطف من الرابط المرفق: لاحظ أنني لست وحدي في آرائي.
    "يقدم الباحثون من جامعة واترلو في كندا وإمبريال كوليدج لندن نظرية مختلفة قليلاً. ووفقا لهم، فإن سرعة الضوء ثابتة بالفعل في جميع الأنظمة المرجعية في أي لحظة معينة، ولكنها تتغير بمرور الوقت وكانت أعلى في الماضي. النظرية التي يقترحونها قد تحل مشكلة موجودة في علم الكونيات وتشرح المراحل المبكرة من تطور الكون." نهاية الاقتباس
    ها هو الرابط الالكتروني:
    https://davidson.weizmann.ac.il/online/askexpert/%D7%94%D7%90%D7%9D-%D7%9E%D7%94%D7%99%D7%A8%D7%95%D7%AA-%D7%94%D7%90%D7%95%D7%A8-%D7%A7%D7%91%D7%95%D7%A2%D7%94
    كما ترى فأنا لست وحدي، ومن المحتمل أن تُمنح جائزة نوبل للآخرين.

  194. يهودا
    أين جائزة نوبل الخاصة بك؟ أقترح عليك مراسلة لجنة جائزة نوبل والمطالبة بإلغاء جميع جوائز الفيزياء الممنوحة سابقاً!!!

    بدون أي إهانة، لكن استجابتك سيئة فحسب - فأنت تعتقد أنك أكثر ذكاءً من الفيزيائيين الذين يعملون في هذا المجال منذ عقود.

    لديك نظرية وهمية لا تفسر شيئا وتتناقض مع كل النظريات الموجودة. محرج حقا….

    مرة أخرى، لا جريمة. عموما أنا أقول رأيي.

  195. آه، كم هو سذاجة المساحين الذين يستخدمون ثوابت هي للأسف ليست ثابتة
    سرعة الضوء ليست ثابتة وتتغير بحوالي 1 سم في الثانية سنويًا، ولا يمكن لأي تجربة لميكلسون مورلي أن تناقض ذلك لأن التجربة المذكورة أعلاه تم إجراؤها في وقت ما في مكان ما في نهاية القرن التاسع عشر. لم يتظاهر أبدًا بقياس سرعة الضوء في أوقات مختلفة ومقارنتها. فمن المستحيل بناءً على قياس واحد أن نقرر أن السرعة ثابتة في جميع الأوقات!
    يتغير الكيلوجرام نفسه لأن ثابت الجاذبية يتغير بسبب توسع الكون، كما أن نفس الكتلة سوف تزن بشكل مختلف في أوقات مختلفة.
    لذلك يمكنك نقش الكيلو بطرق الكوانتوم-الأسطورية-اللوحية الفائقة وبموافقة الحاخامية والبيدز، لكن ذلك لن يساعد، وبمجرد وصولك إلى نتيجة نهائية، سيبدأ كلغك في فقدان وزنه، ببطء ولكن بثبات، أقل قليلاً من نصف ميكروجرام سنويًا. هذا ما هناك!
    وهذا صحيح، أعلم أن الجميع سيهاجمني، لذلك لا أهتم حقًا، لكن سأظل أسأل:-
    يرجى الرد بلطف
    شكر

  196. ليس من الواضح بالنسبة لي ما الذي سيحددون كتلة الكيلوجرام الجديد. حسب الوزن الأصلي بـ 50 ميكروغرام أقل؟ أو ربما حسب وزن معين لكتلة "الشهود"؟ وما هي العواقب على أي حال؟

  197. أبي
    أعتقد أن حل لتر من الماء يمثل مشكلة فنية. عليك أن تتأكد من أن الماء نقي تمامًا، وهذا أمر صعب: فالماء يذيب كل شيء في البيئة، من الهواء إلى أي وعاء تضع فيه الماء.
    بالإضافة إلى ذلك، يحتوي الماء على توتر سطحي يجعل من الصعب جدًا قياس الحجم.

    كانت هناك فكرة مماثلة - استخدام كتلة من ذرات الكربون. بدلًا من تحديد الحجم، افعل شيئًا أبسط، وهو تحديد عدد معين من الذرات. وفي اعتقادي أن هذه الطريقة محدودة بالدقة في حساب الذرات الموجودة في جسم معين (كرة حديدية نقية قطرها 93 ملم).
    هناك طريقة أخرى، ربما تكون أكثر عملية، تعتمد على ثابت بلانك. يربط هذا الثابت بين الكتلة والمسافة والزمن، وبمجرد معرفة حجم الثابت، فإنك تعرف كيفية تحديد وحدة الكتلة. وتبين أن هناك عددًا من التجارب المختلفة التي توفر حجم الثابت بدقة عالية جدًا.

  198. في الماضي، تم تعريف الكيلوجرام بواسطة ثابت أساسي في الكون: 1000 سم مكعب من الماء (عند درجة الحرارة والضغط...)
    ولأسباب عملية تقرر تغيير التعريف ليربطه بقطعة معدنية محفوظة في...
    الآن ترى مدى خطأ ذلك، فلماذا لا نعود إلى التعريف الأصلي؟
    ففي نهاية المطاف، الطول القياسي ثابت بالفعل بواسطة ثابت أساسي في الكون، (طول موجة شعاع الليزر التي يتم إنتاجها من نظير محدد لمادة معينة...) وبالتالي الحجم أيضًا، وبالتالي أيضًا 1000 سم مكعب من الماء.. فما الذي يمكن أن يكون أبسط وأصح من أن أكبر عدد ممكن من القياسات هل ستشير إلى نفس الثابت الأساسي في الكون؟

    كما أن قطع العلاقة بين المول والوزن أمر إشكالي، لأنه عمليا يتم وزن كمية من المادة لمعرفة عدد المولات التي تحتوي عليها، ولا توجد إمكانية عملية لحساب الجزيئات.
    وإذا كان الوزن منسوبًا إلى الماء المنسوب إلى الطول، فسيكون من الممكن دائمًا التحقق من صحة دقة Mol بناءً على معيار الطول الموجي لـ...

ترك الرد

لن يتم نشر البريد الإلكتروني. الحقول الإلزامية مشار إليها *

يستخدم هذا الموقع Akismat لمنع الرسائل غير المرغوب فيها. انقر هنا لمعرفة كيفية معالجة بيانات الرد الخاصة بك.